You are on page 1of 118

Gastrointestinal Disorders

Michelle T. Martin, Pharm.D., BCPS, BCACP


University of Illinois Hospital and Health Sciences System
University of Illinois at Chicago College of Pharmacy
Chicago, Illinois
Gastrointestinal Disorders

Gastrointestinal Disorders
Michelle T. Martin, Pharm.D., BCPS, BCACP
University of Illinois Hospital and Health Sciences System
University of Illinois at Chicago College of Pharmacy
Chicago, Illinois

ACCP Updates in Therapeutics® 2016: Ambulatory Care Pharmacy Preparatory Review and Recertification Course

1-523
Gastrointestinal Disorders

Learning Objectives A. Refraining from food consumption 1 hour


before going to bed.
Gastrointestinal (GI) disorders within the objectives refer B. Eating peppermint after meals.
to the disorders covered in this chapter and include the fol- C. Wearing tight-fitting clothes.
lowing: gastroesophageal reflux disease (GERD), peptic D. Discontinuing smoking.
ulcer disease (PUD), chronic liver disease manifestations,
viral hepatitis, malabsorption syndrome, diarrhea, con- 2. A 67-year-old woman with rheumatoid arthritis
stipation, nausea and vomiting, irritable bowel syndrome takes naproxen 500 mg by mouth daily, metopro-
(IBS), and inflammatory bowel disease (IBD). lol 25 mg by mouth twice daily, aspirin 81 mg by
1. Evaluate and apply national guideline–based treat- mouth once daily, and alendronate 70 mcg by mouth
ment strategies for GI disorders. weekly. Which is best to recommend?
2. Assess the benefits and risk of drug therapy for A. Give lansoprazole 30 mg daily.
patients with GI disorders. B. Give esomeprazole 40 mg twice daily.
3. Recommend appropriate nonpharmacologic and C. Give misoprostol 200 mcg twice daily.
pharmacologic interventions for the management of D. No gastroprotective therapy is necessary.
GI disorders.
4. Develop and implement a patient-specific compre- 3. Which patient would most require counseling on
hensive therapeutic plan for the management of GI the importance of using two forms of contracep-
disorders. tion during hepatitis C virus (HCV) treatment and
5. Review and understand treatment options for for 6 months after completing treatment?
patients whose condition is refractory to conven-
A. A 29-year-old woman receiving sofosbuvir
tional therapies, and determine alternative options
plus ribavirin.
for the management of GI disorders.
B. A 47-year-old woman receiving ombitasvir/
6. Assess the available information to identify non-
paritaprevir/ritonavir plus dasabuvir.
drug factors and drug-related problems that may
C. A 53-year-old woman receiving sofosbuvir
affect response to therapies used in the management
plus daclatasvir.
of GI disorders.
D. A 36-year-old woman receiving ledipasvir/
7. Provide drug-related patient education and coun-
sofosbuvir.
seling for pharmacologic therapies used in the
management of GI disorders.
4. A 52-year old woman with liver cirrhosis of an
8. Make recommendations to manage drug therapy for
unknown etiology presents to the emergency depart-
GI disorders, which may include initiation, modifica-
ment today with increased confusion, change in sleep
tion, or discontinuation of medications as appropriate.
patterns, and decreased ability to function at work.
9. Interpret follow-up patient symptoms, laboratory
Her laboratory values, head computed tomography
results, and diagnostic results to determine whether
(CT) findings, and vital signs are normal. Liver func-
and when adjustments to drug therapy for GI dis-
tion test results are consistent with cirrhosis. The
orders are warranted, and modify patient-specific
model for end-stage liver disease (MELD) score is
plans according to this assessment.
14. Which therapy is best to recommend for treating
her hepatic encephalopathy (HE)?
Self-Assessment Questions A. Rifaximin.
Answers and explanations to these questions may be B. Lactulose.
found at the end of the chapter. C. Flumazenil.
D. Protein restriction.
1. Nonpharmacologic interventions are aimed at
decreasing the incidence of acid reflux and enhancing
esophageal clearance in patients with gastroesophageal
reflux disease (GERD). Which best reflects the option
that would do this?

ACCP Updates in Therapeutics® 2016: Ambulatory Care Pharmacy Preparatory Review and Recertification Course

1-524
Gastrointestinal Disorders

5. A 57-year-old white man with alcoholic liver disease 7. Which best describes the correct population and
(Child-Pugh class B) received initial therapy with methodology recommended for serologic testing for
propranolol 10 mg by mouth twice daily after a immunity after administration of the HBV vaccine?
screening endoscopy 1 month ago. In the clinic today, A. Patients with chronic liver disease: Test for
he appears to be tolerating the propranolol dose and anti–hepatitis B surface antibody (anti-HBs);
reports no signs of light-headedness, fatigue, or test 1–2 months after the last dose of the
shortness of breath. His vital signs from today and his vaccine series.
visit 1 month ago are summarized in the following B. Patients with chronic liver disease: Test for anti–
table. Which drug option is best for this patient? hepatitis B early antigen (anti-HBe); test 3–4
months after the last dose of the vaccine series.
Vital Signs 1 Mo Ago Today
C. Health care workers: Test for anti-HBs; test 1–2
Temperature, °F 98.8 98.7 months after the last dose of the vaccine series.
Blood pressure, mm Hg 130/90 130/83 D. Health care workers: Test for anti-HBe; test 3–4
Respiratory rate, months after the last dose of the vaccine series.
16 15
breaths/minute
Heart rate, 8. You are contacted by one of the new gastrointestinal
89 81 (GI) medical fellows regarding the use of infliximab for
beats/minute
a patient with a history of Crohn disease (CD) and no
A. Change propranolol to nadolol 20 mg by other medical conditions. The patient takes mesalamine
mouth daily. (Pentasa) 1 g by mouth four times daily, azathioprine
B. Add isosorbide mononitrate 10 mg by mouth 100 mg by mouth once daily, esomeprazole 40 mg by
twice daily. mouth once daily, lisinopril 20 mg by mouth once daily,
C. Continue current therapy and reevaluate entecavir 1 mg by mouth once daily, and acetaminophen
in 4 weeks. 325 mg by mouth twice daily. Which recommendation
D. Increase propranolol to 20 mg by mouth is best for you to provide the GI fellow regarding
twice daily. infliximab therapy for this patient?
A. Premedicate with an antihistamine, acetamino-
6. A patient with chronic hepatitis B virus (HBV) infec-
phen, and corticosteroids before the first dose.
tion has taken lamivudine 100 mg by mouth daily
B. Obtain additional information regarding
for 18 months. His HBV DNA became undetectable
this patient’s medical history before prescrib-
after 2 months of therapy, and it had until 6 weeks
ing therapy.
ago remained undetectable. Laboratory data from 6
C. Assess cardiac function by obtaining an echo-
weeks ago are as follows: aspartate aminotransferase
cardiogram before administration.
(AST) 197 IU/mL, alanine aminotransferase (ALT)
D. Order a tuberculosis (TB) test to rule out TB
227 IU/mL, total bilirubin 2.7 mg/dL (indirect 1.9
before administration.
mg/dL, direct 0.8 mg/dL), serum creatinine con-
centration (SCr) 1.1 mg/dL, and HBV DNA 47,600
IU/mL. His HBV DNA value from 1 week ago was
51,200 IU/mL. Which is the best course of action?
A. Add entecavir.
B. Add tenofovir.
C. Discontinue lamivudine and
add tenofovir.
D. Discontinue lamivudine and
add adefovir.

ACCP Updates in Therapeutics® 2016: Ambulatory Care Pharmacy Preparatory Review and Recertification Course

1-525
Gastrointestinal Disorders

I.  GASTROESOPHAGEAL REFLUX DISEASE

A. Definition (according to the most recent 2013 practice guidelines) – Symptoms or complications resulting
from the reflux of gastric contents into the esophagus or beyond, into the oral cavity (including larynx)
or lung. GERD can further be classified as the presence of symptoms without erosions on endoscopic
examination (nonerosive disease or NERD) or symptoms with erosions present (ERD).

B. Pathophysiology (Domain 1, Task 2)


1. Parietal cells secrete acid into the gut lumen in response to three coordinated interacting signals:
Acetylcholine, histamine, and gastrin. The gastroesophageal junction contains a complex anatomic
and physiologic antireflux mechanism that allows gastric contents to move from the stomach into the
intestines. Disruption of the antireflux barrier at the gastroesophageal junction may result in GERD.
Repeated acidic exposure for prolonged periods may result in reflux esophagitis, which may also
progress to erosive esophagitis.
2. Proposed mechanisms of reflux
a. Physiologic: Transient relaxation of the lower esophageal sphincter (LES), hypotensive LES,
and/or anatomic disruption of the junction; often caused by hiatal hernia
b. Pharmacologic agents may induce LES relaxation (e.g., anticholinergics, aminophyllines,
β-adrenergic agonists, benzodiazepines, and nitroglycerines).
c. Increased abdominal esophageal pressure gradient (e.g., obesity, pregnancy)
3. Proposed mechanisms of mucosal damage (i.e., esophagitis)
a. Prolonged esophageal acid clearance times and diminished salivary function
b. Reduced esophageal epithelial defense: Presence of acid disrupts normal pH, which alters
epithelial cell function (cannot volume regulate, so cellular edema occurs; cell proliferation is
stimulated, leading to membrane thickening).
c. Damage in the presence of normal acid exposure caused by heightened esophageal hypersensitivity
4. Studies linking Helicobacter pylori to the pathogenesis of GERD are complicated, and its role
remains undefined.

C. Symptoms (Domain 1, Task 1,3)


1. Often nonspecific and highly variable; do not always correlate with the degree of esophageal
inflammation, but usually correlate with duration of reflux
a. Typical: Heartburn (pyrosis) and regurgitation
b. Atypical: Dyspepsia, epigastric pain, nausea, bloating, and belching
c. Chest pain: Before considering GERD as the cause of chest pain, must distinguish cardiac from
non-cardiac cause
d. May present in a continual or mild intermittent (less than twice weekly) manner
2. GERD classification (according to the Montreal consensus panel) divides GERD-related disease
manifestations into a set of syndromes: esophageal and extraesophageal.
a. Esophageal syndrome
i. Symptomatic syndrome: Symptoms without evidence of esophageal injury
(a) Typical reflux: Heartburn (pyrosis), regurgitation, and acidic taste. May worsen asthma
severity, causing laryngitis and chronic cough
(b) Reflux chest pain syndrome: Chest pain indistinguishable from ischemic cardiac pain
caused by GERD

ACCP Updates in Therapeutics® 2016: Ambulatory Care Pharmacy Preparatory Review and Recertification Course

1-526
Gastrointestinal Disorders

ii. Esophageal injury syndrome: Troublesome esophageal symptoms in addition to esophageal injury
(a) Reflux esophagitis
(b) Reflux stricture
(c) Barrett esophagus
(d) Esophageal adenocarcinoma
b. Extraesophageal syndrome: GERD symptoms, with GERD as a likely contributing etiology but
seldom the sole cause
i. Established associations
(a) Reflux cough syndrome
(b) Reflux laryngitis syndrome
(c) Reflux asthma syndrome
(d) Reflux dental erosion syndrome
ii. Proposed associations
(a) Pharyngitis
(b) Sinusitis
(c) Idiopathic pulmonary fibrosis
(d) Recurrent otitis media

D. Diagnosis (Domain 1, Task 1; Domain 4, Task 3)


1. GERD must be differentiated from the many other conditions involving the esophagus because several
diseases have overlapping symptoms.
2. Primary diagnostic method includes clinical history with presenting symptoms, together with a review
of potential triggers.
3. Diagnostic tests to confirm GERD are usually done by a gastroenterologist. Tests typically conducted
in patients in certain situations
a. Lacks response to empiric therapy
b. Presents with alarm features (see Table 1) that may suggest an alternative diagnosis. Refer patient
to a gastroenterologist for diagnostic testing. Evidence using alarm features as a diagnostic tool is
inconclusive; however, many clinicians use alarm features as part of their screening process.
c. Presents with symptoms that are chronic or suggestive of a more serious condition (i.e.,
extraesophageal syndrome)

Table 1. Gastroesophageal Reflux Disease Alarm Features


Anemia
Chest pain
Choking
Symptoms requiring
Epigastric mass
immediate referral to
GI bleeding
a gastroenterologist
Troublesome dysphagia (not improved with PPI therapy or requiring altered eating patterns)
Vomiting
Weight loss (unintentional)
GI = gastrointestinal; PPI = proton pump inhibitor.

ACCP Updates in Therapeutics® 2016: Ambulatory Care Pharmacy Preparatory Review and Recertification Course

1-527
Gastrointestinal Disorders

4. Diagnostic tests (Domain 1, Task 3)


a. Upper endoscopy (with or without biopsy) is a first-line diagnostic test performed to visualize the
esophageal mucosa.
i. Indicated for alarm features and screening of patients at high risk of complications, including
white men older than 50 years, patients who are overweight with chronic GERD symptoms
ii. Findings diagnostic of GERD include erosive esophagitis, strictures, and those consistent with
Barrett esophagus.
iii. Only method to diagnose Barrett esophagus; biopsy identifies epithelium for dysplasia
iv. Highly specific for the diagnosis of GERD; however, a normal esophagogastroduodenoscopy
(EGD) does not exclude GERD
b. Ambulatory esophageal pH monitoring determines the percentage of time the esophageal pH is
less than 4 during a 24-hour period.
i. Identifies individuals with elevated esophageal acid to correlate acid exposure with symptoms
ii. Reproducibility (84%–93%) and 96% sensitivity/specificity
iii. Results are more accurate when proton pump inhibitors (PPIs) are held for 7 days before
testing; consider performing test before PPI trial.
c. Esophageal manometry
i. Not used for the diagnosis of GERD
ii. Evaluates esophageal peristaltic function in those who are surgical candidates to rule out
major motor disorders and assists in determining the best surgical procedure
d. Barium swallow: Not used for the diagnosis of GERD; used for evaluating dysphagia

E. Treatment (Domain 1, Task 3,4,6,7; Domain 2, Task 5)


1. Goals
a. Reduce frequency and duration of reflux.
b. Reduce and eliminate GERD symptoms.
c. Prevent disease progression and the development of complications.
d. Promote the healing of injured gastrointestinal (GI) mucosa.
2. Target: Reverse pathophysiology to decrease the acidity of the refluxate, enhance esophageal acid
clearance, improve gastric emptying time, increase LES pressure, and protect from further gastric
mucosal damage.
3. Strategies are based on duration of use.
a. On-demand or self-directed
b. Intermittent/short term
c. Continuous
4. Strategies (nonpharmacologic and/or pharmacologic) depend on patient condition, symptom intensity,
degree of inflammation, and presence of complications.
5. Lifestyle interventions are the initial step in managing GERD and are aimed at lessening the incidence
of reflux and enhancing esophageal acid clearance. Although used clinically, evidence is insufficient to
advocate these approaches as monotherapy in all patient populations (Figure 1).
a. Pharmacists are well positioned to counsel and educate patients on lifestyle modifications.
b. According to the guidelines, targeted populations may benefit from the following
nonpharmacologic interventions.
i. Dietary modifications
(a) Avoid reflux-inducing foods/beverages (e.g., alcohol, acidic foods, caffeine, chocolate,
fatty foods, garlic, onion, peppermint, spicy foods).
(b) Avoid supine positions post-meal consumption.
(c) Avoid consuming food (within 2–3 hours) before bedtime.
(d) Reduce portion size.

ACCP Updates in Therapeutics® 2016: Ambulatory Care Pharmacy Preparatory Review and Recertification Course

1-528
Gastrointestinal Disorders

ii. Smoking cessation


iii. Avoid wearing tight-fitting garments.
iv. Avoid medications that may precipitate or induce GERD (e.g., anticholinergics, barbiturates,
dihydropyridine calcium channel blockers, dopamine, estrogen, γ-aminobutyric acid [GABA]
agonists, nitric oxide inhibitors, progesterone, serotonin antagonists, tetracycline, theophylline).
v. Elevate head of bed (6–8 inches).
vi. Weight loss for individuals who are overweight or who have obesity
vii. Chew gum or use oral lozenges.

Typical GERD symptoms

If NO Alarm Symptoms
If Alarm Symptoms
1.  Lifestyle modifications
2.  Antacids/H2RA as needed

Relief in 2 weeks?
NO YES
Diagnostic Evaluation Referral to PCP
(referral to specialist) If Alarm Symptoms
If NO Alarm Symptoms 1. Continue lifestyle modifications
2. On demand or intermittent use
Initiate PPI therapy of OTC products
NO
Relief in 8 weeks? YES
Taper PPI to achieve:
1.  Lowest effective dose
2.  Therapy discontinuation

Figure 1. GERD management algorithm for ambulatory care pharmacists.


GERD = gastroesophageal reflux disease; H2RA = histamine-2 receptor antagonist; OTC = over-the-counter; PCP = primary care physician; PPI = proton
pump inhibitor.
Adapted from: Katz PO, Gerson LB, Vela MF. Guidelines for the diagnosis and management of gastroesophageal reflux disease. Am J Gastroenterol
2013;108:308-28 (2013 GERD Practice Guidelines).

6. Guideline recommendations: American Gastroenterological Association (AGA) (Am J Gastroenterol


2013;108:308-28) (Domain 2, Task 1,5; Domain 3, Task 2)
a. Esophageal syndromes
i. Symptomatic syndrome (mainly managed by primary care physician)
(a) Typical reflux syndrome
(1) Empiric therapy with antisecretory drugs; standard-dose PPIs are more effective
and are preferred to histamine-2 receptor antagonists (H2RAs). Continue therapy for
4–8 weeks and then reevaluate.
(2) Maintenance therapy is unclear for nonerosive disease; guidelines recommend PPI
administration for those who continue to have symptoms after PPI discontinuation.
(3) Intermittent symptoms: Treat with rapid-acting drugs, including antacids, which are
the quickest agents and may be combined with H2RAs and PPIs.
(b) Reflux chest pain syndrome
(1) Rule out ischemic heart disease.
(2) Diagnostic evaluation recommended before initiation of empiric PPI therapy

ACCP Updates in Therapeutics® 2016: Ambulatory Care Pharmacy Preparatory Review and Recertification Course

1-529
Gastrointestinal Disorders

ii. Esophageal injury


(a) Continuous standard-dose PPI therapy twice daily is recommended.
(b) Continuous PPI is recommended for maintenance therapy to maintain healed mucosa;
discontinuation will likely cause recurrence (on-demand therapy is NOT recommended).
b. Extraesophageal syndromes (established and proposed association)
i. GERD may be a contributing etiologic factor, but because accurate confirmatory testing is
lacking, GERD is diagnosed and treated empirically.
(a) Data support treatment when symptomatic reflux is present.
(b) Empiric therapy with PPI twice daily is commonly initiated. Data from open-label or
uncontrolled pH monitoring studies with twice-daily dosing showed normalization of pH,
which is the likely rationale for this dosing. Length of therapy is unclear.
ii. Maintenance therapy: Continuous PPI therapy should be based on the presence and severity
of symptoms. For those who require chronic therapy, administer at the lowest effective dose,
including on-demand or intermittent therapy.
7. Pharmacologic therapies include antacids, H2RAs, and PPI therapy.
a. Typically, one of the following two approaches for pharmacologic intervention is used:
i. Step-up: Initiate lifestyle modifications and gradually increase pharmacologic intervention if
necessary; for example, start with the less-potent H2RAs, and then use the more potent PPIs
for failures.
ii. Step-down: Initiate lifestyle modifications and start therapy with the most potent agents;
initiate the PPIs for rapid symptom control, and then try in a stepwise manner to decrease
the intervention, switching to an H2RA.
b. Antacids
i. Neutralize acid and raise intragastric pH; results in a decreased conversion of pepsinogen to
pepsin and increased LES pressure
ii. Place in therapy
(a) Readily available (over the counter [OTC]) to treat present symptoms and provide quick relief
(b) First-line treatment for mild intermittent (less than twice weekly) or breakthrough
symptoms for patients receiving H2RA or PPI therapy
(c) Not appropriate for chronic symptoms or for healing esophageal erosions
iii. Agents
(a) Aluminum, calcium, and magnesium antacid preparations.
(b) Antacid with alginic acid (Gaviscon) forms a viscous layer that floats on the surface of
gastric contents as a protective barrier, reducing the incidence of reflux episodes.
(c) Short duration of action requires several administrations throughout the day.
(d) Nighttime acid suppression may not be maintained.
iv. Dosing
(a) Varies depending on formulation; usually three or four times daily or intermittently,
depending on symptoms
(b) Take 1–3 hours after meals and other medications to avoid potential drug interactions
(e.g., chelation, increased/decreased absorption).
v. Adverse effects
(a) Fairly well tolerated, minimal adverse effects when used as labeled
(b) Common adverse effects include constipation (especially with products containing
aluminum), chalky taste, abdominal cramps, and diarrhea (especially with products
containing magnesium).
(c) Caution in renal dysfunction because of accumulation/electrolyte disturbances

ACCP Updates in Therapeutics® 2016: Ambulatory Care Pharmacy Preparatory Review and Recertification Course

1-530
Gastrointestinal Disorders

c. H2RAs
i. Block parietal cell acid secretion by reversible H2RA blockade
ii. Place in therapy
(a) Empiric therapy for mild GERD symptoms
(b) Maintenance therapy for those without erosive disease with intermittent symptoms (less
effective than PPIs in healing erosive esophagitis)
(c) Add-on therapy for those with incomplete PPI therapy response; use as on-demand
therapy for intermittent symptoms that may be provoked by food consumption or exercise
and/or at bedtime. Effective for these symptoms because of OTC availability and quick
onset of action (peak in 1–2 hours)
iii. Agents
(a) Cimetidine (Tagamet), famotidine (Pepcid), nizatidine (Axid), and ranitidine (Zantac)
(b) Similar efficacy across class; differences can be overcome by increasing the dose
(c) All available as prescription and OTC
(d) Low cost; generics available for all prescription products
(e) Pepcid Complete, an H2RA and antacid combination product (famotidine, calcium
carbonate, and magnesium oxide), is available. This product combines two classes of
drugs with different onsets of action; antacids work within a few minutes but are short
acting, whereas H2RAs have a longer onset of action and last longer. This product may be
especially beneficial for those who have symptoms after food consumption.
iv. Dosing
(a) OTC dose is usually one-half the entire prescription dose. Each product’s dosing and
length of therapy vary depending on the severity of symptoms; see product labeling for
details.
(b) OTC products should not be used for more than 2 weeks without consulting a health care
practitioner. Pharmacist should discuss length of therapy when obtaining GERD history
from patients.
(c) Prolonged use may lead to reduced efficacy and tachyphylaxis.
(d) Dosage reductions are recommended for patients with renal impairment.
v. Adverse events
(a) Well tolerated
(b) The most common adverse events are central nervous system (CNS) effects, including
headache, dizziness, fatigue, and confusion; increased incidence in patients with renal
impairment and in older adults; constipation, diarrhea
(c) Gynecomastia occurs with cimetidine in a dose- and time-dependent fashion; incidence is
rare.
vi. Drug interactions
(a) Mechanism of action of drugs requiring low gastric pH for absorption may be impaired
(e.g., ketoconazole, itraconazole, HIV protease inhibitors [PIs] [e.g., atazanavir]).
(b) Cimetidine inhibits the cytochrome P450 (CYP) enzyme system—specifically, 1A2,
2C9, 2C19, 2D6, 2E1, and 3A4. Concomitant drugs also metabolized by these enzyme
systems may be affected (i.e., cyclosporine, theophylline, clopidogrel, phenytoin, and
warfarin). The more recently developed H2RAs are less likely to alter CYP metabolism,
with ranitidine being a less potent inhibitor than cimetidine and famotidine; therefore,
clinically significant problems are seldom encountered. See prescribing guidelines for
more information regarding established and theoretical drug interactions.
(c) Clinicians should discuss the potential for drug interactions with their patients and
educate patients to notify them regarding any new medications or changes to current
medications so that patient-specific risks may be evaluated.

ACCP Updates in Therapeutics® 2016: Ambulatory Care Pharmacy Preparatory Review and Recertification Course

1-531
Gastrointestinal Disorders

d. PPIs
i. Weak protonable pyrimidines; accumulate in the acidic region of parietal cells, where they
undergo an acid-catalyzed conversion to a reactive species; rate of conversion varies between
agents. The reactive species irreversibly interacts with the hydrogen potassium adenosine
triphosphate (H-K-ATPase) pump, resulting in a long-lasting impairment in acid secretion.
ii. Place in therapy and dosing
(a) Empiric therapy for individuals with frequent, continual symptoms: Standard once-daily
dosing for 8 weeks
(1) Most potent inhibitors of acid suppression available; superior to H2RAs for
patients with moderate to severe GERD, erosive esophagitis, and GERD-related
complications
(2) Symptom relief is delayed compared with antacids and H2RAs because of longer
onset of action.
(3) Not indicated for episodic, intermittent symptoms
(b) Maintenance therapy: Administer for persistent symptoms and in patients with
complications (e.g., erosive esophagitis, Barrett esophagus). Chronic therapy should be
administered at the lowest effective dose, including on-demand and intermittent strategies.
(c) Individuals with partial or incomplete response: Consider tailoring therapy in the
following way:
(1) Assessing therapy adherence
(2) Adjusting the timing of dose and/or increasing dosing to twice daily, especially for
patients with variable schedules, nighttime symptoms, or sleep disturbances. For
divided dosing, administer second dose before evening meal.
(3) Switching to a different PPI. Clinical trials show similar efficacy among agents, so
it is unclear whether this results in clinically significant improvements.
(d) Nonresponders: See GI specialists for additional evaluation.
iii. Effectiveness
(a) All are similar in efficacy when used for GERD symptoms.
(b) Selection based on prescription plan coverage, formulation, generic availability,
and adverse effects
iv. Administration and formulations
(a) Traditional, delayed-release PPIs administered 30–60 minutes before the first meal of the
day because the availability of H-K-ATPase (proton pump) is greatest after a prolonged
fast. During a meal, not all parietal cells are active; thus, administering PPIs during
this time correlates with suboptimal outcomes. Exceptions may be dexlansoprazole and
omeprazole plus sodium bicarbonate (see Table 2).
(1) Dexlansoprazole (Dexilant): Dual delayed release; independent of food intake and may
be dosed any time of the day
(2) Omeprazole plus sodium bicarbonate (Zegerid): Immediate release; effective to
control nocturnal symptoms when administered at bedtime. May have slightly
quicker onset of action than other products
(b) Do not administer in combination with H2RAs, prostaglandins, or other antisecretory
agents because of the duplicate effects on acid suppression. PPIs may be administered
with H2RAs when the time interval between doses is sufficient. Exact interval is
unknown but recommended if PPI taken early in the day before first meal; H2RAs may be
taken in the evening before bedtime (for nocturnal breakthrough symptoms); however, no
evidence supporting efficacy

ACCP Updates in Therapeutics® 2016: Ambulatory Care Pharmacy Preparatory Review and Recertification Course

1-532
Gastrointestinal Disorders

(c) Alternative administration techniques such as suspensions or through a nasogastric tube


are available for most products. See prescribing guidelines for additional information.
(d) Intravenous formulations: Esomeprazole and pantoprazole
(e) See Box 1 for patient counseling points on PPI administration.

Table 2. Comparison of Proton Pump Inhibitors


Agent Available Formulations Available OTC
Oral—Delayed-release capsule (20 and 40 mg)
Esomeprazole sodium
Oral—Delayed-release suspension (10-, 20-, and 40-mg packets) Yes
(Nexium)
IV solution (20- and 40-mg vials)
Esomeprazole strontium Oral – Delayed-release capsule (49.3 mg) No
Oral—Delayed-release capsule (10, 20, and 40 mg)
Omeprazole (Prilosec) Oral—Delayed-release tablet (20 mg) Yes
Oral—Immediate-release suspension (20- and 40-mg packets)
Oral—Capsule: 20 or 40 mg of omeprazole and 1100 mg of sodium
Omeprazole + sodium bicarbonate (contains sodium 304 mg [13 mEq] per capsule)
Yes
bicarbonate (Zegerid) Oral suspension: 20 or 40 mg of omeprazole and 1680 mg of sodium
bicarbonate (contains sodium 460 mg [20 mEq] per packet)
Lansoprazole (Prevacid) Oral—Delayed-release capsule (15 and 30 mg)
Lansoprazole (Prevacid) Oral—Delayed-release disintegrating tablet (15 and 30 mg) Yes
Oral suspension: 3 mg/mL powder
Oral—Delayed-release enteric-coated tablet (20 mg)
Rabeprazole (AcipHex) No
Oral—Delayed-release capsule (5 and 10 mg)
Oral—Delayed-release enteric-coated tablet (20 and 40 mg)
Pantoprazole (Protonix) IV solution (40 mg/vial) Yes
Granules (40 mg/packet)
Dexlansoprazole
Oral—Delayed-release capsule (30 and 60 mg) No
(Dexilant)
IV = intravenous; OTC = over the counter.

Box 1. Proton Pump Inhibitor Therapy Administration—Patient Counseling


Most products work most effectively when administered 30–60 min before a meal
—Dexlansoprazole can be administered without regard to meals
—Omeprazole + sodium bicarbonate should be administered right before bedtime when used for nocturnal
systems
If taking once daily, best to take before breakfast
If taking twice daily, best to take 30–60 min before breakfast and 30–60 min before evening meal
Capsules and tablets are delayed-release formulations – Should not be crushed or chewed
Patients with swallowing difficulties:
—Open delayed-release capsules and sprinkle contents on applesauce
—Liquid formulations are available for some products

v. Adverse events
(a) May include headache, dizziness, somnolence, diarrhea, constipation, and nausea. For
patients having an adverse event, consider changing to a different agent. Although
common in clinical practice, supporting data are limited

ACCP Updates in Therapeutics® 2016: Ambulatory Care Pharmacy Preparatory Review and Recertification Course

1-533
Gastrointestinal Disorders

(b) Rebound hypersecretion: Reappearance of acid-related symptoms (i.e., heartburn, reflux,


and dyspepsia) when acid-suppressive therapy is discontinued in individuals receiving
therapy for 2 weeks or more
(1) Often leads to the resumption of PPI therapy
(2) Pharmacists should advise those discontinuing therapy to taper off, lower the dose,
and/or decrease the frequency. Taper according to patient symptoms and tolerability;
often takes 4–6 weeks for complete discontinuation
(c) Vitamin B12 deficiency: Initial step in cobalamin absorption requires gastric acid and
pepsin. Clinical evidence is lacking. However, potentially more likely to develop in older
adults or institutionalized long-term PPI users, and should be considered in this cohort
(d) Hip fractures and osteoporosis: Decreased calcium absorption, leading to increased risk
of fracture
(1) May 2010: The U.S. Food and Drug Administration (FDA) issued a warning and
required labeling for all prescription and OTC PPIs to include information regarding
the increased risk of fracture (hip, wrist, and spine) (www.fda.gov/Safety/MedWatch/
SafetyInformation/SafetyAlertsforHumanMedicalProducts/ucm213321.htm).
(2) According to current guidelines, patients with osteoporosis may continue PPI
therapy. Evidence is insufficient to recommend bone density screening or calcium
supplementation for patients receiving PPI therapy. Concern for hip fractures and
osteoporosis should not affect decision for chronic PPI therapy except in those with
other risk factors for hip fracture.
(e) Hypomagnesium: Low magnesium concentrations. March 2011: FDA drug safety
communication stated that prescription PPI drugs may cause low serum magnesium
concentrations if taken for prolonged periods (in most cases, greater than 1 year).
According to the report, in about one-fourth of the cases reviewed, supplemental
magnesium alone did not correct the hypomagnesium, and the PPI had to be
discontinued. The mechanism of this adverse event is not clearly defined. The FDA
required manufacturers to update package labeling to include the potential risk of
hypomagnesia (www.fda.gov/Drugs/DrugSafety/ucm245011.htm).
(f) Association with community-acquired infections including diarrhea and pneumonia
(1) The effect of PPIs in increasing gastric pH levels may encourage the growth of gut
microflora and increase the susceptibility of certain organisms, including Clostridium
difficile (www.fda.gov/Drugs/DrugSafety/DrugSafetyPodcasts/ucm291058.htm).
Because of the potential for PPI therapy to increase the risk of C. difficile infection,
therapy should be used with caution. Pharmacists should advise patients to seek medical
attention if they have symptoms associated with C. difficile infection (i.e., persistent
watery stool, fever, and abdominal pain).
(2) Reports show an increased risk of community-acquired pneumonia with short-term
PPI use; however, guidelines recommend that PPI therapy not be withheld because of
this potential risk.
(g) PPI use was found to be associated with a higher risk of incident chronic kidney disease
(CKD) in the Atherosclerosis Risk in Communities study.
vi. Drug interactions
(a) Drugs with pH-dependent absorption may be altered (e.g., ketoconazole, itraconazole,
HIV PIs).
(b) Drugs that are metabolized by CYP 2C19 or 3A4 substrates
(c) Rilpivirine coadministration is contraindicated.
(d) Clopidogrel (J Am Coll Cardiol 2010;56:2051-66)

ACCP Updates in Therapeutics® 2016: Ambulatory Care Pharmacy Preparatory Review and Recertification Course

1-534
Gastrointestinal Disorders

(1) Potential for reduced effectiveness when administered with PPI because of
CYP2C19-mediated inhibition of conversion of clopidogrel to its active metabolite.
Pharmacokinetic and pharmacodynamic data suggest varying degrees of CYP2C19
inhibition among PPIs; to date, there is no evidence that these differences translate to
differences in clinical outcomes.
(2) In 2009, the FDA issued a warning suggesting that clopidogrel in combination with
omeprazole, esomeprazole, or lansoprazole be avoided because of the potential for
increased adverse cardiovascular (CV) events (www.fda.gov/Safety/MedWatch/
SafetyInformation/SafetyAlertsforHumanMedicalProducts/ucm190848.htm).
(3) Since the 2009 report, findings from additional randomized controlled studies using
all PPIs except dexlansoprazole found no increased risk of CV adverse events;
therefore, the current guidelines state that altering PPI therapy when clopidogrel is
used concomitantly is unnecessary.
vii. PPI use is strongly justified and well tolerated; studies to date have shown minimal
risk and no evidence to warrant routine precautions (i.e., bone density studies, calcium
supplementation). These potential problems related to long-term use provide opportunities for
pharmacists to reevaluate the need for PPI therapy and to encourage use of the lowest dose for
the shortest duration possible in individuals who require therapy.
8. Refractory GERD
a. 10%–40% of patients do not respond to standard-dose PPI therapy and continue to have GERD
symptoms and/or endoscopic evidence of esophagitis.
b. Risk factors for lack of symptom control include longer disease duration, hiatal hernia,
extraesophageal symptoms, and lack of adherence.
c. Evaluate reasons for PPI failure, including improper administration, adherence to regimen,
esophageal hypersensitivity, nocturnal acid breakthrough, characteristics of specific PPIs (e.g.,
bioavailability, metabolism), status, delayed esophageal healing, residual acid reflux, bile acid
reflux, and comorbid conditions.
d. Recommended management of refractory GERD
i. Optimizing antisecretory therapy (e.g., administration instructions, increase dose, changing
PPIs, add-on therapy at bedtime)
ii. If typical symptoms persist despite therapy optimization, upper endoscopy is recommended
to exclude non-GERD etiologies. Specific therapies directed at identified abnormalities
should be implemented. If endoscopic evaluation yields negative findings, ambulatory reflux
monitoring should be performed.
iii. If atypical symptoms persist despite therapy optimization, other etiologies should be assessed.
Referral to other specialists (e.g., ENT, pulmonary, allergist) is recommended. Specific
therapies directed at other issues identified should be implemented. If specialists’ evaluations
yield negative results, ambulatory reflux monitoring should be performed.
iv. Those with refractory symptoms for whom all testing is negative should discontinue PPI therapy.
e. Additional options, including medications and surgery, are available; however, few clinical trials
have evaluated their role for individuals with refractory symptoms.
i. Baclofen: GABA agonist
(a) Reduces LES relaxations and reflux episodes in patients with objective documentation of
continued symptoms, despite optimal PPI therapy
(b) Dose: 5–20 mg three times daily
(c) Adverse events: Dizziness, somnolence, and constipation, which limit use
(d) Long-term data on the efficacy of baclofen for GERD are lacking.

ACCP Updates in Therapeutics® 2016: Ambulatory Care Pharmacy Preparatory Review and Recertification Course

1-535
Gastrointestinal Disorders

ii. Promotility agents are used to improve gastric emptying in select patients, typically in
addition to acid-suppressive therapy.
(a) Bethanechol: Cholinergic agonist
(1) Stimulates gastric motility, increases gastric tone, and restores peristalsis
(2) Dose: 25 mg/dose four times daily; 1 hour before or 2 hours after meals
(3) Adverse events: Blurred vision, headache, tachycardia, abdominal cramping, and
diarrhea. Poor patient tolerability; thus, use of this agent is limited
(b) Metoclopramide: Dopamine antagonist; prokinetic
(1) Dose 10–15 mg/dose up to four times daily; 30 minutes before meals; dose
adjustment necessary for renal impairment. Available as a 5- and 10-mg orally
disintegrating tablet (Metozolv ODT)
(2) Therapy for greater than 12 weeks is not recommended because of the risk of
irreversible tardive dyskinesia.
(3) Adverse events: Confusion, dizziness, drowsiness, fatigue, headache,
hyperprolactinemia, and extrapyramidal symptoms
(c) Cisapride: Cholinergic agonist
(1) Withdrawn from the market in 2000 because of interactions with drugs that are
metabolized by CYP3A4, resulting in significant adverse events (cardiac arrhythmia
and death). Restricted availability in the United States through Janssen’s Limited
Access Program
(2) Increases LES pressure and lower esophageal peristalsis, accelerating gastric emptying
(3) Dose: 5–10 mg four times daily; 15 minutes before meals and at bedtime
iii. Treatment of bile acid reflux
(a) Bile acid is one component of “nonacid” reflux that could contribute to GERD symptoms;
data are weak.
(b) Data regarding the use of cholestyramine or sucralfate to reduce bile reflux are
insufficient to justify use.
iv. Surgical options: Should be considered in young patients requiring high-dose therapy for
symptom control and with a condition unresponsive to therapy (lifestyle modifications and
pharmacotherapy). Preoperative ambulatory pH monitoring is mandatory for all patients
without erosive esophagitis. Specific anatomic-, patient-, and symptom-related factors are
influential in determining which surgery is performed. There is no one best surgical method
for all; however, fundoplication is the most commonly performed procedure. Bariatric surgery
should be considered in patients with obesity having refractory GERD.
9. Role of the pharmacist (Domain 1, Task 5; Domain 2, Task 1,5; Domain 5, Task 2)
a. Because of their accessibility, pharmacists are likely to be the first contact for individuals having
GERD-related symptoms in the ambulatory care setting. Pharmacists can assist with selecting
appropriate OTC empiric therapy and provide education regarding effective and safe use of
antisecretory therapies.
b. Pharmacists should also be able to identify individuals with alarm symptoms (see Table 1) and
those using OTC therapies for more than 2 weeks who require referral to a GI specialist.

ACCP Updates in Therapeutics® 2016: Ambulatory Care Pharmacy Preparatory Review and Recertification Course

1-536
Gastrointestinal Disorders

Patient Case

1. A 40-year-old man with type 2 diabetes and hypertension presents with a 6-week history of intermittent
regurgitation occurring about every other day and an acidic taste in his mouth. He takes metoprolol 100
mg once daily and states that his diabetes is controlled by diet. He avoids chocolate and spicy foods, sleeps
with his head elevated on a wedge pillow, and uses OTC famotidine 10 mg when symptoms intensify and
when he remembers. He reports that he rarely takes it before eating; instead, he usually takes it only once the
symptoms are present and do not dissipate. The symptoms have been so significant that he has not slept and
has missed 2 days of work recently. Which is the best course of action to address his symptoms?
A. Administer metoclopramide 10 mg four times daily.
B. Administer omeprazole 20 mg/day.
C. Continue famotidine 10 mg, but take on a scheduled frequency of four times daily.
D. Continue famotidine, but increase dose to 20 mg, scheduled three or four times daily.

II.  PEPTIC ULCER DISEASE

A. Definition
1. Disease of the upper GI tract characterized by mucosal damage caused by pepsin and gastric acid secretion
2. About 500,000 Americans develop peptic ulcer disease (PUD) each year, with the highest percentage
in those 25–64 years of age. Widespread use of PPIs has led to a slight decline in PUD.
3. Natural history of disease is highly variable, from resolution without intervention to the development
of complications leading to significant morbidity and mortality.

B. Pathophysiology (Domain 1, Task 2)


1. The basis of ulcer formation is multifactorial; normal mucosal defense and healing mechanisms are
disrupted in the presence of gastric acid and pepsin.
2. Most ulcers develop in the stomach and upper duodenum; less often in the esophagus, distal
duodenum, or jejunum
3. Common causes include H. pylori and nonsteroidal anti-inflammatory drug (NSAID) use; other,
less common causes include hypersecretion of gastric acid (Zollinger-Ellison syndrome), infections
(cytomegalovirus, herpes simplex, TB), hypercalcemia, neoplasia, and idiopathic ulcers. Incidence
varies with ulcer type, geographic location, age, sex, and environmental and therapeutic factors.
a. H. pylori is the source of a common worldwide infection, with an estimated 30%–40% of Americans
infected. Only about 10%–15% of infected individuals develop PUD caused by H. pylori. Studies
report that the eradication of H. pylori provides remission and decreases recurrence rates.
b. Ulcer rate is 25% with long-term NSAID user, resulting in more than 100,000 hospital admissions
and 7000–10,000 deaths per year (Am J Gastroenterol 2009;104:728-38).

ACCP Updates in Therapeutics® 2016: Ambulatory Care Pharmacy Preparatory Review and Recertification Course

1-537
Gastrointestinal Disorders

C. Diagnosis (Domain 1, Task 1; Domain 4, Task 3)


1. Clinical presentation
a. General symptoms: Heartburn, abdominal fullness and cramping, epigastric discomfort/pain,
anorexia, weight loss; nocturnal pain may awaken patients from sleep
b. Symptoms and ulcer appearance vary depending on ulcer location.
i. Gastric ulcer: Nausea, vomiting, and anorexia more common; ingestion of food typically
precipitates pain. Ulcers within the stomach are usually concave.
ii. Duodenal ulcer: Onset of pain typically 1–3 hours post-ingestion of food; often relieved by
food. Ulcers within the intestines are usually convex.
c. Complications: Upper GI bleed (melena or hematemesis), perforation and/or penetration (sharp,
sudden pain), and gastric outlet obstruction (bloating, anorexia, nausea, weight loss, and vomiting)
2. Routine laboratory tests are usually not beneficial; hematology laboratory tests (e.g., red blood cell
count, hemoglobin, or hematocrit) may help identify bleeding.
3. Testing for H. pylori infection
a. Indicated for active PUD, history of documented peptic ulcer, or mucosa-associated lymphoid
tissue lymphoma. If infection is present, eradication therapy should be initiated.
b. Diagnostic tests for H. pylori (Am J Gastroenterol 2007;102:1808-25)
i. Selection of test depends on whether patient requires an EGD, an understanding of the
strengths and weaknesses of each test, and the associated costs. There is no gold standard.
ii. Endoscopic testing
(a) Rapid urease test (RUT)
(1) Detect active H. pylori infection by testing for urease activity; urease is metabolized
to ammonia and bicarbonate, producing a change in pH that leads to a pH-sensitive
change in color.
(2) Greater than 90% sensitive and 95% specific
(3) Sensitivity may be decreased by 25% with the use of medications that reduce urease
activity (i.e., bismuth-containing compounds, antibiotics, or PPIs). Recommended to
discontinue agents at least 1–2 weeks before testing
(4) Acute bleeding may decrease sensitivity.
(5) Low-cost, rapid, and simple test; however, clinical utility has declined because of
widespread empiric PPI use for upper GI symptoms
(b) Histology
(1) Because H. pylori prevalence and density vary throughout the stomach, several biopsy
samples from different sites should be obtained to provide an accurate diagnosis.
(2) Sensitivity and specificity greater than 95%
(3) Sensitivity is affected by medications that decrease urease activity (i.e., bismuth-
containing compounds, antibiotics, or PPIs), similar to RUT.
(4) Use in clinical practice is limited because of the high degree of sampling error and
widespread use of PPIs.
(c) Tissue culture
(1) Provides method to identify infection as well as to characterize antimicrobial
sensitivities
(2) 100% specific but time-consuming and costly
(3) Determination of antibiotic resistance by nonculture methodologies is under development.

ACCP Updates in Therapeutics® 2016: Ambulatory Care Pharmacy Preparatory Review and Recertification Course

1-538
Gastrointestinal Disorders

(d) Polymerase chain reaction (PCR)


(1) Technique for rapid DNA amplification
(2) Highly specific and sensitive; one study found H. pylori in 20% of gastric biopsy
specimens by PCR that were not identified by histology
(3) Not yet available commercially
iii. Nonendoscopic
(a) Serum antibody testing
(1) Identifies immunologic reaction to H. pylori infection; detects immunoglobulin G to
H. pylori by ELISA (enzyme-linked immunosorbent assay)
(2) Rapid results, low cost, and wide availability, but lower sensitivity (76%–84%) and
specificity (79%–90%)
(3) Cannot differentiate between active infection and past exposure, so cannot test for
eradication after treatment
(b) Urea breath test (UBT)
(1) Detects active H. pylori infection by urease identification; in the presence of
infection, the ingestion of 13C- or 14C-radiolabeled urea results in labeled CO2,
which can be measured in expired breath
(2) Sensitivity and specificity greater than 95%
(3) Allows accurate posttreatment testing
(4) Medications interfering with urease activity (e.g., bismuth-containing compounds,
antibiotics, and PPIs) decrease test sensitivity.
(5) Bismuth-containing medications and antibiotics should be held for at least 28 days
and PPIs for 7–14 days before test.
(c) Fecal antigen test (FAT)
(1) Identifies the presence of H. pylori in the stool by monoclonal or polyclonal
antibody testing
(2) Allows accurate posttreatment testing; polyclonal test is less well validated than UBT
(3) High sensitivity and specificity both before and after treatment; may be reduced by
bismuth-containing compounds, antibiotics, and PPIs. Recommended to discontinue
agents at least 1 week before testing
iv. Diagnostic test summary
(a) Endoscopic: RUT recommended in those not taking a bismuth-containing agent,
antibiotic, or PPI within 4 weeks of EGD. Those who have been taking these medications
should have a biopsy. Culture and PCR are not routinely available for clinical use.
(b) Nonendoscopic: UBT and FAT are reliable methods to identify active infection. UBT is
the most reliable test to document the eradication of infection, but FAT can also be used.
Although antibody testing has low cost and rapid results, its usefulness is limited because of
its lower sensitivity and specificity and, more importantly, its inability to test for eradication.

D. Treatment (Domain 1, Task 3,4,6,7)


1. Treatment goals include H. pylori eradication, relief of symptoms, ulcer healing, and prevention of
ulcer recurrence.
2. Strategies
a. Nonpharmacologic approaches may include smoking cessation, avoiding NSAID use (including
aspirin), avoiding foods that exacerbate symptoms, and eliminating or reducing physiologic stress.
Data validating these approaches are lacking, but these approaches are still typically included in
treatment plan.

ACCP Updates in Therapeutics® 2016: Ambulatory Care Pharmacy Preparatory Review and Recertification Course

1-539
Gastrointestinal Disorders

b. Pharmacologic: H. pylori–induced ulcers (Am J Gastroenterol 2007;102:1805-25)


i. Primary treatment (see Table 3)
(a) Clarithromycin-based triple therapy
(1) PPI plus clarithromycin plus amoxicillin
(2) First-line therapy for those who have not previously received a regimen
(3) Metronidazole can be substituted for amoxicillin in those with a penicillin allergy.
Similar efficacy between regimens including either amoxicillin or metronidazole
(4) Recommended therapy duration is 10–14 days. Many studies have used the same
regimen with shorter therapy; efficacy was 80% or less.
(5) Similar efficacy regardless of which PPI is included in regimen
(b) Bismuth quadruple therapy
(1) PPI plus bismuth subsalicylate plus metronidazole plus tetracycline
(2) If patient is adherent to therapy, eradication rates are usually higher than with
clarithromycin triple therapy.
(3) Favored in patients allergic to penicillin and those who have previously received
macrolide antibiotics
(4) More complex regimen; high pill count and more frequent daily dosing
(5) Eradication rates similar to triple-based therapy. Typically, used as second-line
therapy when first-line therapy fails or when patient is intolerant or allergic
(6) Alternative formulation – PPI twice daily plus Pylera (contains bismuth subcitrate,
metronidazole, and tetracycline). A high pill burden because dose is 3 capsules of
Pylera four times daily, together with a PPI twice daily
(c) Although other strategies have been sought to improve efficacy and reduce pill burden,
only clarithromycin triple therapy and bismuth quadruple therapy are recommended
regimens according to the guidelines.
ii. Sequential therapy
(a) PPI plus amoxicillin for 5 days, followed by PPI plus clarithromycin plus tinidazole for
an additional 5 days (10 days of therapy)
(b) More data necessary before using in clinical practice
iii. Salvage therapies for persistent infections
(a) Avoid regimens with antibiotics that were taken previously.
(b) Bismuth quadruple regimen for 7–14 days is an accepted regimen.
(c) Levofloxacin-based triple therapy for 10 days is another option.

ACCP Updates in Therapeutics® 2016: Ambulatory Care Pharmacy Preparatory Review and Recertification Course

1-540
Gastrointestinal Disorders

Table 3. Treatment Regimens for H. pyloria


Triple-Therapy Options (PPI + clarithromycin + amoxicillin [or] metronidazole)
Clarithromycin Amoxicillin 1000 Metronidazole Length, Efficacy,
PPI
500 mg PO BID mg PO BID 500 mg PO BID days %
Esomeprazole 40 mg daily × × —
Esomeprazole 40 mg daily × — ×
Lansoprazole 30 mg BID × × —
Lansoprazole 30 mg BID × — ×
Omeprazole 20 mg BID × × —
10–14 70–85
Omeprazole 20 mg BID × — ×
Pantoprazole 20 mg BID × × —
Pantoprazole 20 mg BID × — ×
Rabeprazole 20 mg BID × × —
Rabeprazole 20 mg BID × — ×
Quadruple-Therapy Options (PPI [or] H2RA + bismuth subsalicylate + metronidazole + tetracycline)
Bismuth
Metronidazole Tetracycline 500 Length, Efficacy,
PPI or H2RA 1
Subsalicylate 525
250 mg PO QID mg PO QID days %
mg PO QID
Esomeprazole 40 mg daily × × ×
Lansoprazole 30 mg BID × × ×
Omeprazole 20 mg BID × × ×
10–14 75–90
Pantoprazole 20 mg BID × × ×
Rabeprazole 20 mg BID × × ×
Ranitidine 150 mg BID × × ×

1
PPI therapy is preferred to H2RA therapy.
Note: Pylera is a combination product that also provides quadruple therapy in alternative formulations as discussed in the text.

Sequential Therapy
(PPI + amoxicillin for 5 days, followed by PPI + clarithromycin + tinidazole for an additional 5 days)
Amoxicillin 1000 Clarithromycin Tinidazole 500
Length, Efficacy,
PPI (for 10 days) mg PO BID for 500 mg PO BID mg PO BID for
days %
days 1–5 for days 6–10 days 6–10
Esomeprazole 40 mg daily × × ×
Lansoprazole 30 mg BID × × ×
Omeprazole 20 mg BID × × × 10 > 90
Pantoprazole 20 mg BID × × ×
Rabeprazole 20 mg BID × × ×
Dexlansoprazole and pantoprazole are not FDA approved; both are taken by the oral route.
a

BID = twice daily; H2RA = histamine-2 receptor blocker; PO = orally; QID = four times daily; (—) = drug not included as part of the regimen.
Adapted from: Am J Gastroenterol 2007;102:1805-25.

ACCP Updates in Therapeutics® 2016: Ambulatory Care Pharmacy Preparatory Review and Recertification Course

1-541
Gastrointestinal Disorders

iv. Adverse effects are reported to occur with similar frequency in about 5%–20% of patients
taking either a triple or a quadruple drug-based regimen.
(a) PPIs: Headache, diarrhea
(b) Clarithromycin: Diarrhea, GI upset, metallic taste
(c) Amoxicillin: GI upset, headache, diarrhea
(d) Metronidazole: Altered taste, dyspepsia, disulfiram-like reaction with alcohol consumption
(e) Tetracycline: GI upset, photosensitivity
(f) Bismuth: Darkening of tongue and stool, nausea, GI upset
v. Treatment outcomes
(a) Ideally, eradication testing should be performed in everyone; however, universal testing is
not practical or cost-effective.
(1) According to the current guidelines, the following patients are recommended to
undergo eradication testing post-antibiotic treatment:
(A) Those with H. pylori–induced ulcer
(B) Those with persistent dyspeptic symptoms despite the treat-and-test strategy
(C) Those with mucosa-associated lymphoid tissue lymphoma associated with H. pylori
(D) Those who have undergone early gastric cancer resection
(2) UBT and monoclonal FAT are preferred tests for eradication; test no sooner than 4
weeks after therapy completion.
(b) Lack of adherence and antibiotic resistance to regimen are the most important predictors
of treatment failure.
(1) Adherence
(A) Pharmacists have the opportunity to educate, monitor, and improve adherence.
(B) Strategies to overcome adherence issues include educating patients about the
importance of taking the regimen as prescribed and completing the course of
therapy. Patients should be educated about the most likely adverse effects of the
medication so that they will know what to expect while receiving therapy.
(2) Antibiotic resistance
(A) H. pylori resistance patterns are regional and continuously changing;
clarithromycin resistance appears to be increasing.
(B) Bacterial and host factors may also influence the development of resistance.
c. Pharmacologic: NSAID-induced ulcers (Am J Gastroenterol 2009;104:728-38)
i. Primary prevention
(a) Risk factor modification, when possible
(1) Risk factors associated with NSAID-related GI complications include previous GI
event (especially if complicated), age (older than 65 years), concomitant medications
(e.g., anticoagulants, corticosteroids, other NSAIDs [including low-dose aspirin],
high-dose NSAIDs), and chronic debilitating disorders, especially CV disease.
(2) H. pylori infection increases risk. Test for H. pylori infection; if infection is present,
eradication therapy should be initiated.
(b) Preventive therapy
(1) Common strategies used to prevent peptic ulceration and mucosal injury
(A) Concomitant therapy with PPI, high-dose (double dose) H2RA, or misoprostol
(B) Replace NSAID with cyclooxygenase-2 (COX-2) inhibitor.

ACCP Updates in Therapeutics® 2016: Ambulatory Care Pharmacy Preparatory Review and Recertification Course

1-542
Gastrointestinal Disorders

(2) Preventive therapy is recommended in certain patient populations according to a


combined assessment of GI and CV risk.
(A) Determine GI risk related to NSAID therapy. Risk is stratified into three risk
categories (low, moderate, and high).
• Low risk: No risk factors present
• Moderate risk: Presence of one or two of the following risk factors:
Age older than 65 years
High-dose NSAID therapy
History of uncomplicated ulcer
Concurrent use of aspirin (including low dose), corticosteroids,
or anticoagulants
• High risk: History of complicated ulcer or more than two risk factors
(B) Determine CV risk.
• High risk: Is arbitrarily defined as a requirement for low-dose aspirin for the
prevention of cardiac events
• Naproxen does not appear to increase CV risk; therefore, it is the preferred
NSAID for patients with CV risk.
(C) Table 4 summarizes the recommendations according to GI and CV risk.

Table 4. Therapy Recommendations for NSAID Ulcer Prevention According to GI and CV Risk
CV Risk
GI Risk
Low High
Low NSAID alone Naproxen + PPI (or) misoprostol
Moderate NSAID + PPI (or) misoprostol Naproxen + PPI (or) misoprostol
Try to replace NSAID therapy (if possible)
High Avoid NSAID (or) COX-2 inhibitors
(or) COX-2 inhibitor + PPI/misoprostol
COX-2 = cyclooxygenase-2; CV = cardiovascular; GI = gastrointestinal; NSAID = nonsteroidal anti-inflammatory drug.

(3) Pharmacologic agents


(A) PPIs
• Significantly reduce the development of gastric and duodenal ulcers and
related complications in patients taking NSAIDs (or COX-2 inhibitors)
• Standard dosing for unspecified length of therapy
• Similar efficacy for all formulations
(B) COX-2 inhibitors (celecoxib)
• Reduced incidence of gastric and duodenal ulcers compared with traditional
NSAIDs
• Beneficial effect is cancelled in patients taking low-dose aspirin.
• Therapeutic impact reduced by the association with CV events
(e.g., myocardial infarction and thrombotic events); thus, to minimize risk,
recommend lowest dose (200 mg/day)
(C) Misoprostol (Cytotec)
• Full dose (200 mcg four times daily) is efficacious; however, clinical use is
limited by GI effects (diarrhea and cramping) and high-frequency dosing,
resulting in poor adherence.
• Lower dosing may limit the incidence of adverse events and provide a
protective effect.

ACCP Updates in Therapeutics® 2016: Ambulatory Care Pharmacy Preparatory Review and Recertification Course

1-543
Gastrointestinal Disorders

(D) High-dose H2RAs


• Superior to placebo in preventing duodenal ulcers, but data in support of
NSAID-induced gastric ulcers are lacking
• Duexis, an H2RA and NSAID combination product (famotidine and
ibuprofen), is available. Formulated for use in those requiring NSAID therapy
also requiring GI risk reduction
(E) Summary: PPIs are the preferred agent for prevention; however, must evaluate
GI and CV risk
(4) Concurrent use of NSAIDs and antiplatelet therapy (i.e., aspirin and clopidogrel) (J
Am Coll Cardiol 2010;56:2051-66)
(A) Although antiplatelet therapy decreases ischemic risks, it may increase bleeding (GI)
complications; when used concurrently with NSAIDs, the bleeding risk is increased.
(B) Gastroprotective strategies (e.g., PPI) for patients with high GI risk are warranted.
(C) Communication among multidisciplinary team members (i.e., cardiology,
gastroenterology, and primary care) is necessary to determine the appropriate
course for individuals with high GI risk requiring antiplatelet therapy.
ii. Treatment (secondary prevention)
(a) Risk factor modification, when possible
(b) Reduce dose or eliminate NSAID therapy, when possible.
(c) Test for H. pylori infection; if infection is present, treat.
(d) First-line therapy: PPIs indicated for ulcer healing and secondary prevention; use is
evaluated by GI and CV risk assessment. Similar benefits for all PPI formulations.
NSAID/PPI combination product – Vimovo is available; contains naproxen and
esomeprazole in two different formulations: 375 mg/20 mg and 500 mg/20 mg
(e) Other options
(1) Misoprostol appears to be as effective as PPIs for secondary prevention, but use is
limited because of adverse events.
(2) H2RAs inferior to other agents
(3) COX-2 limited because of potential CV events
(4) Antiplatelet therapy (i.e., aspirin and clopidogrel) and an NSAID used concurrently
will require evaluation.

ACCP Updates in Therapeutics® 2016: Ambulatory Care Pharmacy Preparatory Review and Recertification Course

1-544
Gastrointestinal Disorders

Patient Case

Questions 2 and 3 pertain to the following case.


A 45-year-old woman presents with sharp epigastric pain that is worse after meals. She says that she feels bloated
all the time and often belches. She finds some relief with antacids, but the pain returns about 3 hours after each
dose. She states she has only begun having these symptoms every other day for the past 2 months. A UBT is
positive for H. pylori. Current medications include citalopram 20 mg/day, loratadine 10 mg/day, and a daily
multivitamin. She has a penicillin allergy.

2. Which treatment for H. pylori is best?


A. Amoxicillin 1 g twice daily plus clarithromycin 500 mg twice daily plus esomeprazole 40 mg once daily
for 7 days.
B. Amoxicillin 1 g twice daily plus clarithromycin 500 mg twice daily plus esomeprazole 40 mg twice
daily for 14 days.
C. Bismuth subsalicylate 525 mg four times daily plus metronidazole 250 mg four times daily plus
tetracycline 500 mg four times daily plus esomeprazole 40 mg once daily for 7 days.
D. Bismuth subsalicylate 525 mg four times daily plus metronidazole 250 mg four times daily plus
tetracycline 500 mg four times daily plus esomeprazole 40 mg once daily for 14 days.

3. The patient presents for a follow-up 4 weeks after completing H. pylori treatment and asks her physician if
the H. pylori infection has been cleared. Which laboratory assessment is best to perform to confirm H. pylori
eradication on completing H. pylori therapy for this patient at this time?
A. Tissue culture.
B. UBT.
C. RUT.
D. Serum antibody test.

III.  COMPLICATIONS OF CHRONIC LIVER DISEASE

A. Cirrhosis occurs when fibrosis replaces destroyed hepatocytes; this can result in decreased liver synthetic
and metabolic function and resistance to blood flow (portal hypertension).
1. Cirrhosis is the 12th leading cause of death in the United States, with more than 36,000 deaths each
year as of 2013 (www.cdc.gov/nchs/data/nvsr/nvsr64/nvsr64_02.pdf).
2. Clinical presentation of patients with cirrhosis varies from asymptomatic to life threatening.
3. Cirrhosis is divided into two stages: Compensated and decompensated. In decompensated cirrhosis,
complications of cirrhosis occur, which is defined as the presence of gastroesophageal variceal
bleeding, ascites, HE, and/or jaundice.
4. Patients are at a high risk of dying from the complications of decompensated cirrhosis; predicted
survival is less than 5 years without a liver transplant.
5. Table 5 lists the clinical presentation of cirrhosis.

ACCP Updates in Therapeutics® 2016: Ambulatory Care Pharmacy Preparatory Review and Recertification Course

1-545
Gastrointestinal Disorders

Table 5. Cirrhosis: Clinical Presentation


Signs and symptoms Anorexia
Ascites
Edema
Encephalopathy
Gastroesophageal varices
Gynecomastia
Hepatomegaly
Hyperpigmentation
Jaundice
Malaise
Palmar erythema
Pleural effusion
Portal hypertensive gastropathy
Pruritus
Respiratory difficulties
Spider angioma
Splenomegaly
Weight loss
Laboratory ↑ PT/INR
abnormalities Hypoalbuminemia
Thrombocytopenia
INR = international normalized ratio; PT = prothrombin time.

B. Etiology of Cirrhosis
1. Infectious (e.g., viral hepatitis)
2. Metabolic (e.g., hemochromatosis, Wilson disease, α1-antitrypsin deficiency)
3. Immune mediated (e.g., autoimmune hepatitis, primary biliary cirrhosis, primary sclerosing cholangitis)
4. Biliary obstruction (e.g., atresia, strictures, gallstones)
5. Vascular (e.g., Budd-Chiari syndrome, veno-occlusive disease)
6. Toxins (e.g., alcohol)
7. Other (e.g., nonalcoholic fatty liver disease (NAFLD), sarcoidosis)
8. Cryptogenic (much of which may be unrecognized NAFLD)

C. Disease Severity (Domain 5, Task 2)


1. Assessment of disease severity by scoring systems is performed to predict survival and risk of
complications.
2. Child-Pugh classification of cirrhosis
a. Grades the degree of liver disease on the basis of laboratory values and clinical manifestations.
Points are assigned and then summed according to Table 6 to determine the patient’s score. Score
ranges from 5 to 15 points and is classified as A, B, or C as follows:
i. Grade A: 5–6 points (mild disease)
ii. Grade B: 7–9 points (moderate disease)
iii. Grade C: 10–15 points (severe disease)

ACCP Updates in Therapeutics® 2016: Ambulatory Care Pharmacy Preparatory Review and Recertification Course

1-546
Gastrointestinal Disorders

Table 6. Child-Pugh Classification


Score
1 Point 2 Points 3 Points
Encephalopathy Absent Medically controlled Poorly controlled
Ascites Absent Diuretic responsive Diuretic refractory
Bilirubin, mg/dL <2 2–3 >3
Albumin, mg/dL > 3.5 2.8–3.5 < 2.8
Prothrombin time, seconds
1–4 4–6 >6
prolonged
(< 1.7) (1.7–2.3) (> 2.3)
(or the INR)

b. The Child-Turcotte-Pugh (CTP) classification is no longer used to determine disease severity for
organ transplantation because of the inclusion of subjective variables within the score calculation
(e.g., encephalopathy and ascites). It is used in medication labeling to describe dosing for individuals
with varying degrees of liver dysfunction and in guidelines for HCV medication selection.
3. Model for end-stage liver disease (MELD) (Hepatology 2007;45:797-805)
a. This scoring system was created to predict survival after the elective transjugular intrahepatic
portosystemic shunt (TIPS) procedure.
b. MELD formula incorporates three objective laboratory values: international normalized ratio
(INR), SCr, and serum bilirubin: (9.57 x ln[creatinine (mg/dL)] + 3.78 x ln[total bilirubin (mg/dL)]
+ 11.2 x ln[INR] + 6.43).
c. In February 2002, the United Network for Organ Sharing (UNOS) adopted the MELD scoring
system, replacing the CTP-based organ allocation system to accurately rank patients with cirrhosis
awaiting liver transplantation according to their mortality risk. The UNOS ranking system
includes the following modifications to the MELD score calculation:
i. Limits on laboratory values
(a) Laboratory values for SCr, bilirubin, and INR that are less than 1.0 are set to 1.0 for the
MELD calculation. The maximum SCr used in the calculation is 4.0 mg/dL.
(b) For patients who have undergone two or more dialysis sessions or 24 hours of continuous
venovenous hemodialysis within the previous week, SCr is set at 4.0 mg/dL.
ii. Maximal MELD score is 40 (range 0–40). MELD score calculator is available at http://optn.
transplant.hrsa.gov/converge/resources/MeldPeldCalculator.asp?index=98.
iii. Patients who meet the criteria for exceptions are awarded additional points to increase their
MELD scores (e.g., hepatocellular carcinoma, hepatopulmonary syndrome).

D. General Approach to Managing Cirrhosis


1. Identify and eliminate cause when possible (e.g., alcohol, HCV) in an effort to prevent disease
progression and complications. Appearance of complications marks the onset of decompensation.
2. Assess, diagnose, and treat complications of decompensated cirrhosis appropriately.
3. Liver transplantation is the only curative treatment modality for decompensated cirrhosis.

E. Complications of Cirrhosis (Domain 2, Task 5)


1. Ascites
a. Most common complication; results in an abnormal sequestration of fluid in the peritoneal cavity.
About 50% of patients with compensated cirrhosis will develop ascites within 10 years. About
50% of patients who develop ascites will die within 5 years.

ACCP Updates in Therapeutics® 2016: Ambulatory Care Pharmacy Preparatory Review and Recertification Course

1-547
Gastrointestinal Disorders

b. Ascites is associated with decreased quality of life, increased risk of developing spontaneous
bacterial peritonitis (SBP), and renal failure.
c. Pathophysiology – Portal hypertension occurs when hepatic parenchyma is replaced with fibrotic
tissue, causing a resistance to liver blood flow. Portal hypertension results in release of systemic
and splanchnic vasodilators. Subsequent neurohormonal response to this vasodilation leads to
sodium and water retention and renal vasoconstriction. Portal hypertension and low intravascular
oncotic pressure lead to third spacing of the resulting excess fluid to the peritoneal space.
d. Clinical presentation
i. Symptoms include progressive abdominal heaviness, fullness, pressure, and pain;
shortness of breath; early satiety
ii. Tense ascites: Clinical symptoms usually increased and more significant,
requiring paracentesis
iii. Refractory ascites: Fluid overload unresponsive to dietary sodium restriction and diuretic therapy
e. Diagnosis (Domain 1, Task 1; Domain 4, Task 3)
i. Initial history should focus on risk factors for liver disease and cirrhosis in addition to non-
hepatic causes (e.g., heart failure, metastatic cancer, nephrotic syndrome, pancreatitis).
ii. Physical examination: Abdominal distension, shifting dullness, bulging flanks, and fluid wave
iii. Abdominal ultrasonography
(a) Detects ascites, especially in patients with obesity, when a physical examination may
be inconclusive
iv. Diagnostic abdominal paracentesis with ascitic fluid analysis
(a) Ascitic fluid may accumulate as transudates (extravascular fluid with low protein content)
or exudates (extravascular fluid with high protein content). Transudates result from
increased portal pressure, and exudates result from other non–cirrhotic-related causes
(e.g., pancreatitis).
(b) Ascitic fluid accumulation can be differentiated as a transudate or an exudate by
calculating the serum-ascites albumin gradient (SAAG). SAAG formula = (ascites
albumin concentration − serum albumin concentration). Note: Serum and ascitic albumin
measurements must be obtained the same day.
(1) A SAAG of 1.1 g/dL or greater suggests portal hypertension as the cause
(with 97% accuracy).
(2) A SAAG less than 1.1 g/dL suggests non-portal hypertension causes of ascites.
f. Treatment (Domain 1, Task 1-7)
i. Goals
(a) Improve quality of life.
(b) Prevent complications and treatment-related adverse effects.
ii. If possible, treat underlying cause of liver disease (e.g., ceasing alcohol consumption for
alcoholic-induced disease).
iii. First-line therapy is dietary sodium restriction and diuretics.
(a) Dietary sodium restriction (2000 mg/day) and diet education
(b) Fluid restriction necessary in patients with symptomatic severe hyponatremia when
serum sodium concentration is less than 120 mmol/L
(c) Oral diuretic therapy
(1) Minimal fluid overload; may be treated with single diuretic therapy (spironolactone);
however, a combination regimen including furosemide is usually preferred to avoid
hyperkalemia and to provide several mechanisms of action

ACCP Updates in Therapeutics® 2016: Ambulatory Care Pharmacy Preparatory Review and Recertification Course

1-548
Gastrointestinal Disorders

(2) When used in combination, a ratio of 40 mg of furosemide to every 100 mg of


spironolactone is typically used, and doses are increased simultaneously every 3–5
days. Usual maximal doses are furosemide 160 mg/day and spironolactone 400 mg/
day. Maximal weight-loss goal is 0.5 kg/day in patients without edema; no maximum
in patients with significant edema
(3) Gynecomastia may occur with spironolactone. Amiloride is an alternative at a dose
of 10–40 mg/day. Amiloride is less effective than spironolactone, but it will help
maintain normokalemia.
(4) Other diuretics, including hydrochlorothiazide and metolazone, are generally avoided
because of concerns for hyponatremia and too-rapid diuresis. Eplerenone has not
been well studied for cirrhosis and ascites.
(5) Patients whose diuretic therapy fails are said to have refractory ascites and will likely
require serial therapeutic paracentesis. Diuretic failure may present as minimal to no
weight loss and inadequate sodium excretion (less than 78 mmol/day) despite diuretics, or
as the development of clinically significant complications associated with diuretic therapy
such as encephalopathy, elevated SCr (greater than 2.0 mg/dL), elevated serum potassium
concentration (greater than 6.0 mmol/L), or decreased serum sodium concentration (less
than 120 mmol/L).
(6) Methods to assess sodium excretion
(A) 24-hour urine sodium measurement. Target urinary sodium excretion is greater
than 78 mmol/day. Much more difficult to collect than random urine samples
(B) Random urine sodium/potassium ratio. Target sodium concentration is greater
than the potassium concentration (ratio greater than 1).
iv. Discontinue NSAIDs or drugs that may interfere with sodium and water retention.
v. Therapeutic abdominal paracentesis
(a) Large-volume paracentesis (several liters or more) is performed to relieve pain and
pressure from tense ascites.
(b) Colloid replacement after large-volume paracentesis remains controversial; however,
albumin replacement appears acceptable if more than 5 L is removed. Typically, albumin
is administered at a dose of 6–8 g/L of ascitic fluid removed.
(c) Ideally, diuretic therapy will be optimized to avoid necessity of repeat paracentesis; less
than 10% of patients have refractory ascites and require serial paracentesis.
vi. Other treatment options for patients with refractory ascites (defined as fluid overload
unresponsive to sodium diet restrictions and medical management with diuretics, or recurring
quickly after therapeutic paracentesis):
(a) β-Blocker therapy: Assess risk-benefit; use may shorten survival in this patient population.
(b) Midodrine (7.5 mg by mouth three times daily): Consider adding to diuretic therapy to
increase blood pressure by increasing urine volume, urine sodium, and mean arterial pressure.
A randomized trial reports increased survival when used in this patient population.
(c) Serial therapeutic paracentesis: Clinical trial data show this approach to be safe and
effective. Procedure is typically performed once diuretics have been discontinued,
about every 2 weeks.
(d) Liver transplants should be considered once ascites develops; 21% of people with ascites
die within 6 months of the condition becoming refractory to routine medical therapy.
(e) TIPS procedure, which is a side-to-side portacaval shunt. Four large-scale, multicenter,
randomized controlled trials comparing TIPS with large-volume paracentesis were
performed; all reported better control of refractory ascites with TIPS.

ACCP Updates in Therapeutics® 2016: Ambulatory Care Pharmacy Preparatory Review and Recertification Course

1-549
Gastrointestinal Disorders

(f) Peritoneovenous shunt: Because of complications and no survival advantage, this


procedure is not routinely performed. Considered in patients who are not candidates for
serial paracentesis, TIPS, or liver transplantation
vii. Hospitalization may be required to treat tense ascites, especially if infection or renal
dysfunction is suspected.
g. Role of the pharmacist (Domain 1, Task 5; Domain 2, Task 1,5; Domain 5, Task 2)
i. Patients with ascites are often nonadherent to sodium restriction and diuretic therapy because
of adverse effects. In addition, drug therapies eventually become less effective over time.
ii. Pharmacists should continually assess therapies to treat/prevent ascites and target their
recommendations to the patient’s adherence, adverse effects, and lack of effectiveness of regimens.
The pharmacist may consider switching agents to improve patient adherence.
iii. Continual monitoring for electrolyte imbalances and renal function and communication with
the treatment team

Patient Case

4. A 60-year-old woman has Child-Pugh class B cirrhosis secondary to autoimmune hepatitis. Her medical
history includes hypothyroidism and chronic back pain. She has new-onset abdominal pain and shortness of
breath. She is afebrile with abdominal tenderness, including flank bulging and shifting dullness as well as a
small pleural effusion. All laboratory values regarding electrolytes and renal function are within the normal
limits. She takes levothyroxine 75 mcg once daily and oxycodone 10 mg every 8 hours as needed for pain.
She reports no drug allergies. Which is the best recommendation to treat her new-onset ascites?
A. Spironolactone 100 mg by mouth daily plus furosemide 40 mg by mouth daily.
B. Spironolactone 100 mg by mouth daily.
C. Spironolactone 40 mg by mouth daily plus furosemide 100 mg by mouth daily.
D. Furosemide 40 mg by mouth daily.

2. SBP
a. Definition
i. Bacterial infection of preexisting ascitic fluid without evidence of an intra-abdominal source
ii. Occurs in 15%–26% of hospitalized patients with liver disease complicated by ascites
b. Pathophysiology (Domain 1, Task 2)
i. Source of ascitic bacterial inoculation is unclear, but because enteric organisms are usually
isolated, the GI tract is often suggested as the source of bacterial contamination. The likely
mechanism is translocation of gut pathogens to the bloodstream.
ii. Pathogens include Escherichia coli, Klebsiella pneumoniae, pneumococci, and Enterococcus
spp., with E. coli being the most common species identified.
c. Clinical presentation: Highly variable and may be asymptomatic in the outpatient population.
Patients with SBP may have one or more of the following:
i. Local symptoms and/or signs of peritonitis (e.g., abdominal tenderness or pain, vomiting, diarrhea)
ii. Signs and symptoms of systemic disease (e.g., hyperthermia or hypothermia, chills, abnormal
white blood cell count, tachycardia and/or tachypnea)
iii. Worsening encephalopathy findings
iv. Increase in INR, total bilirubin
v. Renal failure
vi. Shock

ACCP Updates in Therapeutics® 2016: Ambulatory Care Pharmacy Preparatory Review and Recertification Course

1-550
Gastrointestinal Disorders

d. Diagnosis (Domain 1, Task 1; Domain 4, Task 3)


i. Abdominal paracentesis with ascitic fluid analysis and culture must be performed. Ideally, this
should be done before the first dose of antibiotics.
(a) Peritoneal infection causes an inflammatory reaction, resulting in an increased number of
polymorphonuclear cells. The presence of more than 250 polymorphonuclear cells/mm3
in the ascites fluid is diagnostic for SBP.
(b) Ascitic fluid cultures are negative in most cases; they are not necessary for the diagnosis
of SBP but, when positive, are important to guide antibiotic therapy.
ii. Blood cultures should be performed for all patients thought to have SBP before antibiotic
therapy is initiated.
e. Treatment (Domain 1, Task 3,4,6,7)
i. Must be initiated promptly when laboratory results are positive for SBP
ii. Hospital management
(a) Empiric antibiotic therapy targeting likely enteric gram-negative organisms should be
initiated for all patients with ascitic fluid polymorphonuclear counts of 250 cells/mm3 or
greater until susceptibility testing is available. Patients with polymorphonuclear counts
less than 250 cells/mm3 and signs or symptoms of infection should also receive empiric
therapy while culture results are pending.
(1) Third-generation cephalosporins are preferred.
(A) Cefotaxime (2 g every 8 hours) or
(B) Ceftriaxone (2 g/day)
(2) Fluoroquinolones may be used in patients with severe penicillin allergies.
(3) Data show that therapy for 5 days is as effective as therapy for 10 days; therefore,
recommendations are to treat for 5 days.
(4) In one study, oral ofloxacin (400 mg twice daily for 8 days) was an acceptable
substitute in those without prior quinolone exposure, vomiting, shock, grade 2 or higher
encephalopathy, or SCr greater than 3 mg/dL, but the therapy was given in hospital.
(5) Therapy should be narrowed according to culture and sensitivity results,
when possible.
(6) Therapy selection and dosing may vary depending on patient-specific information (i.e.,
medication allergies, renal function, antibiotic exposure history, and active GI bleed).
(b) Albumin administration for patients with SBP
(1) Patients with ascites and SBP are intravascularly depleted with decreased organ
perfusion, resulting in renal hypoperfusion and the potential of hepatorenal
syndrome (HRS).
(2) Albumin administration is believed to help restore intravascular volume and possibly
modulate the action of cytokines, and it increases survival in patients with SBP.
(3) Current North American guidelines recommend administering albumin (1.5 g/kg
on diagnosis of SBP and then 1 g/kg on day 3), together with antibiotics, in patients
meeting the following criteria: SCr greater than 1 mg/dL, blood urea nitrogen (BUN)
greater than 30 mg/dL, or total bilirubin greater than 4 mg/dL. In contrast, European
guidelines recommend this albumin dose for all patients with a diagnosis of SBP
until further studies are done to elucidate the subpopulations that benefit.
(c) Repeated paracentesis is not necessary to document the eradication of infection in routine
cases. When antibiotic resistance is suspected, atypical organisms are identified on
culture, or expected clinical response does not occur, repeat paracentesis should be done
after 48 hours of treatment.

ACCP Updates in Therapeutics® 2016: Ambulatory Care Pharmacy Preparatory Review and Recertification Course

1-551
Gastrointestinal Disorders

f. Prevention
i. Up to 70% of patients have a recurrent episode after the first episode of SBP; thus, antibiotic
prophylaxis is recommended.
ii. Risk factors for recurrence
(a) Ascitic fluid protein concentration less than 1.5 g/dL
(b) Previous episode of SBP
(c) PPI use: Therapy with these agents has been associated with SBP; however, clinical
significance remains to be determined. Restricting PPI use to approved indications may
help reduce the incidence of SBP.
iii. Antibiotic prophylaxis
(a) Initiate primary prophylaxis in the following patient population: Cirrhosis and ascites
with low ascitic protein concentration (less than 1.5 g/dL) and at least one of the
following criteria:
(1) SCr of 1.2 mg/dL or greater
(2) BUN of 25 mg/dL or greater
(3) Serum sodium concentration of 130 mmol/L or less
(4) Child-Pugh score of 9 points or greater with bilirubin concentration of 3 mg/dL or greater
(b) Initiate secondary prophylaxis in any patient with a history of SBP.
(c) Antibiotic therapy options for primary and secondary prophylaxis:
(1) Norfloxacin 400 mg orally daily (most-studied option, but U.S. manufacturer has
stopped production with no apparent plan to resume)
(2) Ciprofloxacin 500 mg orally daily
(3) Trimethoprim/sulfamethoxazole (1 double-strength tablet daily)
(4) Ciprofloxacin 750 mg once weekly has been studied, but guidelines recommend
against intermittent dosing because of concerns regarding increased resistance;
further studies are warranted.
(5) Primary and secondary prophylaxis should be long term (until liver transplantation
or complete resolution of ascites).
iv. Antibiotic prophylaxis for upper GI bleeding
(a) Administering antibiotics to patients with cirrhosis with upper GI bleeding reduces
mortality. It was initially thought that this was simply because of a reduction in SBP, but
the survival benefit pertains to patients with cirrhosis without ascites as well.
(b) For an acute GI bleed in a patient with cirrhosis: Administer a 7-day course of
ceftriaxone or norfloxacin (400 mg twice daily) while the patient is hospitalized. A
randomized trial showed that a 7-day course of ceftriaxone was superior to norfloxacin.
g. Role of the pharmacist (Domain 1, Task 5; Domain 2, Task 1,5; Domain 5, Task 2)
i. Treatment recommendations should consider the patient’s medication allergies,
previous antibiotic use, and resistance patterns—information that pharmacists should
have readily available.
ii. Pharmacists in the ambulatory setting are positioned to perform medication reconciliation,
which will ensure that patients requiring long-term SBP prophylaxis continue to receive
the appropriate antibiotic therapy. Continual education may be necessary so that the patient
understands the medication’s purpose and continues taking it.
3. HRS
Note: Type 1 HRS is a late complication of cirrhosis, usually requiring hospitalization. Although type
1 HRS is not usually encountered in the ambulatory setting, a brief overview is provided in this chapter
for completeness.
a. HRS occurs when renal function is impaired secondary to cirrhosis, with a decline in renal perfusion,
glomerular filtration rate (GFR), and the kidneys’ ability to excrete sodium and free water.

ACCP Updates in Therapeutics® 2016: Ambulatory Care Pharmacy Preparatory Review and Recertification Course

1-552
Gastrointestinal Disorders

b. Two types of HRS have been characterized.


i. Type 1: Rapidly progressive decline in renal function; doubling of the initial SCr to a
concentration greater than 2.5 mg/dL or a 50% reduction in the initial 24-hour creatinine
clearance (CrCl) to a concentration less than 20 mL/minute/1.73 m2 in less than
2 weeks: High mortality
ii. Type 2: Slowly progressing. Generally occurs in those with refractory ascites and requires
no further management than diuretic avoidance, serial paracentesis, and consideration of
kidney transplantation
c. Diagnosis of HRS in the setting of cirrhosis includes the following:
i. Cirrhosis with ascites
ii. SCr greater than 1.5 mg/dL
iii. No improvement in SCr after at least 2 days with diuretic withdrawal and volume expansion
with albumin (recommended albumin dose is 1 g/kg/day, no more than 100 g/day)
iv. Absence of shock
v. No current or recent treatment with nephrotoxic medications
vi. Absence of parenchymal kidney disease, defined by proteinuria greater than 500 mg/day and/
or abnormal renal ultrasonography
d. Treatment options for type 1 HRS (i.e., when the renal dysfunction is abrupt and severe):
i. Liver transplantation is the only definitive treatment.
ii. Pharmacologic
(a) Aim of pharmacologic therapy is to reverse splanchnic vasodilation and improve kidney
perfusion/function and survival as a bridge while waiting for transplantation.
(b) Options: North America: Octreotide plus midodrine plus albumin (or norepinephrine
plus albumin in the ICU). Other countries use terlipressin plus albumin preferentially, but
terlipressin is not available in the United States.
iii. Hemodialysis: Controls azotemia and maintains electrolyte balance as a last resort
4. Hepatic encephalopathy (HE) (Hepatology 2014;60:715-35).
a. Defined as a CNS dysfunction caused by liver insufficiency and/or portosystemic shunting
that manifests as a wide spectrum of neurological or psychiatric abnormalities ranging from
subclinical alterations to coma
b. Pathophysiology (Domain 1, Task 2)
i. Systemic accumulation of toxins from the gut because of reduced liver metabolism and
shunting through portosystemic collaterals, bypassing the liver. Accrued substances then
enter the CNS, altering neurotransmission. Exact mechanism is unknown, but it is believed to
be multimodal, likely involving abnormalities in glutamatergic, serotoninergic, GABA, and
catecholamine pathways.
ii. Nitrogenous substances, including ammonia, are important; however, ammonia concentrations
are poorly correlated with the degree of encephalopathy.
c. Clinical presentation
i. Wide range of disturbances, from minimally impaired social and work functioning to deep coma
ii. Typical manifestations may include confusion, disorientation, asterixis (flapping tremor),
decreased energy level, sleep-wake cycle impairment, abnormal speech patterns, and
cognitive deficits.
d. Diagnosis (Domain 1, Task 1; Domain 4, Task 3)
i. No clinical test is available to assess entire spectrum of disease. HE is a diagnosis of
exclusion. Typically, clinicians can diagnose cognitive dysfunction; however, the difficulty is
linking the cognitive impairment to HE (i.e., liver impairment).
ii. Appropriate testing differs according to acuity of presentation and degree of impairment.

ACCP Updates in Therapeutics® 2016: Ambulatory Care Pharmacy Preparatory Review and Recertification Course

1-553
Gastrointestinal Disorders

iii. Classification: According to the current guidelines, each episode of HE should be classified on
the basis of the following four factors:
(a) Type of underlying disease
(1) Type A – Caused by acute liver failure
(2) Type B – Predominantly caused by portosystemic bypass or shunting
(3) Type C – Caused by cirrhosis
(b) Severity of manifestations or grade
(1) Categorize as covert or overt according to level of impairment and symptoms
(A) Covert: Some cognitive/behavioral deficiency despite orientation to time and space
(B) Overt: Disoriented to time and/or space and cognitive/behavioral deficiencies
(2) In an effort to standardize the classification scheme, the new guidelines combine the
West Haven criteria (WHC), which allow for a subjective assessment of mental status
and an assignment of severity (grades 0–4) based on observed symptoms, and the
ISHEN (International Society for Hepatic Encephalopathy and Nitrogen Metabolism)
categories of covert and overt as shown in Table 7.
(3) It is important to realize that the severity of the abnormality may fluctuate and that
individuals may rapidly cycle among stages. HE exacerbations are episodic.

Table 7. Classification of Hepatic Encephalopathy


WHC ISHEN Description Operative Criteria
Unimpaired • No encephalopathy Tested and no impairments identified
• Lack of clinical evidence of mental
changes Oriented to time and space
Minimal • Evidence for subtle changes on tests of Abnormal test results
neuropsychological or psychometric Lacking clinical manifestations
function
Covert
• Trivial lack of awareness
• Shortened attention span Oriented to time and space
Grade
• Impaired addition or subtraction Minimal cognitive/behavioral impairment
1
• Altered sleep pattern
• Euphoria or anxiety
• Lethargy or apathy
• Disorientation for time
Grade • Personality change that is obvious Disoriented to time
II • Inappropriate behavior ± cognitive/behavioral impairments
• Slurred speech
• Asterixis
Overt
• Responsive to stimuli
• Confused
Grade Disoriented to time and space
• Gross disorientation
III ± cognitive/behavioral impairments
• Bizarre behavior
• Somnolence to semistupor
Grade
• Coma Unresponsive, even to painful stimuli
IV
ISHEN = International Society for Hepatic Encephalopathy and Nitrogen Metabolism; WHC = West Haven criteria.

ACCP Updates in Therapeutics® 2016: Ambulatory Care Pharmacy Preparatory Review and Recertification Course

1-554
Gastrointestinal Disorders

(4) Glasgow Coma Scale


(A) Gold standard for those with significantly altered consciousness
(B) Scale includes three tests: Eyes, verbal, and motor responses. Individual scores as
well as the sum of scores are considered. Range 3 (deep coma) to 15 (fully awake)
(c) Time course
(1) Episodic
(2) Recurrent – Bouts of HE occur within a time course of 6 months or less.
(3) Persistent – Some behavioral alteration always present, interspersed with relapses
of overt HE (OHE)
(d) Existence of precipitating factors
(1) Non-precipitated – Spontaneous
(2) Precipitated – Examples of precipitants: Infections, GI bleed, diuretic overdose,
electrolyte disorder, constipation, unidentified. For HE type C, exacerbations
typically have a precipitant; precipitating factors should be identified and treated.
e. Treatment recommendations according to the 2014 American Association for the Study of Liver
Diseases (AASLD) practice guidelines (Domain 1, Task 3,4,6,7)
i. General recommendations
(a) OHE
(1) An episode (spontaneous or precipitated) should be actively treated.
(2) A four-pronged approach to treatment is recommended.
(A) Initiate care for individuals with altered consciousness.
(B) Alternative causes of altered mental status should be sought and treated.
(C) Identify precipitating causes and try to correct.
(D) Start empiric HE treatment.
(b) Secondary prophylaxis after an episode of OHE is recommended.
(c) Primary prophylaxis only required in patients with cirrhosis with a high risk
of developing HE
(d) Recurrent intractable OHE together with liver failure is an indication for
liver transplantation.
ii. Therapy recommendations for episodes of OHE
(a) Identify and treat precipitating factors of HE.
(b) Lactulose is the first choice for treatment of episodic OHE.
(c) Rifaximin is an effective add-on therapy to prevent OHE recurrence.
(d) Branched-chain amino acids oral therapy can be used as an alternative or additional agent
to treat those whose condition is unresponsive to conventional therapy.
(e) Intravenous l-ornithine l-aspartate can be used as an alternative or additional agent to
treat those whose condition is unresponsive to conventional therapy.
(f) Neomycin: Alternative option to treat OHE
(g) Metronidazole: Alternative option to treat OHE
iii. Therapy recommendations for prevention of OHE
(a) Lactulose for prevention of recurrent episodes after the initial episode of HE
(b) Rifaximin as add-on therapy for prevention after the second episode of HE
(c) Routine prophylactic therapy (lactulose or rifaximin) is not recommended for post-TIPS HE.
(d) Discontinuation of prophylactic therapy should be considered when precipitating factors
have been well controlled or liver function or nutritional status has improved.
iv. Therapy recommendations for minimal and covert HE
(a) Not routinely recommended
(b) Must assess on a case-by-case basis

ACCP Updates in Therapeutics® 2016: Ambulatory Care Pharmacy Preparatory Review and Recertification Course

1-555
Gastrointestinal Disorders

v. Nutritional management
(a) Daily energy intake should be 35–40 kcal/kg.
(b) Daily protein intake should be 1.2–1.5 g/kg/day.
(c) Offer small meals or nutritional supplements evenly distributed throughout the day.
f. Pharmacotherapy
i. Nonabsorbable disaccharides (lactulose)
(a) First choice for treatment of episodic OHE and first line for prevention of HE recurrence
(b) Degraded by colonic bacteria to acid, lowering the gut pH and converting NH3 in the GI
lumen to NH4+
(c) Dosing
(1) 45 mL every 1–2 hours until at least 2 soft or loose bowel movements per day are
produced; then dose titration (15–45 mL every 8–12 hours) to achieve 2 or 3 soft
stools a day. Powder formulation (KRISTALOSE) may be more palatable than
lactulose (10- or 20-g packets that may be dissolved in 4 oz of water; 10 g = 15 mL
of syrup formulation).
(2) For patients unable to receive oral therapy, administer as retention enema (300 mL of
lactulose syrup in 700 mL of water, held for 30–60 minutes). However, oral therapy
is more effective and can be given by nasogastric tube, if needed.
(d) Common adverse effects include flatulence, diarrhea, abdominal cramping, and unpleasant
taste. Serious adverse effects include dehydration and hypernatremia.
(e) Challenge of therapy is relying on patient/caregiver to self-adjust dose.
ii. Antibiotics: Adjunct to nonabsorbable disaccharides; function is to reduce bacterial
production of toxins. A synergistic effect may be achieved by combining with lactulose.
(a) Rifaximin
(1) Effective add-on therapy to lactulose for prevention of OHE recurrence after the first
recurrence while administering lactulose according to 2014 practice guidelines
(2) 550 mg orally twice daily is approved for overt encephalopathy. It is believed
to decrease intestinal production and absorption of toxins by altering GI flora.
Commonly reserved for patients whose condition does not respond to, or who are
intolerant of, lactulose. Although drug costs are higher, it is hoped this can be offset
by shorter or fewer hospital stays. No clinical trial data supporting monotherapy
(b) Metronidazole: As short-term therapy. Adverse event potential (neurotoxicity) generally
precludes use as first-line therapy.
(c) Neomycin (initial: 3–6 g/day orally in three or four divided doses for 1–2 weeks;
maintenance: 1–2 g/day orally). Adverse event potential generally precludes use as first- or
second-line therapy, especially given the concern for renal toxicity, ototoxicity. According
to the guidelines, metronidazole and neomycin are alternatives for treatment of OHE.
iii. Others
(a) Zinc deficiency is common in those with HE; zinc is an essential element that acts
as a cofactor in the urea cycle. Thus, zinc supplementation may reduce ammonia
concentration and may be reasonable in those with zinc deficiency.
(b) Drugs that affect neurotransmission: Flumazenil and bromocriptine. No formal
recommendations in the guidelines. Essentially no role in practice today. Flumazenil
in particular is extremely short acting.
(c) l-ornithine l-aspartate: Showed improvement in psychometric testing and postprandial
ammonia concentrations in individuals with persistent HE in a randomized control
trial. Oral supplementation has not proved effective. According to the guidelines, can
be used as an alternative or additional agent in those whose condition is unresponsive to
conventional therapies

ACCP Updates in Therapeutics® 2016: Ambulatory Care Pharmacy Preparatory Review and Recertification Course

1-556
Gastrointestinal Disorders

(d) Branched-chain amino acids: Oral therapy. Meta-analysis of eight randomized controlled
trials reports improvement in episodic HE, whether OHE and minimal HE. According
to the guidelines, may be used as alternative or additional agent to treat those whose
condition is unresponsive to conventional therapy
(e) Metabolic ammonia scavengers: Ornithine phenylacetate and glyceryl phenylbutyrate
remove ammonia from the circulation by binding with its substrates; effective in reducing
ammonia concentrations and improving cognitive function. More clinical trial data are
necessary before any recommendations regarding therapy
(f) Probiotics and microbiologic dietary supplements may reduce the necessary substrates of
pathogenic bacteria and supply fermentation products for beneficial bacteria. Preliminary
data are positive for minimal HE, but further studies are needed.
(g) Polyethylene glycol 3350 electrolyte solution resulted in faster HE resolution than
rifaximin in patients with cirrhosis who were hospitalized for acute HE. Administered
polyethylene glycol 3350 electrolyte dose was 4 L for 4 hours orally or through
nasogastric tube (JAMA Intern Med 2014:174:1727-33).
g. Role of the pharmacist (Domain 1, Task 5; Domain 2, Task 1,5; Domain 5, Task 2)
i. Patients with HE are often nonadherent to lactulose and antibiotic therapies because of
frequency of administration, adverse effects, cost, and recurrences of confusion.
ii. Pharmacists in the ambulatory setting can help ensure that patients adhere to prescribed
regimens. While performing medication reconciliation, pharmacists can identify patients
with HE requiring therapy and review their medication profile to ensure that treatment
is appropriately prescribed and to assess the patient’s adherence to therapy. Frequent and
continual education may be necessary so that the patient understands the medication’s
purpose and continues taking it, even during symptom-free periods. Counseling on patient-
managed titration of lactulose therapy is often necessary.

Patient Case

5. A 56-year-old man presents with Child-Pugh class B cirrhosis secondary to alcohol abuse. Other past medical
conditions include chronic renal insufficiency and gout. Vital signs include temperature 98.7°F, heart rate
91 beats/minute, respiratory rate 18 breaths/minute, and blood pressure 126/87 mm Hg. Today’s laboratory
values are within normal limits, except for AST 120 IU/mL, ALT 187 IU/mL, INR 2.0, hemoglobin 8.6 g/dL,
platelet count 76,000/mm3, and SCr 1.6 mg/dL. Current medications include spironolactone 100 mg orally
daily, furosemide 40 mg orally daily, and a daily multivitamin. He is allergic to sulfa and shellfish. During
the visit, you notice the patient seems more confused than usual, with slurred speech and asterixis. S.P.
cannot give a good medical history, so you question his wife, who states that the confusion and disorientation
started the past week. He has not received treatment for HE. Which therapeutic choice would be best?
A. Rifaximin 550 mg twice daily by mouth and lactulose by mouth as needed.
B. Neomycin 1000 mg by mouth every 6 hours.
C. Lactulose 45 mL by mouth three times daily.
D. Lactulose 45 mL/hour by mouth until evacuation occurs; then titrate dose as needed to achieve three
bowel movements a day.

ACCP Updates in Therapeutics® 2016: Ambulatory Care Pharmacy Preparatory Review and Recertification Course

1-557
Gastrointestinal Disorders

5. Gastroesophageal varices
a. Liver cirrhosis results in architectural distortion of the liver, leading to increased portal pressure
(portal hypertension). Varices, or alternative routes of blood flow, develop to overcome this
resistance. They can occur at any level of the GI tract; however, esophageal varices are the most
clinically significant. Hepatic venous pressure gradient (HVPG) is the pressure difference between
the portal vein and the inferior vena cava; normal HVPG is 3–6 mm Hg; when HVPG exceeds 10
mm Hg, varices are more likely to develop.
b. Varices are present in about 50% of patients with cirrhosis; presence correlates with severity of
liver disease; 40% in Child-Pugh class A compared with 85% in Child-Pugh class C. Variceal
bleeding carries a high mortality; prophylaxis is thus important.
c. Varices may progress to variceal bleed.
i. Strong predictors of variceal bleeding include the following:
(a) Varix size (those greater than 5 mm have the highest risk of bleed)
(b) Tension on variceal wall, which is related to the degree of cirrhosis (Child-Pugh class C
highest risk of bleed)
(c) Presence of ascites or tense ascites
(d) History of variceal bleed
ii. Variceal bleeds are a potentially life-threatening complication of cirrhosis. After the initial
bleed, mortality is 5%–50%, depending on the severity of the underlying liver disease. Risk of
rebleeding after initial episode is 60%–70%.
iii. Management of acute bleed in hospitalized patients (Am J Gastroenterol 2009;104:1802-29). Note:
Occurs in hospitalized patients; brief overview is provided in this chapter for completeness
(a) Should aim to promptly diagnose, control bleeding, and prevent complications
(b) Volume expansion and hemodynamic stabilization are strategies used to control bleeding.
(c) Treatment options (Domain 1, Task 3,4,6,7)
(1) Once the patient is hemodynamically stable, an EGD should be performed as soon as
possible to visualize the extent of disease and assess the potential to intervene.
(A) Variceal band ligation (endoscopic variceal ligation) is done once source of
bleeding is confirmed; best within 12 hours of admission. Preferred intervention:
More effective with fewer complications. Adverse events include temporary
dysphagia, chest discomfort/pressure, and banding ulcers.
(B) Sclerotherapy is less effective; greater risk of complications; performed when
band ligation cannot be performed
(2) Pharmacotherapy: Causes splanchnic vasoconstriction, subsequently decreasing
portal blood flow. Octreotide by intravenous drip is generally used first line.
(3) Antibiotic prophylaxis (short term) reduces mortality in patients with cirrhosis who
have an upper GI bleed. (See section on SBP.)
(4) Other options typically used when EGD and drugs fail; may include balloon
tamponade or TIPS shunt (performed to shunt blood out of portal circulation)
d. Prevention
i. Primary prophylaxis
(a) Once a diagnosis of cirrhosis is made, a screening EGD is recommended.
(b) Objective of EGD is to detect the presence, size, and appearance of varices.

ACCP Updates in Therapeutics® 2016: Ambulatory Care Pharmacy Preparatory Review and Recertification Course

1-558
Gastrointestinal Disorders

(c) Patients with medium to large varices should be given primary prophylaxis against
variceal bleeding. Patients with small varices are not considered for prophylaxis unless
they are at increased risk of bleeding, although the quality of evidence for these patients
is poor. Patients with small varices at increased risk of bleeding include patients who are
Child-Pugh class B/C (advanced liver disease) or whose varices have red wale marks.
Accepted therapies for primary prophylaxis are as follows:
(1) Nonselective β-blockers reduce portal pressure by decreasing cardiac output (β1-
blockade effect) and reducing portal blood inflow by splanchnic vasoconstriction
(β2-blockade effect). Highly effective in decreasing the incidence of variceal bleed
(A) Agents
• Propranolol 20 mg orally twice daily
• Nadolol 40 mg orally once daily
(B) Guidelines recommend giving the maximum tolerated dose of non-selective
β-blocker. Aiming for a heart rate goal of 55–60 beats/minute is reasonable if
blood pressure tolerates
(C) Caution in patients with bronchospastic disease, patients with peripheral
vascular disease or Raynaud’s disease, and some patients with diabetes (may
potentiate hypoglycemia and/or mask signs and symptoms)
(D) Adverse effects include light-headedness, fatigue, shortness of breath,
bradycardia, and sexual dysfunction.
(E) Head-to-head comparator trials have not been conducted; however, nadolol
seems to have fewer adverse effects and can be dosed once daily at bedtime.
(2) Endoscopic variceal ligation is a local therapy in which bands are placed around
varices; variceal obliteration may be an alternative to β-blocker.
(A) Repeated every 1–2 weeks until obliteration of varices
(B) Adverse effects are minor; usually transient dysphagia and chest discomfort and
risk of banding ulcers
(C) Once corrected, perform EGD 1–3 months afterward; then every 6–12 months
(3) Therapies to avoid
(A) Long-acting nitrates (isosorbide mononitrate or dinitrate) are not recommended
for primary prophylaxis; lack of efficacy and associated with higher mortality as
monotherapy in some studies
(B) Endoscopic sclerotherapy because of controversial results and shunt surgery
because of more frequent encephalopathy and mortality
ii. Secondary prophylaxis
(a) Patients surviving variceal bleed episodes have a very high risk of rebleeding or death;
thus, ALL patients with bleed history must receive secondary prophylaxis.
(b) A combined approach with endoscopic variceal ligation and nonselective β-blockers is
recommended.
e. Role of the pharmacist (Domain 1, Task 5; Domain 2, Task 1,5; Domain 5, Task 2)
i. Pharmacists in the ambulatory setting are in a unique position to ensure that patients
requiring prophylaxis continue therapy.
ii. Continual education may be necessary so that the patient understands the medication’s
purpose and continues taking it.

ACCP Updates in Therapeutics® 2016: Ambulatory Care Pharmacy Preparatory Review and Recertification Course

1-559
Gastrointestinal Disorders

Patient Case

6. A 60-year-old man with Child-Pugh class C cirrhosis secondary to HCV infection, hepatocellular carcinoma,
HE, and type 2 diabetes returns today for a follow-up. He takes rifaximin 550 mg twice daily, lactulose
20 g twice daily, famotidine 20 mg twice daily, a multivitamin once daily, and calcium when he remembers
it. He reports no allergies to medications. He states that his diabetes is controlled by diet. He had an EGD
2 weeks ago that revealed several medium-sized varices. He has no history of a GI bleed. Which is the best
recommendation regarding prophylaxis against variceal bleeding?
A. Atenolol.
B. Isosorbide mononitrate.
C. Propranolol.
D. Prophylaxis not recommended.

IV.  VIRAL HEPATITIS

A. General Definitions
1. Acute viral hepatitis (hepatitis A, B, or C virus: HAV, HBV, or HCV): Infection for less than 6 months
2. Chronic HBV, or HCV infection: Infection for more than 6 months
3. Table 8 reports the viral hepatitis burden in the United States.

Table 8. Estimated Viral Hepatitis Disease Burden in the United States in 2013a
Acute Disease Chronic Disease
No. of Individuals Living with
Reported New Cases Estimated New Cases
Chronic Infection
Hepatitis A 1781 3473b 0
Hepatitis B 3050 19,764 c
700,000 – 1.4 million
Hepatitis C 2138 29,718 d
3.2 million
Data from: Centers for Disease Control and Prevention. Atlanta: CDC. Available at www.cdc.gov. Accessed October 15, 2015.
a

b
Actual acute cases estimated to be 1.95 times the number of reported cases in any year.
Actual acute cases estimated to be 6.48 times the number of reported cases in any year.
c

d
Actual acute cases estimated to be 13.9 times the number of reported cases in any year.

B. Hepatitis A Virus (Domain 2, Task 5)


1. Background
a. RNA virus associated with an acute, self-limited hepatitis
b. Transmitted from person to person through the fecal-oral route (primary mode of transmission) or
ingestion of contaminated food or water
c. Incidence correlates with low socioeconomic status, overcrowding, and poor sanitation.
2. Clinical presentation and diagnosis
a. Average incubation period is 28 days (range 15–50 days).
b. Clinical symptoms are not always present; when they do occur, they are often nonspecific and appear
abruptly and may include fatigue, abdominal pain, loss of appetite, nausea, vomiting, jaundice, and pruritus.
c. History to identify recent potential exposures
d. Laboratory data, including elevation of aminotransferase values and detection of IgM
(immunoglobulin M) antibody to HAV (anti-HAV)

ACCP Updates in Therapeutics® 2016: Ambulatory Care Pharmacy Preparatory Review and Recertification Course

1-560
Gastrointestinal Disorders

3. Management of acute infection


a. No specific treatment; however, if known when HAV contracted, administration of immunoglobulin
or vaccine may be indicated, depending on the specific situation (see section below)
b. Supportive therapy
c. Avoid hepatotoxic insults (e.g., alcohol consumption, hepatotoxic medications).
4. Prevention
a. Avoid exposure, and institute good hand-washing techniques and personal hygiene practices.
b. Preexposure prophylaxis (Table 9)
i. Active (vaccine) or passive (immunoglobulin) is available.
(a) HAV single-antigen vaccines (Havrix and Vaqta) – Pediatric and adult formulations
available; see Table 9. More than 95% develop protective antibodies after single dose.
(b) Combination product (Twinrix); contains HAV and HBV vaccines; adult formulation available
(c) Immunoglobulin dose, given intramuscularly: 0.02 mL/kg (for protection up to
3 months) or 0.06 mL/kg (for protection of 3 months or more; repeat after 5 months
if travel prolonged)

Table 9. Available Single-Antigen Hepatitis A Vaccines


Age Dose/Volume No. of Doses Schedule, mo
Havrix 12 mo to 18 yr 720 EL.U. (0.5 mL) 2 0, 6–12
(GlaxoSmithKline) ≥ 19 yr 1440 EL.U. (1.0 mL) 2 0, 6–12
VAQTA 12 mo to 18 yr 25 units (0.5 mL) 2 0, 6–18
(Merck & Co.) ≥ 19 yr 50 units (1.0 mL) 2 0, 6–18
EL.U. = enzyme-linked immunosorbent assay units.

ii. Routine HAV vaccine is recommended in the following populations:


(a) All children older than 1 year (ideally at 12–23 months)
(b) Individuals at increased risk of infection or severe outcomes, including the following:
(1) Men who have sex with men
(2) Users of injection and non-injection illegal drugs
(3) Those with occupational risk of infection (e.g., individuals working with HAV-
infected primates or with HAV in a research setting)
(4) Those with chronic liver disease
(5) Those with clotting factor disorders
(c) Individuals wishing to obtain immunity
(d) Those traveling to, or working in, countries with high or intermediate HAV endemicity:
www.cdc.gov/travel/contentdisease.aspx (accessed October 15, 2015)
c. Administer HAV immunoglobulin to the following populations:
i. Children younger than 1 year at risk of infection
ii. Travelers to countries with high or intermediate endemicity as an alternative if vaccine cannot
be given, or as an adjunct to vaccine in those planning travel in 2 weeks or sooner who are
older than 40 years or immunocompromised or have chronic liver disease

ACCP Updates in Therapeutics® 2016: Ambulatory Care Pharmacy Preparatory Review and Recertification Course

1-561
Gastrointestinal Disorders

5. Postexposure prophylaxis
a. If individual has received vaccination, no further treatment is needed.
b. If individual has not received vaccination, he or she should receive vaccine or immunoglobulin
as soon as possible within 2 weeks of the exposure. Immunoglobulin dose is 0.02 mL/kg
intramuscularly. Specific patient populations are as follows:
i. Healthy individuals 12 months to 40 years of age should receive HAV vaccine at an age-
appropriate dose because of the ease of administration and long-term protection.
ii. For individuals older than 40 years, immunoglobulin is preferred because of the lack of data on
the vaccine in this patient population and the possibility of severe manifestations of disease.
iii. Immunoglobulin is also preferred in individuals younger than 12 months, in the
immunocompromised population, and in those with chronic liver disease.
6. Patient education resource available at www.cdc.gov/hepatitis/HAV/PatientEduHAV.htm
(accessed October 15, 2015)

C. Hepatitis B Virus (Domain 1, Task 1-7, Domain 5, Task 2)


1. Background
a. Associated with acute and chronic disease. Risk of acute disease developing into chronic infection
decreases with age; risk of developing chronic infection is 90% in neonates and 10% in adults.
b. DNA virus; replication takes place in the liver
c. Eight genotypes (A–H) identified; prevalence varies according to geographic location. In the
United States, all genotypes have been observed. Data suggest disease progression is linked to
genotype. Current guidelines do not recommend genotype testing.
d. Transmission
i. Through exposure to infectious blood or bodily fluids (semen, saliva, and vaginal)
ii. Routes
(a) Body fluids (e.g., saliva, semen, vaginal fluid)
(b) Parenteral (e.g., needle-sharing, needlestick injuries)
(c) Perinatal
e. Natural history
i. Complex disease course; infected individuals may pass through various stages of active and
inactive disease
ii. Strong risk factor for developing liver cirrhosis and hepatocellular carcinoma with chronic infection
(a) Risk factors for developing hepatocellular carcinoma include male sex, older age and
positive family history, and HCV/HIV coinfection as well as patients acquiring infection at
a young age (i.e., perinatal transmission). HBV disease–related factors include prolonged
HBV DNA elevation, ALT elevation, and presence of hepatitis B early antigen (HBeAg).
(b) Screening for hepatocellular cancer is recommended in the population with chronic HBV.
2. HBV serology – Serologic markers provide information regarding immunity and acute and chronic
infection (Tables 10 and 11).

ACCP Updates in Therapeutics® 2016: Ambulatory Care Pharmacy Preparatory Review and Recertification Course

1-562
Gastrointestinal Disorders

Table 10. HBV Serology


Serologic Marker Abbreviation Comments
Indicates person is infectious; present during acute and
Hepatitis B surface antigen HBsAg
chronic infection
Confers protective immunity; present after recovery from
Hepatitis B surface antibody Anti-HBs
acute infection or after vaccination
Appears at symptom onset and persists for life; denotes
Hepatitis B core antibody Anti-HBc
previous or ongoing infection
Hepatitis B E (early) antigen HBeAg Indicates ongoing viral replication
Appears temporarily during acute infection or consistently
Hepatitis B E (early) antibody Anti-HBe
after burst of viral replication
Hepatitis B DNA HBV DNA Quantifies HBV; indicates active replication
HBV = hepatitis B virus.
Adapted from: Centers for Disease Control and Prevention. Viral Hepatitis. Atlanta: CDC. Available at www.cdc.gov/hepatitits. Accessed October 15, 2015.

Table 11. Interpretation of HBV Serologic Test Results


HBsAg Anti-HBs Anti-HBc IgM Anti-HBc
Susceptible (no infection, no immunization) – – – N/A
Acute infection + – + +
Chronic infection + – + –
Immune (natural infection) – + + –
Immune (vaccination) – + – N/A
Unclear; possible interpretations: –
Resolved infection with low titer anti-HBs
False positive anti-HBc – – + (except IgM will
Chronic infection (low level) be positive if
Window phase of acute infection window phase)
anti-HBc = hepatitis B core antibody; anti-HBs = hepatitis B surface antibody; HBsAg = hepatitis B surface antigen; IgM anti-HBc = IgM antibody to
hepatitis B core antigen; N/A = not applicable.
Adapted from: Centers for Disease Control and Prevention. Viral Hepatitis. Atlanta: CDC. Available at www.cdc.gov/hepatitits. Accessed October 15, 2015.

3. Chronic infection
a. The natural history of chronic HBV infection is dynamic and consists of various phases of
disease (i.e., immune tolerant, immune active, and inactive). Each phase can last for years and can
suddenly change. See Table 12.
i. Immune-tolerant phase: Characterized by active viral replication with minimal evidence of
disease activity. Liver disease does not appear to progress during this phase. Occurs most
often in those infected by perinatal transmission; these individuals typically remain in this
phase for many decades

ACCP Updates in Therapeutics® 2016: Ambulatory Care Pharmacy Preparatory Review and Recertification Course

1-563
Gastrointestinal Disorders

ii. Immune-active phase (sometimes called the “chronic hepatitis B” or “immune clearance”
phase): Transition to this phase occurs as the immune response to HBV becomes more robust.
The host’s immune system recognizes the virus and initiates an immune response. Significant
liver damage may occur during this phase.
(a) Characterized by the presence of HBeAg and elevated and/or fluctuating HBV DNA and
ALT values
(b) Individuals infected after birth (i.e., horizontal transmission) may advance to this phase
shortly after infection, whereas individuals infected at birth (i.e., perinatal transmission)
may transition to this phase after many years/decades in the immune-tolerant phase.
(c) Within this phase, infected individuals are categorized as HBeAg positive or HBeAg negative.
(1) HBeAg positive
(A) An outcome of this phase may be seroconversion, the clearance of HBeAg, and
the development of hepatitis B early antibody (anti-HBe) in an individual who
was previously HBeAg positive and anti-HBe negative. Seroconversion occurs
either spontaneously (rate of 8%–12% per year) or because of antiviral therapy.
(B) After seroconversion, most individuals transition to the inactive phase.
(2) HBeAg negative
(A) This population results from various genetic mutations that affect transcription
and translation. A precore mutation preventing the production of HBeAg is the
most common cause.
(B) Monitoring for the clearance of HBeAg (or seroconversion) is not useful in these cases.
iii. Inactive “carrier” phase: Transition to this phase occurs after seroconversion.
(a) Characterized by persistent infection without ongoing liver disease. This is accompanied
by low or undetectable HBV DNA levels, leading to ALT normalization and reduced
liver inflammation.
(b) 4%–20% will undergo one or more reversions back to the immune-active phase – HBV
reactivation
(c) Some eventually will clear hepatitis B surface antigen (HBsAg) (about 0.5% annually);
these are considered to have “resolved HBV” (or lack of virologic, biochemical, or
histologic evidence of disease). This state is called “occult” or “latent” HBV.

Table 12. Summary of Chronic HBV Phases


Immune Tolerant Immune Active Inactive
HBsAg Positive Positive Positive Positive
HBeAg Positive Positive Negative Negative
Anti-HBe Negative Negative Positive Positive
HBV DNA, IU/mL > 20,000 > 20,000 2000–20,000 < 2000
ALT/AST Normal Elevated Elevated Normal
Confirms absence of Confirms absence of
Liver biopsy Active inflammation with or without fibrosis
significant hepatitis significant hepatitis
ALT = alanine aminotransferase; anti-HBe = hepatitis B early antibody; AST = aspartate aminotransferase; HBeAg = hepatitis B early antigen.

ACCP Updates in Therapeutics® 2016: Ambulatory Care Pharmacy Preparatory Review and Recertification Course

1-564
Gastrointestinal Disorders

b. Diagnosis (Domain 1, Task 1; Domain 2, Task 2; Domain 4, Task 3)


i. Clinical signs and symptoms: Abdominal pain, diarrhea, fever, jaundice, myalgia, nausea,
and vomiting (many patients are asymptomatic [around 30%])
ii. Initial evaluation should include the following:
(a) History, with special emphasis on risk factors for alcohol use, coinfection, and family
history of liver disease and hepatocellular carcinoma
(b) Physical examination
(c) Laboratory and serologic testing, including the following: Complete blood cell count
[CBC]; hepatic panel; serologic markers (i.e., HBsAg, hepatitis B core antibody [anti-
HBc], hepatitis B surface antibody [anti-HBs]); HBV DNA quantification based on PCR
amplification; typical lower limits of detection are 50–200 IU/mL; more recently, a more
sensitive assay can detect a lower limit of 5–10 IU/mL
iii. Serum alfa fetoprotein and ultrasonography to evaluate for hepatocellular carcinoma
iv. On the basis of the initial findings, a liver biopsy may be indicated to further evaluate the
degree of liver inflammation and/or fibrosis.
c. Treatment (according to the AASLD guidelines: Hepatology 2009;50:1-36): Primary goals are to
achieve viral suppression, decrease necroinflammation of the liver, reverse fibrosis, and prevent the
progression of liver disease (cirrhosis, liver failure, hepatocellular carcinoma) and ultimately death.
d. Response (efficacy): Current therapies suppress but do not eradicate the virus, and long-term
efficacy is unknown.
e. Indication for treatment
i. Guidelines recommend that decisions regarding treatment be made according to clinical status,
HBeAg status, HBV DNA levels, ALT concentration, and liver histology (when available).
ii. Treatment should be initiated regardless of HBV DNA or ALT values for individuals with life-
threatening liver disease (i.e., acute liver failure, decompensated cirrhosis, severe exacerbation of
HBV disease, jaundice) and in those awaiting liver transplantation. Antiviral therapy to decrease
viral suppression decreases the risk of disease recurrence post-transplantation.
iii. Treatment recommendations according to phase of chronic HBV disease (AASLD guidelines;
Hepatology 2009;50:1-36)
(a) Immune tolerant (HBeAg positive, elevated HBV DNA, normal ALT)
(1) No treatment indicated
(2) Monitor ALT every 3–6 months.
(3) If ALT elevation occurs, continue to monitor for another 3–6 months for spontaneous
conversion (HBeAg positive to anti–HBe positive) before considering therapy.
(4) If the ALT elevation continues without HBeAg conversion, treat the patient as
immune active (see below).
(5) Patients older than 40 years in whom HBeAg remains positive and HBV DNA
remains greater than 20,000 IU/mL should be considered for liver biopsy, and
treatment should be considered if biopsy shows moderate/severe inflammation or
significant fibrosis.
(b) Inactive (HBeAg negative, anti–HBe positive, HBV DNA less than 2000 IU/mL, normal ALT)
(1) No treatment indicated
(2) Monitor ALT concentration every 3 months for the first year to verify truly inactive
carriers; then every 6–12 months
(3) If ALT concentration becomes elevated, increase monitoring frequency; may need to
include HBV DNA monitoring to check for reactivation

ACCP Updates in Therapeutics® 2016: Ambulatory Care Pharmacy Preparatory Review and Recertification Course

1-565
Gastrointestinal Disorders

(c) Immune active (HBeAg positive, HBV DNA greater than 20,000 IU/mL, with ALT elevation)
(1) Delay treatment decisions for 3–6 months in those with compensated liver disease to
determine whether spontaneous HBeAg seroconversion occurs.
(2) If ALT greater than 2 x upper limit of normal (ULN) for 3–6 months (with HBeAg
remaining positive and HBV remaining greater than 20,000 IU/mL), treatment
should be considered
(3) If ALT 1–2 x ULN for 3–6 months (with HBeAg remaining positive and HBV
remaining greater than 20,000 IU/mL), consider liver biopsy, especially in those
older than 40 years, and consider treatment if biopsy shows moderate/severe
inflammation or significant fibrosis
(d) Immune active (HBeAg negative, HBV DNA greater than 2000 IU/mL, with ALT elevation)
(1) ALT greater than 2 x ULN: Consider treatment.
(2) HBV DNA 2000–20,000 IU/mL and borderline normal or minimally elevated ALT
concentration (1–2 x ULN): Consider liver biopsy. If biopsy shows moderate/severe
inflammation or significant fibrosis, consider treatment.
iv. Treatment considerations in special populations
(a) Those with compensated cirrhosis: Consider treatment when serum HBV DNA is greater
than 2000 IU/mL.
(b) Those with decompensated cirrhosis: Initiate treatment promptly regardless of HBV
DNA or ALT concentrations.
(1) Nucleoside or nucleotide analogs (NAs) producing rapid viral suppression with low
risk of drug resistance is recommended; entecavir or tenofovir
(2) Interferon-α or pegylated interferon is contraindicated and should not be used.
(3) See a liver transplant center.
(c) HBV carriers are at risk of hepatitis flares when undergoing cancer chemotherapy and
immunosuppressive therapies such as rituximab, infliximab, and other anti–tumor
necrosis factor (TNF) agents.
(1) Patients should be screened for HBsAg and anti-HBc before initiation of
chemotherapy or immunosuppression.
(2) Those who are HBsAg positive should be offered antiviral prophylaxis.
(3) Lamivudine has been studied well in this setting, but because of resistance, entecavir
or tenofovir is preferred for courses greater than 1 year or if HBV DNA is detectable
at baseline.
(4) If baseline HBV DNA is less than 2000 IU/mL, prophylaxis should continue for 6
months after cessation of immunosuppressive therapy. If baseline DNA is greater
than 2000 IU/mL, antiviral therapy should continue until treatment end points are
met, as with immunocompetent patients.
(5) Patients with isolated anti-HBc and those with anti-HBc plus anti-HBs should be
monitored and therapy initiated if HBV DNA becomes detectable.
f. Pharmacotherapy (Domain 1, Task 3,4,6,7)
i. Suppresses viral replication by antiviral effects or immune modulation. Currently, interferons
or NAs are approved for treating adults with chronic HBV in the United States. See Table 13
for comparison.
ii. Guideline recommendations
(a) First-line therapy: Pegylated interferon, entecavir, or tenofovir as monotherapy
(b) Choice of therapy is patient-specific; should be selected according to patient profile,
treatment history, and safety and efficacy of medications

ACCP Updates in Therapeutics® 2016: Ambulatory Care Pharmacy Preparatory Review and Recertification Course

1-566
Gastrointestinal Disorders

(c) In individuals with cirrhosis (compensated or decompensated): NAs are preferred to interferon
because of the risk of hepatic flares and decompensation associated with interferons.

Table 13. Comparison of HBV Treatment Strategies


Advantages Disadvantages
Subcutaneous administration
Defined therapy duration
PEG-IFN Significant adverse event profile
Minimal resistance
High cost
Oral administration
Indefinite therapy duration
Variable costs among agents
Nucleoside/nucleotide analogs Development of resistance
Improved efficacy
High cost
Better safety profile

iii. Interferon
(a) Several mechanisms of action, including antiviral, antiproliferative, and
immunomodulatory effects
(b) Dosing
(1) Pegylated interferon – Pegylated interferon-α2a (PEGASYS): 180 mcg
subcutaneously once weekly. Only pegylated agent that is FDA approved for HBV.
Available in single-dose vials, in a four-pack of prefilled 180-mcg syringes, and as a
prefilled autoinjector pen (PEGASYS ProClick). The autoinjector device is ready to
use with the correct dose; no mixing or dialing is necessary.
(2) Conventional interferon-α has been replaced with pegylated interferon because of
frequent dosing with conventional interferon and some data for improved efficacy
with pegylated interferon. Conventional interferon-α required 5 million units/day or
10 million units three times weekly.
(c) Duration
(1) Interferon offers option for defined course of therapy compared with NA agents.
(2) Pegylated interferon-α2a: 48 weeks for HBeAg-positive or HBeAg-negative disease
(3) Conventional interferon-α: 16–24 weeks for HBeAg-positive disease and 12–24
months for HBeAg-negative disease
(d) Efficacy
(1) Patients with HBeAg-positive disease receiving pegylated interferon-α2a therapy for
48 weeks had a 25% reduction in HBV DNA levels and a 30% loss of HBeAg.
(2) Factors associated with a favorable response to treatment in patients with HBeAg-
positive disease include high pretreatment ALT values, low HBV DNA level, HBV
genotypes A and B, and active inflammation on liver biopsy.
(3) No factors have a consistently predicted response in patients with HBeAg-negative disease.
(e) Monitoring
(1) Baseline: Although guidelines provide no specific recommendations, the following
laboratory values are typically obtained before therapy initiation: CBC with differential,
hepatic panel, thyroid-stimulating hormone (TSH), HBV serology, and HBV DNA. Other
laboratory tests may be indicated, depending on the individual situation.

ACCP Updates in Therapeutics® 2016: Ambulatory Care Pharmacy Preparatory Review and Recertification Course

1-567
Gastrointestinal Disorders

(2) While receiving treatment


(A) CBC with differential and hepatic panel every 4 weeks according to treatment
guidelines; however, in clinical practice, monitoring typically occurs more
frequently: Every 2 weeks for 6–8 weeks and then monthly once stable.
If abnormalities develop during treatment, monitoring frequency may
require adjustment.
(B) TSH and HBV DNA every 12 weeks. In addition, data have shown that
therapy (i.e., interferons) may affect lipids; thus, in clinical practice, some
clinicians monitor fasting lipid panels at baseline and every 12 weeks during
therapy; however, current treatment guidelines provide no specific monitoring
recommendations.
(C) HBeAg and anti-HBe every 24 weeks (if HBeAg-positive disease at start of therapy)
(3) Posttreatment
(A) CBC with differential, hepatic panel, and TSH concentration
(B) HBV DNA level every 12 weeks during the first 24 weeks after treatment
(f) Adverse effects: Many serious effects are associated with interferon administration;
frequent and vigilant monitoring is necessary. (NOTE: This list is not all-inclusive; please
see prescribing guidelines for additional information.)
(1) Transient elevations in liver function test values often occur after treatment initiation;
therapy should be continued.
(2) All interferons have similar adverse effect profiles.
(A) Bone marrow suppression, especially neutropenia and thrombocytopenia;
dose reduction or discontinuation may be necessary. Follow manufacturer’s
recommendations. Supplemental therapies for management are often used in
clinical practice.
(B) Headache, flu-like symptoms, nausea, anorexia
(C) Psychosis, depression, anxiety, sleep disturbances (occur in 22%–30% of
patients). Suicide in less than 1%
(D) Thyroid abnormalities
(E) Neuropathy
(F) Injection-site reactions, rash. Educate patient on proper administration
techniques, including rotating injection site.
(G) Worsen existing autoimmune disorders, increase infectious risk
iv. NAs
(a) Function is to inhibit DNA synthesis.
(b) Five approved for the treatment of chronic HBV: Adefovir, entecavir, lamivudine,
telbivudine, and tenofovir; preferred agents are entecavir and tenofovir
(c) Black box warning for all agents on the development of lactic acidosis
(d) Rebound hepatitis (flare in liver enzymes), likely on therapy discontinuation, is a common
adverse effect with all NAs.
(e) Monitoring
(1) Although guidelines provide no specific recommendations, the following laboratory
tests are typically obtained before therapy initiation: Hepatic panel, renal panel, HBV
serology, and HBV DNA. Other laboratory tests may be indicated depending on the
individual situation.
(2) While receiving treatment
(A) Hepatic panel every 12 weeks
(B) SCr should be checked every 12 weeks for those taking adefovir or tenofovir.

ACCP Updates in Therapeutics® 2016: Ambulatory Care Pharmacy Preparatory Review and Recertification Course

1-568
Gastrointestinal Disorders

(C) HBV DNA testing every 12–24 weeks


(D) HBeAg and anti-HBe testing every 24 weeks (if HBeAg-positive disease at start
of therapy)
(E) HBsAg testing every 6–12 months in individuals with HBeAg-negative disease
who have persistently undetectable HBV DNA levels
(3) Posttreatment – Guidelines provide no recommendations.
(f) Antiviral resistance was a major concern with the earlier NAs; however, it is less of an
issue with the newer NAs (i.e., entecavir and tenofovir). See Table 14.
(1) Development of resistant mutants (i.e., genotypic resistance) may be followed by
increases in HBV DNA and AST/ALT. Hepatitis flares and hepatic decompensation are
possible with resistance; thus, early detection and intervention is necessary for prevention.
(2) Cross-resistance among NAs may limit future treatment options.
(g) Treatment end points
(1) Loss of HBeAg and gain of anti-HBe; loss of HBsAg and gain of anti-HBs (rare)
(2) Suppression of HBV DNA to undetectable levels by sensitive PCR assays
(h) Treatment duration
(1) Optimal duration has not yet been established.
(2) HBeAg positive – Continue for a minimum of 12 months, and at least 6 months
after the loss of HBeAg and the appearance of anti-HBe; however, in clinical
practice, most therapies continue for 12 months after seroconversion. Not all patients
seroconvert; therefore, therapy may be indefinite.
(3) HBeAg negative/anti-HBe positive – Therapy duration is unknown; however,
in clinical practice, most are treated indefinitely (or until the rare occurrence
of HBsAg clearance).
(i) Treatment efficacy
(1) All agents suppress viral replication with similar efficacy; however, some of the
newer agents are likely superior. In general, first-line therapies are selected on the
basis of resistance rates.
(2) Phase III clinical trials (Gastroenterology 2012;142:1360-8)
(A) HBeAg positive: After 1 year of NA therapy:
• Undetectable HBV DNA (21%–76%); increased to 94%–98% of patients
treated with entecavir or tenofovir for 5 years
• Normalization of ALT concentration (41%–77%)
• HBeAg seroconversion (12%–22%); increased to 31%–48% after 4–5 years
of therapy
• Loss of HBsAg (0%–3%); after 4–5 years of therapy, remains low (0%–10%)
(B) HBeAg negative
• Undetectable HBV DNA (51%–93%)
• Normalization of ALT concentration (62%–78%)
• Loss of HBsAg (0%–1%); after 4–5 years of therapy, remains low (0%–5%)
(3) Predictors of response in patients with HBeAg-positive disease: High pretreatment
ALT concentrations
(4) Predictors of response in patients with HBeAg-negative disease: Have not been defined
(5) Current guidelines recommend entecavir and tenofovir as first-line NA therapy.

ACCP Updates in Therapeutics® 2016: Ambulatory Care Pharmacy Preparatory Review and Recertification Course

1-569
Gastrointestinal Disorders

(j) NA therapy: Preferred options


(1) Entecavir (Baraclude)
(A) HBV replication is inhibited at three steps: HBV DNA polymerase priming,
reverse transcriptase, and synthesis of RNA-positive strand.
(B) Dose: 0.5 mg/day orally for nucleoside naive; 1.0 mg/day orally for decompensated
liver disease and for those with documented lamivudine or telbivudine resistance
(although with availability of tenofovir, it is not recommended to use entecavir in
those with such resistance) – Adjust for renal impairment.
(C) Resistance: 0% at 1 year and 1.2% at 5 years; however, risk of resistance much
higher in patients with lamivudine resistance: 51% after 5 years of therapy
(D) Adverse event profile: Similar to that of lamivudine in clinical trials
(2) Tenofovir disoproxil fumarate (Viread)
(A) Inhibits HBV polymerase
(B) Dose: 300 mg/day orally – Adjust for renal impairment.
(C) Resistance: 0% and 1 and 5 years. Effective for lamivudine resistance
(D) Adverse event profile: Similar to that of other agents, with added potential for
osteomalacia and renal toxicity
(k) NA therapy: Non-preferred options
(1) Lamivudine (Epivir-HBV)
(A) Triphosphate (3TC-TP) is incorporated into the DNA chain, resulting in
premature termination; thus, DNA synthesis is inhibited.
(B) Dose: 100 mg/day orally; higher dose in HBV-HIV coinfection (300 mg/day) –
Adjust for renal impairment.
(C) Because of high resistance rates, this agent is no longer used as first-line therapy.
Resistance rates: 27% at 1 year and 65% at 5 years. Lamivudine resistance
increases the risk of entecavir resistance but not the risk of tenofovir resistance.
(D) Most common mutation involves substitution of methionine in the tyrosine-
methionine-aspartate-aspartate (YMDD) motif of the HBV DNA polymerase for
valine or isoleucine.
(E) Adverse event profile: Similar to placebo in clinical trials; well tolerated; fatigue,
headache, and nausea
(2) Adefovir dipivoxil (Hepsera)
(A) Prodrug of adefovir, nucleotide analog of adenosine monophosphate. Inhibits
HBV viral RNA-dependent DNA polymerase
(B) Dose: 10 mg/day orally – Adjust for renal impairment.
(C) Resistance: 0% at 1 year and 42% at 5 years. Adefovir resistance decreases
susceptibility to tenofovir. Effective in suppressing lamivudine-resistant HBV
(D) Adverse event profile: Well tolerated overall, similar to placebo in clinical
trials; fatigue, headache, nausea, vomiting. Boxed warning for nephrotoxicity,
especially in those with underlying renal dysfunction
(3) Telbivudine (Tyzeka)
(A) Inhibits HBV polymerase
(B) Dose: 600 mg/day orally – Adjust for renal impairment.
(C) Resistance: 4.4% at 1 year and 21% at 5 years. Telbivudine resistance increases
risk of entecavir resistance but not tenofovir resistance. Not effective if
lamivudine resistance

ACCP Updates in Therapeutics® 2016: Ambulatory Care Pharmacy Preparatory Review and Recertification Course

1-570
Gastrointestinal Disorders

(D) Adverse event profile: Similar to that of lamivudine in clinical trials; peripheral
neuropathy and myopathy reported

Table 14. Comparison of FDA-Approved Agents for Treatment of Adult Chronic HBV Infection
IFN-α Lamivudine Adefovir Entecavir Telbivudine Tenofovir
PEG-IFN (Epivir) (Hepsera) (Baraclude) (Tyzeka) (Viread)
Route of
Subcutaneous Oral Oral Oral Oral Oral
administration
Tablet Tablet Tablet
Formulations Solution Tablet Tablet
Solution Solution Powder
Nephrotoxic
Myopathy potential,
Nephrotoxic
Adverse events Many Minimal Minimal Peripheral decreased
potential
neuropathy bone min-
eral density
Incidence of 0% 1 yr;
27% 1 yr; 0% 1 yr; 4.4% 1 yr; 0% 1 yr; 0%
resistance (if N/A 1.2% 5 yr;
65% 5 yr 42% 5 yr 21% 2 yr 5 yr
lamivudine naive) 51% 5 yra
Costs High Low Moderate High Moderate High
Therapy duration
Conventional
Minimum of 12 mo of therapy. Continue 6 mo to 1 yr after anti-HBe
IFN-α: 16–24 wk
HBeAg positive seroconversion, and perhaps indefinitely if seroconversion does not
PEG-IFN-α:
occur
48 wk
Conventional
IFN- α: 1–2 yr
HBeAg negative > 12 mo; perhaps indefinitely unless HBsAg clearance is achieved
PEG-IFN-α:
48 wk
For those with a history of lamivudine-resistant HBV.
a

IFN-α = interferon alfa; N/A = not applicable.

(l) Management of treatment failures


(1) Nonresponders to previous interferon-α or pegylated interferon therapy: Treat with
NA if they meet the virologic, serologic, and histologic criteria described earlier.
(2) Did not achieve primary response to NA (defined as less than a 2-log decrease
in serum HBV DNA levels after 6 months of adherence to therapy): Switch to an
alternative treatment.
(3) Develop breakthrough infection while receiving NA therapy (increase of HBV DNA
by 10-fold from nadir on therapy in patients who achieved a primary response):
(A) Assess therapy adherence.
(B) Test for antiviral-resistant mutations, if possible.
(C) Salvage therapy should be considered, especially with decompensated disease
or hepatitis flares. Choice of therapy depends on current and prior therapies and
pattern of drug resistance mutations (see Table 15).

ACCP Updates in Therapeutics® 2016: Ambulatory Care Pharmacy Preparatory Review and Recertification Course

1-571
Gastrointestinal Disorders

Table 15. Salvage Therapy Recommendations According to the AASLD 2009 Guidelines and Current Clinical Practice
Failed Drug Salvage Therapy Options
(1) Add adefovir or tenofovir
Lamivudine (2) Switch to tenofovir/emtricitabine
NOTE: Entecavir is not an optimal option because of the increased risk of entecavir resistance
(1) Add lamivudine or entecavir
Adefovir (2) Switch to tenofovir/emtricitabine
(3) Switch to entecavir
(1) Switch to tenofovir
Entecavir
(2) Switch to tenofovir/emtricitabine
(1) Add adefovir or tenofovir
Telbivudine (2) Switch to tenofovir/emtricitabine
NOTE: Entecavir is not optimal because of the increased risk of entecavir resistance
AASLD = American Association for the Study of Liver Diseases.

4. Prevention (MMWR 2013;62(RR10):1-19; MMWR 2008;57(RR08):1-20; MMWR 2006;55(RR16):1-33)


a. Screening
i. Prompt identification of chronic HBV allows the following:
(a) Therapy initiation to prevent or delay progression of liver disease
(b) Primary prevention of ongoing HBV transmission
ii. All individuals who are likely to have active HBV infection should be tested for chronic
HBV infection.
iii. Testing for chronic HBV infection meets public health screening criteria (i.e., screening as a
health tool allows identification of unrecognized health conditions so that treatment can be
offered early).
iv. Screening should include HBsAg; screening should be performed in the following HBV high-
risk populations:
(a) Born in regions of intermediate or high endemicity (HBsAg prevalence of 2% or greater)
(b) Born in the United States, not vaccinated as an infant, parents were born in highly
endemic regions (8% or greater)
(c) Injection drug users (past or present)
(d) Men having sex with men
(e) Immunocompromised, including chemotherapy and immunosuppression related to organ
transplantation, rheumatologic, or gastroenterologic disorders
(f) Individuals with liver disease (elevated ALT/AST values) of unknown etiology
(g) Donors of blood, plasma, organs, tissues, or semen
(h) Patients undergoing hemodialysis
(i) Individuals infected with HCV or HIV
(j) All pregnant women
(k) Infants born to HBsAg-positive mothers
(l) Family or household members, needle-sharing, or sexual contacts of HBsAg-positive
individuals
(m) Individuals who are the source of blood or body fluid exposures that might require
postexposure prophylaxis (e.g., needlestick, sexual assault)
(n) Inmates of correctional facilities
(o) Individuals with multiple sexual partners or history of sexually transmitted disease

ACCP Updates in Therapeutics® 2016: Ambulatory Care Pharmacy Preparatory Review and Recertification Course

1-572
Gastrointestinal Disorders

b. Testing in those with a vaccination history:


i. HBsAg testing is recommended in the following populations:
(a) Individuals born in regions of intermediate or high HBsAg prevalence (2% or more)
(b) Born in the United States, not vaccinated at birth, whose parents were born in highly
endemic regions (8% or more)
(c) Individuals who received vaccination as adolescents or adults after the initiation of risk
behaviors (e.g., sexual activity, injection drug use)
ii. Recommended regardless of vaccine history because some may have been infected with HBV
before being vaccinated
c. Immunization
i. Vaccination is the most effective measure to prevent the spread of HBV disease.
Immunization should be offered to all screened individuals testing seronegatively.
ii. Initial vaccines were issued in the United States in 1982; subsequently, a comprehensive
vaccination strategy was developed, which includes:
(a) Universal vaccination of infants at birth
(b) Routine screening of pregnant women to prevent perinatal infection and provide
postexposure immunoprophylaxis in infants born to HBV-positive mothers
(c) Vaccination of all children and adolescents not vaccinated previously
(d) Vaccination of adults not previously vaccinated who are at high risk of infection
iii. Universal vaccination of adults
(a) Is recommended in the following settings:
(1) Sexually transmitted disease/HIV testing and treatment facilities
(2) Drug abuse treatment and prevention facilities
(3) Health care settings targeting services to injection drug abusers and men having
sex with men
(4) Correctional facilities
(b) Vaccinations should be administered to these groups even when testing is not feasible.
iv. HBV vaccines are indicated in the following adult populations:
(a) Any individual seeking protection from HBV infection
(b) Sex partners of HBsAg-positive individuals
(c) Individuals with several sex partners
(d) Men having sex with men
(e) Individuals being evaluated for sexually transmitted disease
(f) Current or recent intravenous drug users
(g) Household contacts of HBsAg-positive individuals
(h) Those with chronic liver disease
(i) HIV-positive patients
(j) Recipients of clotting factors
(k) Patients with end-stage renal disease, including predialysis, hemodialysis, peritoneal
dialysis, and home dialysis
(l) Health care providers whose work, training, and volunteer activities involve a risk of
exposure. If health care providers refuse vaccination, they must document their choice by
signing a declination statement.
(m) International travelers to regions highly endemic for HBV (prevalence of 2% or greater)
(n) Residents and staff of developmental disability facilities

ACCP Updates in Therapeutics® 2016: Ambulatory Care Pharmacy Preparatory Review and Recertification Course

1-573
Gastrointestinal Disorders

v. HBV vaccines (See Table 16)


(a) Single-antigen formulations
(1) Recombivax HB: Contains recombinant HBsAg
(2) Engerix-B: Contains recombinant HBsAg
(b) Combination formulations
(1) Twinrix: Contains recombinant HBsAg and inactivated HAV. Indicated for
individuals 18 years and older at risk of both HAV and HBV
(2) Products for infants and children
(A) Pediarix: Contains recombinant HBsAg, diphtheria and tetanus toxoids and
acellular pertussis adsorbed, and inactive polio virus; cannot be used for the
birth dose of the hepatitis B series
(B) Comvax: Production discontinued in early 2015. (Contained recombinant
HBsAg and Hib [Haemophilus influenzae type b] vaccine)
(c) Dosing schedules vary by age; schedule usually includes a three-dose series at 0, 1, and 6
months administered intramuscularly.
(1) Antibody response increases with each dose, reaching more than 90% by the third
dose; thus, completion of the series is necessary to achieve adequate immunity.
(2) Serologic testing for immunity. Recommended only for those whose subsequent
clinical management depends on knowing their status, including the following:
(A) Health care workers and public safety workers at high risk of continued exposure
(B) Individuals receiving chronic hemodialysis, individuals with HIV infection, or
individuals otherwise immunosuppressed
(C) Infants born to HBsAg-positive mothers and sex partners of HBsAg-
positive individuals
(3) Testing for anti-HBs concentrations should be performed 1–2 months after the last
dose of the vaccine series, but not before 9 months of age.
(A) Anti-HBs concentrations of 10 mIU/mL or greater are considered to
confer immunity.
(B) Those with anti-HBs concentrations less than 10 mIU/mL should be
revaccinated with a second three-dose series.

Table 16. Available HBV Vaccines


Infants Children Adolescents Adults Special Populationsa
< 1 yr 1–10 yr 11–15 yr 11–19 yr ≥ 20 yr < 20 yr > 20 yr
Recombivax HB 5 mcg 5 mcg 10 mcgd 5 mcg 10 mcg 5 mcg 40 mcgb
(Merck & Co.) (0.5 mL) (0.5 mL) (1.0 mL) (0.5 mL) (1.0 mL) (0.5 mL) (1.0 mL)
Engerix-B 10 mcg 10 mcg N/A 10 mcg 20 mcg 10 mcg 40 mcgc
(GlaxoSmithKline) (0.5 mL) (0.5 mL) (0.5 mL) (1.0 mL) (0.5 mL) (2.0 mL)
Twinrix N/A 20 mcg
N/A N/A N/A N/A N/A
(GlaxoSmithKline) (1.0 mL)
Special populations include adults who are immunocompromised or receiving hemodialysis.
a

b
Dialysis formulation administered on a three-dose schedule at 0, 1, and 6 mo.
Two 1.0 mL doses administered at one site, on a four-dose schedule at 0, 1, 2 and 6 mo.
c

d
Adult formulation administered on a two-dose schedule separated by 4–6 mo; must be completed by 16th birthday.
HB = hepatitis B; N/A = not applicable.

ACCP Updates in Therapeutics® 2016: Ambulatory Care Pharmacy Preparatory Review and Recertification Course

1-574
Gastrointestinal Disorders

d. Postexposure prophylaxis (MMWR 2006;55(RR16):1-40)


i. Administer HBV vaccine and/or hepatitis B immunoglobulin (HBIG).
ii. HBIG standard dose is 0.06 mL/kg intramuscularly for adults and children 12 months and
older and 0.5 mL for infants younger than 12 months (when administering with HBV vaccine,
use different injection site).
iii. Effectiveness of postexposure prophylaxis is determined by how quickly the vaccine is
administered, within 24 hours being ideal. Recommended within 7 days for needlestick
exposure and within 14 days for sexual exposures
iv. Indicated in the following populations:
(a) Perinatal exposure – Born to HBsAg-positive mothers; administer HBV vaccine and HBIG
within 12–24 hours after birth, followed by completion of HBV vaccine three-dose series
(b) Non-occupational exposure
(1) Exposure source with HBsAg-positive status
(A) Individuals who have completed vaccine series should receive booster dose (unless
anti-HBs concentration was measured post-vaccination and showed immunity).
(B) Individuals who are in the process of completing vaccine series should receive
HBIG and complete vaccine series.
(C) Individuals who are unvaccinated should receive HBIG and begin vaccine series
within 24 hours postexposure.
(2) Exposure source with unknown HBsAg status
(A) For individuals who have completed vaccine series, no further treatment is
required; however, some providers may offer a booster.
(B) Individuals who are in the process of completing vaccine series should continue.
(C) Individuals who are unvaccinated should begin vaccine series within 24 hours of
exposure or as soon as possible.
(c) Percutaneous or permucosal occupational exposure (e.g., needlestick, bite, laceration)
– Management is the same as for non-occupational exposure except that previously
vaccinated individuals with an unknown antibody response are tested for anti-HBs; if
anti-HBs is adequate, no treatment is needed, and if inadequate, HBIG and a vaccine
booster are given unless the source is proven to be HBsAg negative.
5. Role of the pharmacist (Domain 1, Task 5; Domain 2, Task 1,5; Domain 5, Task 2)
a. Medication adherence is necessary to achieve positive outcomes and is important in preventing drug
resistance with NA therapy for the treatment of chronic HBV. Pharmacists are in position to directly
intervene and counsel on adherence and issues with drug resistance in this patient population.
b. An article published in the Journal of Hepatology (J Hepatol 2011;54:12-8) on patient persistence
and adherence to NA therapy for treatment of chronic HBV reports that in 11,100 patients, the
mean adherence rate was 87.8% plus or minus 19.1%. The study found that the adherence rates
were higher among patients already receiving therapy than among patients new to therapy. In
addition, young adults were less likely to achieve adherence rates greater than 90%. This study
shows suboptimal adherence rates in patients receiving antiviral NA therapy for chronic HBV
infection, with opportunities for pharmacist involvement to counsel patients, especially young
adults and those new to therapy.
6. Patient education resources
a. www.cdc.gov/hepatitis/HBV/PatientEduHBV.htm
b. Medication assistance programs are often available for patients meeting defined income criteria.
See individual drug company websites for more information.

ACCP Updates in Therapeutics® 2016: Ambulatory Care Pharmacy Preparatory Review and Recertification Course

1-575
Gastrointestinal Disorders

Patient Case

7. A 57-year-old woman with a history of intravenous drug and alcohol abuse was given a diagnosis of chronic HBV
6 months ago. Vital signs at the visit were as follows: height 67 inches, weight 61 kg, temperature 98.7°F, heart rate
79 beats/minute, respiratory rate 15 breaths/minute, and blood pressure 130/80 mm Hg. She had laboratory tests
1 week ago, which revealed the following: HBsAg and HBeAg positivity, AST 478 IU/mL, ALT 780 IU/mL,
albumin 3.3 g/dL, INR 1.1, SCr 1.3 mg/dL, and HBV DNA 94,000 IU/mL. A liver biopsy performed after she
received the initial diagnosis revealed significant fibrosis (stage 3). Her medical history is significant for depression,
for which she takes citalopram 20 mg once daily. She reports taking no other medications or OTC products. Which
is the best option for this patient?
A. Initiate pegylated interferon 180 mcg subcutaneously once weekly.
B. Initiate lamivudine 100 mg orally for the first dose; then 50 mg orally daily.
C. Initiate tenofovir 300 mg orally once daily.
D. Initiate tenofovir 300 mg orally every other day.

D. Hepatitis C
1. Background
a. Most common bloodborne infection in the United States; principal cause of death from liver disease
and leading indication for liver transplantation. About 180 million people infected worldwide and
4–5 million chronically infected in the United States (www.cdc.gov/hepatitis/hcv/cfaq.htm)
b. HCV is a single-stranded RNA virus; seven genotypes classified (six have treatment
recommendations) and 67 subtypes (Smith DB, et al. Hepatology 2014)
i. In the United States, genotype 1 is most common (subtypes 1a and 1b), followed by
genotypes 2 and 3.
ii. Other genotypes are more commonly found in other parts of the world (genotype 4 [Egypt],
genotype 5 [South Africa], and genotype 6 [Asia]) and are less common in the United States.
c. Natural history in Figure 2

Resolution
15%–25%
Acute infection
75%–85%
Chronic infection Mild, moderate, severe End-stage liver disease, transplant
20%
3%–6%/yr r
Cirrhosis Decompensation

1%–4%/yr
Hepatocellular carcinoma

0.5 10 20
Approximate time (years)

Figure 2. Progression of hepatitis C virus.


Information from: McHutchison JG, Bacon BR. Chronic hepatitis C: an age wave of disease burden. Am J Manag Care 2005;11:S286-S295.

ACCP Updates in Therapeutics® 2016: Ambulatory Care Pharmacy Preparatory Review and Recertification Course

1-576
Gastrointestinal Disorders

2. Clinical presentation
a. Acute: Usually asymptomatic; if symptoms do appear, they are generally nonspecific and include
fatigue, weakness, anorexia, and jaundice; typically appear within 4–12 hours after exposure. Rapid
progression to fulminant liver disease is infrequent. Diagnosis of acute infection is uncommon.
b. Chronic: Most common symptom is fatigue. Most who present for medical attention have
chronic infection.
i. Most remain asymptomatic for years, presenting with symptoms after decades of having
the infection.
ii. Some patients may present for medical care with anorexia, abdominal pain, fever, jaundice,
malaise, nausea, or symptoms associated with hepatocellular carcinoma and liver cirrhosis.
Extrahepatic disease (e.g., cryoglobulinemia, glomerulonephritis) may also be present.
iii. Table 17 lists factors associated with accelerated disease progression (i.e., fibrosis).

Table 17. Factors Associated with Accelerated Hepatitis C Virus Disease Progression
Fibrosis stage
Inflammation grade
Nonmodifiable (host) Older age at time of infection
Male sex
Organ transplantation
Genotype 3
Nonmodifiable (viral)
Coinfection with HIV or HBV
Alcohol consumption
Nonalcoholic fatty liver disease
Modifiable (host)
Obesity
Insulin resistance
Adapted from: AASLD/IDSA Recommendations for Testing, Managing, and Treating Hepatitis C Guidance Document. Available at www.hcvguidelines.org.
Accessed October 15, 2015.

3. Screening:
a. The Centers for Disease Control and Prevention (CDC) recommends screening of high-risk individuals
and patients with identifiable risk factors and clinical symptoms (MMWR 2013;62:362-5).
i. Screen individuals born 1945–1965 without respect to risk factors because of the high
prevalence (1 in 30) of HCV infection within this population.
ii. Other individuals should be screened on the basis of risk factors for HCV infection.
b. Risk factors and modes of transmission
i. Risk behaviors
(a) Injection drug use currently or ever (intravenous drug use is the most common mode of
HCV transmission in the United States)
(b) Intranasal illicit drug use
ii. Risk exposures
(a) Long-term hemodialysis
(b) Tattoo in an unregulated setting
(c) Needlesticks, sharps, or mucosal exposure to HCV-infected blood for health care,
emergency medical, and public safety workers
(d) Children born to HCV-infected mothers

ACCP Updates in Therapeutics® 2016: Ambulatory Care Pharmacy Preparatory Review and Recertification Course

1-577
Gastrointestinal Disorders

(e) Prior recipients of blood transfusions or organ transplants in the following situations:
(1) Were notified that the donor later tested positive for HCV infection
(2) Received a transfusion or transplant before 1992
(3) Received clotting factor concentrates produced before 1987
(f) Incarceration
iii. Other medical conditions
(a) HIV infection
(b) Unexplained chronic liver disease and chronic hepatitis with elevated ALT concentrations
(c) Solid organ donors
c. Laboratory testing
i. Antibody to HCV (anti-HCV)
(a) Used to screen for HCV exposure. Anti-HCV will not differentiate between acute,
chronic, and resolved infection.
(b) Positive anti-HCV should be confirmed by HCV RNA; a positive viral load will confirm
current infection.
(c) Testing should be done with an FDA-approved test including laboratory-based assays and
point-of-care assay (i.e., OraQuick HCV Rapid Antibody Test).
ii. HCV nucleic acid test: HCV RNA. Quantitative:
(a) Reverse transcriptase–PCR is the typical methodology used for viral load quantification;
has improved sensitivity, with a lower limit of detection of 5–50 IU/mL
(b) Used to detect and/or quantify HCV RNA in blood to confirm current (active) viral
nucleic acid in the following individuals:
(1) Positive anti-HCV result
(2) Negative anti-HCV result and thought to have liver disease
(3) Negative anti-HCV result and who might have been exposed to HCV within the
past 6 months
(4) Those who are immunocompromised and thought to have HCV
(c) Does not evaluate disease severity or prognosis; assists with identifying treatment response
(d) All assays are 98%–99% specific; international reporting standard for HCV RNA is in
international units per milliliter (rather than copies per milliliter).
iii. HCV genotype assays: Several methodologies available
(a) Currently, genotyping is necessary for determining appropriate treatment options.
(b) Incorrect genotype is rare.
(c) Mixed genotypes occur but are uncommon.
4. Diagnosis (Domain 1, Task 1; Domain 4, Task 3)
a. Discriminating acute from chronic disease requires evaluating clinical presentation, laboratory
results, and non-invasive markers of disease progression or biopsy findings (if available).
b. Evaluation of laboratory tests should be done with caution.
i. HCV RNA is present in the serum before anti-HCV appears after acute exposure.
ii. HCV RNA can be detected 1–2 weeks postexposure; anti-HCV can be detected
8–12 weeks postexposure.
c. First rapid blood test for antibodies to HCV (OraQuick HCV Rapid Antibody Test) was approved
by the FDA in June 2010.
i. Approved for screening individuals older than 15 years who are considered at risk of infection
ii. Test is run from a venous blood sample, with results in about 20 minutes.
iii. Caution patients and providers that this tests for HCV exposure only; diagnosis of current
HCV exposure requires confirmation with HCV RNA test.

ACCP Updates in Therapeutics® 2016: Ambulatory Care Pharmacy Preparatory Review and Recertification Course

1-578
Gastrointestinal Disorders

d. Noninvasive methods to estimate liver disease severity


i. Liver-directed physical examination
ii. Routine blood tests (i.e., liver panel, CBC)
iii. Liver imaging (i.e., ultrasonography, CT scan, magnetic resonance imaging [MRI])
iv. Serum biomarkers (e.g., FibroSURE); laboratory blood tests; equation uses markers to
estimate level of scarring and inflammation
v. Liver transient elastography (e.g., Fibroscan). Measures liver stiffness and correlates well with
measurement of substantial fibrosis. Costly, portable machine; procedure can be done in clinic
or at bedside
e. Invasive – Liver biopsy
i. Provides objective information about the amount and pattern of collagen or scar tissue in the
liver; also used to help assess the severity of liver inflammation and hepatic steatosis and to
help exclude competing causes of liver injury
ii. Other biopsy findings may indicate the need for additional interventions (e.g., drug toxicity,
steatosis, hepatocellular carcinoma).
iii. The Metavir and Ishak fibrosis scores are commonly used to quantify the amount of hepatic
collagen and help with decisions regarding the initiation of HCV antiviral therapy.
(a) Metavir scoring system consists of a grade and a stage; preferred scoring system
(1) The grade or the degree of inflammation is assigned a number from 0 (no activity) to
4 (severe activity).
(2) The stage or the amount of fibrosis or scarring is assigned a number from 0 to 4.
Definitions of each stage are as follows:
0 = no scarring
1 = minimal scarring
2 = scarring occurs and extends outside areas containing blood vessels
3 = bridging fibrosis, connecting other fibrotic areas
4 = advanced scarring or cirrhosis
(b) Ishak modified hepatic activity index
(1) Used for scoring necroinflammatory activity in chronic hepatitis
(2) Degree of inflammation is assigned a number from 0 (no fibrosis) to 6 (fibrosis).
iv. Biopsy was previously considered the gold standard to evaluate disease severity; drawbacks
are risk related to an invasive procedure (e.g., pain, bleeding, possibility of other poor
outcomes), sampling error, and costs. Non-invasive markers of fibrosis (e.g., transient
elastography and laboratory tests) may be preferred or standard with certain patient
populations and providers, depending on patient factors.
f. Stage of disease is used in combination with the clinical scenario to determine when antiviral
therapy should be requested.
5. Treatment – Acute infection
a. Treatment during an acute infection should be delayed at least 12–16 weeks to allow for
spontaneous clearance before therapy initiation. HCV RNA level should be monitored every
4–8 weeks during this time.
b. After the 12- to 16-week waiting period, the practitioner and patient must decide whether to
initiate treatment currently, depending on the clinical situation. Some may start therapy, and
others may decide to wait longer (i.e., 6–12 months) to allow for spontaneous clearance.
c. If HCV infection spontaneously clears, treatment is not recommended.
d. If HCV infection persists, the current guidelines recommend the following treatment:
i. Because of high safety and efficacy, the same regimens are recommended as for chronic
disease (see next section for details).
ii. HCV guidelines are updated often and available at www.hcvguidelines.org.

ACCP Updates in Therapeutics® 2016: Ambulatory Care Pharmacy Preparatory Review and Recertification Course

1-579
Gastrointestinal Disorders

6. Treatment – Chronic infection (Domain 1, Task 3,4,6,7)


a. In addition to therapy, HCV-infected individuals should receive education about how to prevent
further liver damage and prevent disease transmission (see Box 2).

Box 2. Education for Hepatitis C Virus–Infected Individuals to Prevent HCV Transmission


• Avoid sharing toothbrushes and dental or shaving equipment
• Cover any bleeding wound to prevent the possibility of others coming into contact with infected blood
• Counsel to avoid using illicit drugs and enter substance abuse treatment
• Counsel to avoid reusing or sharing syringes, needles, or any supplies for those continuing to use injectable drugs
• Avoid donating blood
• In those coinfected with HIV, use barrier precautions to prevent sexual transmission
• Always wear gloves when cleaning up blood spills
• Proper cleaning of contaminated surfaces with a dilution of 1 part household bleach to 9 parts water
Adapted from: AASLD/IDSA Recommendations for Testing, Managing, and Treating Hepatitis C Guidance Document.
Available at www.hcvguidelines.org. Accessed October 15, 2015.

b. Therapy goals are to achieve a sustained virologic response (SVR; defined as the absence of
detectable HCV RNA at least 12 weeks after completion of therapy) and to reduce all-cause
mortality and liver-related health adverse consequences, including end-stage liver disease, the need
for liver transplantation, and hepatocellular carcinoma.
c. Treatment response
i. SVR is a marker for cure of HCV infection. Patients achieving SVR have HCV antibodies but
no longer have detectable HCV RNA.
(a) Documentation of SVR requires the use of an FDA-approved quantitative or qualitative
NAT (HCV RNA) with a detection level of 25 IU/mL or lower.
(b) Individuals achieving SVR have several health benefits, including a decrease in liver
inflammation, as reflected by improved aminotransferase concentration and rate of
progression of liver fibrosis. SVR is associated with more than 70% reduction in liver
cancer risk and 90% reduction in risk of liver-related mortality and liver transplantation
(Ann Intern Med 2013;158:329-37).
ii. Relapse can occur after the completion of HCV treatment, defined by a detectable viral load
after the completion of treatment after having an undetectable level while receiving treatment
and at the end of treatment. Previous treatment regimens (before 2013) used additional
terminology to describe treatment response (e.g., partial responder).
d. Measured outcomes
i. Biochemical (normalization of ALT concentration)
ii. Virologic (undetectable viral load)
iii. Histologic (improved inflammatory score without worsening of fibrosis)
iv. Additional long-term outcomes are often studied (e.g., liver disease complications,
hepatocellular carcinoma, and death); however, these are difficult to evaluate in clinical trials
because long-term follow-up is lacking.
e. HCV treatment guidelines (AASLD/IDSA [Infectious Diseases Society of America])

ACCP Updates in Therapeutics® 2016: Ambulatory Care Pharmacy Preparatory Review and Recertification Course

1-580
Gastrointestinal Disorders

i. The landscape of treatment for HCV infection has evolved drastically since 2011 with
the introduction of HCV PIs, the first direct-acting antivirals (DAAs) for HCV treatment.
Additional changes are expected as several medications with different mechanisms of
actions become available. The AASLD/IDSA guidance document is a “living document”
that is updated often as new information and treatments become available. Therefore, it is
recommended that readers access the guidance document on the website (www.hcvguidelines.
org) for the latest, most up-to-date recommendations.
ii. Guideline recommendations are based on scientific evidence and expert opinion. Each
recommendation is rated on the level of evidence (i.e., Roman numeral) and strength of the
recommendation (i.e., letter). See the website for additional details regarding the rating system.
iii. To assist with making the best treatment decision for each patient, each recommendation is
classified as follows:
(a) Recommended – Favored for most patients
(b) Alternative – Optimal in a particular subset of patients
(c) Not recommended – Inferior or harmful; should not be administered
NOTE: If several regimens are offered at the same recommendation level, they are listed
in alphabetical order. Selection of which regimen to initiate should be determined from
patient-specific information.
iv. General considerations
(a) Providers should assess for comorbidities – Ongoing monitoring while receiving
therapy for medication-related adverse events and drug-drug interactions is necessary.
A clinically useful, reliable, comprehensive, up-to-date, evidence-based drug-drug
interaction resource is freely available to health care workers, patients, and researchers
(http://www.hep-druginteractions.org/).
(b) Treatment regimens and length vary according to HCV genotype, previous treatment
history, and presence or absence of cirrhosis. Renal impairment and posttransplant status
will further affect treatment options.
(c) The most immediate and high-impact benefits of SVR will be realized in populations that
are at the highest risk of liver-related complications caused by liver disease progression.
Guidelines recommend treating all patients with HCV, except those with short life
expectancies that cannot be remediated by treatment. In a setting of reduced resources,
insurance providers often limit coverage to certain groups of patients. Earlier versions of
the guidelines listed the following groups..
(1) Highest treatment priority – Because of the highest risk of severe complications
(A) Advanced fibrosis (Metavir F3) or compensated cirrhosis (Metavir F4)
(B) Organ transplant recipients
(C) Those with clinically severe extrahepatic manifestations such as those with type
2 or type 3 essential mixed cryoglobulinemia with end-organ manifestations
(e.g., vasculitis), proteinuria, nephrotic syndrome, or membranoproliferative
glomerulonephritis
(2) High treatment priority – Because of high risk of complications
(A) Fibrosis (Metavir F2)
(B) HIV-1 coinfection
(C) HBV coinfection
(D) Other coexistent liver disease (e.g., NASH [nonalcoholic steatohepatitis])
(E) Debilitating fatigue
(F) Diabetes mellitus (insulin resistant)
(G) Porphyria cutanea tarda

ACCP Updates in Therapeutics® 2016: Ambulatory Care Pharmacy Preparatory Review and Recertification Course

1-581
Gastrointestinal Disorders

(3) Other populations for which achieving SVR would likely decrease the risk of
further disease transmission, potentially yielding long-term benefits from decreased
transmission and disease prevalence
(A) Men who have sex with men with high-risk sexual practices
(B) Active injection drug users
(C) Incarcerated individuals
(D) Individuals receiving long-term hemodialysis
(E) HCV-infected women of childbearing potential wishing to get pregnant
(F) HCV-infected health care workers who perform exposure-prone procedures
(4) Global treatment concerns
(A) Monotherapy with pegylated interferon, ribavirin, or a DAA is never
recommended in HCV treatment; resistance to a single DAA develops rapidly.
(B) Telaprevir- or boceprevir-based regimens are no longer used in the United States
because of poor efficacy compared with newer DAAs and lack of availability.
v. Guideline categories for HCV treatment
(a) Initial treatment includes treatment-naive patients, those who have not been previously treated
with any HCV antiviral agents (i.e., interferon, pegylated interferon, or any HCV DAA)
(1) Initial treatment options by genotype as of February 29, 2016, are listed in Table 18.
Additional agents will be included in the guidelines later in 2016.
(2) The length of therapy and addition of ribavirin depend on the presence of cirrhosis.
HCV treatment guidelines provide a summary of clinical data supporting these
recommendations (www.hcvguidelines.org).
(b) Retreatment includes patients whose prior therapy has failed. Recommendations vary
according to type of previous therapy. See guidelines for details.

Table 18. Regimens for Initial HCV Treatment (as of 2/29/2016*)


HCV Treatment Options
NOT Recommended
Genotype Recommended Alternative
• DCV + SOF ± RBV for 12–24 wk
• EBR/GRZ +/- RBV for 12–16 wk • SOF + RBV for 24 wk
depending on baseline NS5A RAVs • PEG-IFN + RBV ± SOF, SMV, TVR,
1a • LDV/SOF for 12 wk None BOC for 12–48 wk
• PTV/r/OBV + DAS + RBV for • Monotherapy with PEG-IFN, RBV, or
12–24 wk a DAA
• SMV + SOF ± RBV for 12–24 wk
• DCV + SOF ± RBV for 12–24 wk • SOF + RBV for 24 wk
• EBR/GRZ for 12 wk • PEG-IFN + RBV ± SOF, SMV, TVR,
1b • LDV/SOF for 12 wk None BOC for 12–48 wk
PTV/r/OBV + DAS for 12–24 wk • Monotherapy with PEG-IFN, RBV, or
• SMV + SOF ± RBV for 12–24 wk a DAA
• PEG-IFN + RBV for 24 wk
• TVR-, BOC-, or LDV-containing
• DCV + SOF for 12–24 wk
2 None regimens
• SOF + RBV for 16–24 wk
• Monotherapy with PEG-IFN, RBV, or
a DAA

ACCP Updates in Therapeutics® 2016: Ambulatory Care Pharmacy Preparatory Review and Recertification Course

1-582
Gastrointestinal Disorders

Table 18. Regimens for Initial HCV Treatment (as of 2/29/2016*) (continued)
• PEG-IFN + RBV for 24–48 wk
• DCV + SOF ± RBV for 12–24 wk • SOF + RBV for • TVR-, BOC-, or SMV-based regimens
3
• SOF + PEG-IFN + RBV for 12 wk 24 wk • Monotherapy with PEG-IFN, RBV, or
a DAA
• EBR/GRZ for 12 wk • PEG-IFN + RBV ± SMV for 24–48 wk
• LDV/SOF for 12 wk • SOF + PEG-IFN • TVR- or BOC-containing regimens
4
• PTV/r/OBV + RBV for 12 wk + RBV for 12 wk • Monotherapy with PEG-IFN, RBV, or
SOF (24) + RBV for 24 wk a DAA
HCV Treatment Options
NOT Recommended
Genotype Recommended Alternative
• PEG-IFN + RBV ± SMV for 24–48 wk
• SOF + PEG-IFN • TVR- or BOC-containing regimens
5 or 6 • LDV/SOF for 12 wk
+ RBV for 12 wk • Monotherapy with PEG-IFN, RBV, or
a DAA
BOC = boceprevir; DAA = direct-acting antiviral; DAS = dasabuvir; DCV = daclatasvir; LDV = ledipasvir; OBV = ombitasvir; PTV = paritaprevir; PEG-IFN
= pegylated interferon; RBV = ribavirin; RAVs = resistance associated variants; SMV = simeprevir; SOF = sofosbuvir; TVR = telaprevir.
Adapted from: AASLD/IDSA Recommendations for Testing, Managing, and Treating Hepatitis C Guidance Document. Available at www.hcvguidelines.org.
Accessed February 29, 2016.

vi. Specific patient populations including the following:


(a) Decompensated cirrhosis – Includes those with moderate to severe hepatic impairment
(CTP class B or C)
(b) HIV/HCV coinfection
(1) HIV/HCV-coinfected individuals should be treated for HCV the same as individuals
without HIV infection; however, caution must be taken to recognize and manage
drug interactions with antiretroviral medications.
(2) General considerations
(A) Potential exists for significant complex drug-drug interactions that can occur
between HCV therapies and antiretroviral medications. If needed, any antiviral
drug changes should be done in collaboration with the HIV practitioner.
(B) Antiretroviral treatment interruption to allow HCV therapy is not recommended.
(C) See the HCV treatment guidelines for additional information regarding
pharmacokinetics and drug interactions.
(3) The following combinations are NOT recommended:
(A) Ledipasvir/sofosbuvir with cobicistat and tenofovir disoproxil fumarate
(B) Ledipasvir/sofosbuvir with tipranavir
(C) Sofosbuvir with tipranavir
(D) Paritaprevir/ritonavir/ombitasvir plus dasabuvir with darunavir, efavirenz,
rilpivirine, or ritonavir-boosted lopinavir
(E) Paritaprevir/ritonavir/ombitasvir with or without dasabuvir in patients not
receiving antiretroviral therapy
(F) Simeprevir with cobicistat, efavirenz, etravirine, nevirapine, or any HIV PIs
(G) Ribavirin with didanosine, stavudine, or zidovudine
(H) Pegylated interferon with or without simeprevir, telaprevir, or boceprevir
for 24–48 weeks
(I) HCV DAA treatment shorter than 12 weeks
(J) Monotherapy with pegylated interferon, ribavirin, or a DAA

ACCP Updates in Therapeutics® 2016: Ambulatory Care Pharmacy Preparatory Review and Recertification Course

1-583
Gastrointestinal Disorders

(K) Pegylated interferon plus ribavirin with or without simeprevir, telaprevir,


boceprevir for 24–48 weeks
(c) Recurrent infection post-liver transplantation
(1) Because of the potential for drug-drug interactions with PIs and transplant
immunosuppressant medications (e.g., calcineurin inhibitors), careful assessment
before treatment and monitoring during treatment is necessary. Clinicians must be
aware of dose adjustments required for immunosuppressants.
(2) The following regimens are not recommended for treatment-naive individuals with
compensated allograft HCV infection:
(A) Regimens containing pegylated interferon
(B) Monotherapy with pegylated interferon, ribavirin, or a DAA
(3) The following regimens are not recommended for individuals with decompensated
allograft HCV infection:
(A) Regimens containing pegylated interferon
(B) Regimens containing simeprevir
(C) Regimens containing fixed-dose paritaprevir, ritonavir, ombitasvir plus
dasabuvir plus ribavirin
(D) Monotherapy with pegylated interferon, ribavirin, or a DAA
(E) Telaprevir- or boceprevir-based regimens
(4) HCV treatment guidelines provide a summary of clinical trial data supporting these
recommendations (www.hcvguidelines.org).
(d) Renal impairment (Table 19). Only elbasvir/grazoprevir is FDA-approved for use in
patients on dialysis.

Table 19. Summary of Recommendationsa: HCV Treatment – Dose Adjustments for Renal Impairment
eGFR/
Renal CrCl, mL/ EBR/ PTV/r/ PEG-
LDV DAS DCV SOF SMV RBV
Impairment minute/ GRZ OBV IFN-2a
1.73 m2
180
Mild 50−80 Standard Standard Standard Standard Standard Standard Standard Standard
mcg/wk
Alternate
doses of
200
mg/day 180
Moderate 30−50 Standard Standard Standard Standard Standard Standard Standard
and 400 mcg/wk
mg/day
every
other day
Data not Limited Limited Limited Limited 200 mg/ 135
Severe < 30 Standard Standard
available data data data data day mcg/wk

End-stage renal disease Data not Limited Limited Limited Limited Limited 200 mg/ 135
Standard
or hemodialysis available data data data data data day mcg/wk
a
Adapted from: AASLD/IDSA Recommendations for Testing, Managing, and Treating Hepatitis C Guidance Document. Available at www.hcvguidelines.
org. Accessed February 8, 2016.
AASLD/IDSA = American Association for the Study of Liver Diseases/Infectious Diseases Society of America; DAS = dasabuvir; DCV = daclatasvir;
EBR/GRZ = elbasvir / grazoprevir; eGFR/CrCl = estimated glomerular filtration rate/creatinine clearance; LDV = ledipasvir; OBV = ombitasvir; PTV =
paritaprevir; PEG-IFN = pegylated interferon; RBV = ribavirin; r = ritonavir; SMV = simeprevir; SOF = sofosbuvir.

ACCP Updates in Therapeutics® 2016: Ambulatory Care Pharmacy Preparatory Review and Recertification Course

1-584
Gastrointestinal Disorders

7. Monitoring
a. This section provides a summary of monitoring recommendations before, during, and after
HCV therapy. For additional details, see HCV treatment guidelines (www.hcvguidelines.org).
Furthermore, additional monitoring may be required, based on the potential for drug-related
adverse events and drug interactions.
b. Recommended monitoring: Before initiating HCV therapy
i. Assess the potential for drug-drug interactions.
ii. The following laboratory tests are recommended within 12 weeks of therapy initiation: CBC,
INR, hepatic function panel, TSH (if regimen contains pegylated interferon), calculated GFR
iii. The following laboratory tests are recommended before therapy initiation: HCV genotype and
subtype and quantitative HCV viral load; also, if quantification will influence therapy duration
(or insurance coverage), may need to obtain laboratory measurement during therapy
c. Recommended monitoring: During HCV therapy
i. Assessment of medication adherence, monitoring of adverse effects, and potential for drug-drug
interactions should occur through clinic visits or telephone contact as clinically indicated.
ii. The following laboratory tests are recommended within 4 weeks of initiation therapy: CBC,
creatinine concentration, calculated GFR, hepatic function panel. Consider increasing the
frequency if the regimen contains medications with an increased likelihood for drug-related
toxicities such as ribavirin (e.g., may need to obtain CBC more often).
iii. If regimen contains pegylated interferon: TSH every 12 weeks
iv. HCV quantitative viral load after 4 weeks of therapy and 12 weeks after therapy is completed.
Some may also obtain at the end of treatment and 24 weeks after therapy is completed.
d. Recommended monitoring: For discontinuation of therapy that is not effective
i. Monitor HCV quantitative viral load
ii. If HCV quantitative viral load is detectable at treatment week 4, repeat test after an additional
2 weeks (i.e., treatment week 6). If treatment week 6 viral load has increased by greater
than 10-fold (greater than 1 log IU/mL), it is recommended to discontinue therapy. No
recommendations are available if viral load is decreased at week 6 or 8.
e. Recommended monitoring: Pregnancy-related issues while receiving ribavirin
i. Women of childbearing potential should have serum pregnancy test before initiation of
therapy if regimen contains ribavirin.
ii. Contraception use and possible pregnancy should be assessed during therapy at appropriate
intervals and for 6 months after the completion of treatment for women of childbearing
potential and for female partners of men who receive ribavirin.
f. Recommended monitoring: For those without achievement of SVR
i. The following laboratory tests are recommended every 6–12 months to assess disease
progression: Hepatic function panel, CBC, and INR
ii. For individuals with advanced fibrosis (i.e., Metavir stage F3 or F4): Ultrasound testing every
6 months for surveillance of hepatocellular cancer
iii. For individuals with cirrhosis: Endoscopic surveillance for esophageal varices
iv. As new HCV therapies become available, evaluation for retreatment
g. Recommended monitoring: For those with achievement of SVR
i. For individuals without advanced fibrosis (i.e., Metavir stage 0–2), no additional follow-up is
necessary. Follow up as if they were never infected with HCV.
ii. For individuals with ongoing risk of infection or if hepatic dysfunction develops, obtain
quantitative HCV viral load to assess for HCV recurrence.
iii. For individuals with advanced fibrosis (i.e., Metavir stage F3 or F4): Ultrasound testing every
6 months for surveillance of hepatocellular cancer

ACCP Updates in Therapeutics® 2016: Ambulatory Care Pharmacy Preparatory Review and Recertification Course

1-585
Gastrointestinal Disorders

iv. For individuals with cirrhosis: Baseline endoscopy to screen for varices. If present, treat and
follow as indicated.
8. Pharmacotherapy (Domain 1, Task 3,4,6,7)
a. Pegylated interferon – Pegylated interferon-α2a (PEGASYS)
i. Dosing is the same for all genotypes. Follow manufacturer’s recommendations for dose
adjustments. Table 19 summarizes recommendations for dose adjustments for renal
insufficiency according to the treatment guidelines.
ii. Efficacy comparable between the two approved agents
iii. Adverse events include fever, chills, headache, fatigue, anorexia, arthralgia, nausea, injection-
site reactions, depression, anxiety, insomnia, and alopecia.
iv. Use with ribavirin was the standard of care before the availability of DAAs in 2011; currently
used only as alternative options in combination with ribavirin and sofosbuvir
b. Ribavirin: Nucleoside analog
i. Inhibits viral replication by viral protein synthesis inhibition
ii. Available formulations
(a) Tablet: 200 mg (Copegus); 200, 400, 600 mg (Ribasphere)
(b) Capsule: 200 mg (Rebetol, Ribasphere)
(c) Dose packs: 400- and 600-mg options (Ribasphere RibaPak, Moderiba)
(d) Oral solution: 40 mg/mL (Rebetol)
NOTE: Many generic products are available.
iii. Dosing
(a) According to the prescribing guidelines, the dosing varies according to HCV genotype
and patient weight. However, the current treatment guidelines specify dosing according to
patient weight (75 kg or more: 1200 mg per day; less than 75 kg: 1000 mg per day), which
reflects the most recent clinical trial data.
(b) See the HCV treatment guidelines for dosing in special populations and dose adjustments.
(1) Recurrent HCV infection post-liver transplantation: Initiate ribavirin at 600 mg per
day and increase dose as tolerated.
(2) Renal impairment: Table 19 summarizes recommendations for dose adjustments for
renal insufficiency according to treatment guidelines.
iv. Adverse events
(a) Hemolytic anemia (black box warning); occurs in 10%–13% of patients; usually within the
first 2 weeks of therapy initiation. May worsen underlying significant or unstable cardiac
disease. Dose reduction or discontinuation may be necessary; follow manufacturer’s
recommendations. Supplemental therapies (e.g., erythropoiesis-stimulating agents) for
management were used more commonly in clinical practice before DAA approval. Should
confirm that iron stores are adequate before and during therapy
(b) Teratogenicity: Pregnancy category X; recommendations according to prescribing
information include the following:
(1) Female patients and female partners of male patients undergoing treatment should
not become pregnant during treatment or for 6 months after treatment.
(2) Two effective forms of birth control must be used during therapy and for 6 months
after therapy.
(3) Serum pregnancy tests must be included in monitoring plan (baseline, monthly on
treatment, and up to 6 months posttreatment).
(4) If the patient or the patient’s partner becomes pregnant while taking ribavirin or
within 6 months of therapy completion, educate patients to notify their health care
providers immediately. The ribavirin pregnancy registry should be contacted at
1-800-593-2214.

ACCP Updates in Therapeutics® 2016: Ambulatory Care Pharmacy Preparatory Review and Recertification Course

1-586
Gastrointestinal Disorders

(c) Anorexia, fatigue, headache, pulmonary dysfunction, insomnia, nausea


(d) Dose adjustments for renal dysfunction are necessary; see Table 19 and the HCV treatment
guidelines. In addition, use is contraindicated when CrCl is less than 50 mL/minute/1.73
m2 according to the Rebetol prescribing guidelines (November 2013), but there are no
contraindications according to CrCl in the Copegus prescribing guidelines (February 2013).
c. PIs
i. Mechanism: Prevent HCV replication by inhibiting the HCV NS3/4A protease enzyme.
ii. First-generation PIs boceprevir (Victrelis) and telaprevir (Incivek) are no longer recommended
therapy in the United States according to the treatment guidelines. These agents were FDA
approved in May 2011 and were the first DAAs approved for HCV. They were more effective
than the combination of pegylated interferon and ribavirin, but they are less effective than the
NS5B and NS5A DAAs and other PIs that were approved in 2013–2015.
iii. Drug interactions: CYP3A drug interactions are common with PIs, producing altered drug
concentrations, which may have significant clinical consequences and/or adverse outcomes
requiring vigilant monitoring and dose adjustments.
iv. Available agents
(a) Simeprevir (Olysio)
(1) FDA indication (approved December 2013): Treatment of chronic HCV genotype 1
infection as a component of a combination antiviral treatment regimen
(A) Monotherapy not recommended
(B) Previously used in combination with pegylated interferon and ribavirin; now
only recommended for use in combination with sofosbuvir with or without
ribavirin
(C) Patients with genotype 1a should be screened for NS3 Q80K polymorphism. If
detected, alternative therapy should be considered, especially in patients with
cirrhosis, because of decreased SVR rates.
(D) Not recommended in individuals previously treated with simeprevir or other
HCV PIs
(2) Dose and administration
(A) Simeprevir 150 mg by mouth once daily with food in combination with other
HCV antiviral drugs
(B) No dose adjustments required for mild or moderate renal impairment. Safety
and efficacy have not been established in severe renal impairment (CrCl less
than 30 mL/minute/1.73 m2) or end-stage renal disease and in dialysis. Highly
protein bound so unlikely to be removed by dialysis
(C) No dose recommendations are provided for patients with moderate or severe
hepatic impairment. Not studied in patients with decompensated cirrhosis (CTP
class B or C)
(3) Formulations: Oral; 150-mg capsule
(4) Treatment duration
(A) Depends on patient population; 12 or 24 weeks with or without ribavirin,
depending on previous treatment experience and presence of cirrhosis
(B) For up-to-date information regarding HCV treatment recommendations, see the
HCV treatment guidelines (www.hcvguidelines.org).
(5) No treatment discontinuation rules apply to the use of simeprevir with sofosbuvir.
(6) Contraindications
(A) Hypersensitivity to simeprevir

ACCP Updates in Therapeutics® 2016: Ambulatory Care Pharmacy Preparatory Review and Recertification Course

1-587
Gastrointestinal Disorders

(B) Coadministration with inducers or inhibitors of CYP3A is not recommended


because this may lead to significant alterations in simeprevir drug exposure.
See prescribing guidelines for details regarding established and potential
drug interactions.
(7) Adverse events
(A) The most notable adverse events include rash, photosensitivity, pruritus, and nausea.
• Photosensitivity reactions occurred most often during the initial 4 weeks of
therapy but may occur at any time during therapy. Recommended to limit sun
exposure and use sun-protective measures when exposed to the sun
• Rash occurred most often during initial 4 weeks of therapy but may occur at
any time during therapy. Recommended to discuss with provider immediately
if severe rash occurs
(B) Simeprevir contains a sulfonamide moiety. In registration trials, those with
sulfa allergy (n=16) did not report any adverse events when taking simeprevir.
Currently, data are insufficient to exclude an association among sulfa allergy and
simeprevir adverse events.
(8) Monitoring: The following should be considered when using combination therapy
that includes simeprevir.
(A) Adverse effects: Clinicians should assess adverse effects at each visit and
provide continual education reminding patients to contact their health care
providers should they have any adverse events, new symptoms, or changes in
their clinical status.
(B) Pharmacists should perform medication reconciliation at each visit to ensure
proper adherence to the HCV treatment regimen as well as the avoidance of
medications recommended not to be administered concomitantly.
(b) Paritaprevir
(1) FDA indications: Approved December 2014 for treatment of HCV genotype 1
infection, including for those with compensated cirrhosis, in combination with
ombitasvir, ritonavir, and dasabuvir (Viekira Pak) with or without ribavirin and
approved July 24, 2015, for treatment of patients without cirrhosis with genotype 4 in
combination with ombitasvir and ritonavir (Technivie) and ribavirin
(2) See section on Viekira Pak and Technivie for detailed information.
(3) See Table 20 for more information about select combination therapy.
(c) Grazoprevir
(1) FDA indications: Approved January 28, 2016 for treatment of HCV genotype 1 and
4 infection, including for those with compensated cirrhosis, in combination with
elbasvir (Zepatier)
(2) See section on Zepatier for detailed information

ACCP Updates in Therapeutics® 2016: Ambulatory Care Pharmacy Preparatory Review and Recertification Course

1-588
Table 20. Select FDA-Approved Combination Therapies for Hepatitis C Virus
Direct Acting
SMV PTV/r/ OBV +
Antiviral LDV/SOFa PTV/r/OBVb DCV (+ SOF)a EBR/GRZa
(+ SOF)a DASa
Combination

Genotype 1 1, 4, 5, 6 1 4 1–3 1, 4

Ritonavir-boosted Ritonavir-boosted
Class PI + NS5B NS5A/NS5B NS5A + NS5B NS5A/PI
PI/NS5A + NS5B PI/NS5A
FDA Approval SMV 11/22/13 DCV 7/24/15
10/10/14 12/19/14 7/24/15 1/28/2016
Date SOF 12/6/13 SOF 12/6/13
Fatigue,
Asthenia, nausea, Asthenia, nausea,
headache, nausea, Fatigue, headache, Fatigue, headache, Fatigue, headache,
ADRs pruritus, insomnia, pruritus, insomnia,
photosensitivity, nausea, diarrhea nausea, diarrhea nausea
diarrhea diarrhea
rash, diarrhea
Gilead Sciences: Bristol-Myers Squibb:
AbbVie: AbbVie: www.bms.com/ Merck:
Janssen www.
https://www.viekira. https://www.viekira. products/Pages/ http://www.
Financial http://www.jjpaf. mysupportpath.
com/proceed-support com/proceed-support merckaccessprogram-

1-589
org/ com/ programs.aspx
Assistance zepatier.com
1-844-2PROCEED 1-844-2PROCEED 1-844-44-CONNECT
1-800-652-6227 1-855-7-MYPATH
(844-277-6233) (844-277-6233) 1-866-251-6013
(1-855-769-7284)
Gastrointestinal Disorders

(844-442-6663)
a
Used with or without RBV.
b
Used in combination with RBV.
ADR = adverse drug reaction; LDV/SOF = ledipasvir/sofosbuvir; PTV/r/OBV + DAS = paritaprevir/ritonavir/ombitasvir plus dasabuvir; PI = protease
inhibitor.

ACCP Updates in Therapeutics® 2016: Ambulatory Care Pharmacy Preparatory Review and Recertification Course
Gastrointestinal Disorders

d. NS5A inhibitors
i. Mechanism: Interfere with HCV replication by inhibiting the HCV replication complex
ii. Available agents
(a) Ledipasvir; approved in fixed-dose combination with sofosbuvir (Harvoni)
(1) FDA indication (approved October 2014):
(A) Treatment of chronic HCV genotype 1 infection as a fixed-dose combination
with sofosbuvir
(B) Not recommended to administer with other products containing sofosbuvir
(2) Dose and administration
(A) 1 tablet (90 mg of ledipasvir and 400 mg of sofosbuvir) by mouth once daily
with or without food
(B) No dose adjustments required for mild or moderate renal impairment. Limited
data exist for patients with severe renal impairment (CrCl less than 30 mL/
minute/1.73 m2) and end-stage renal disease and in dialysis; not recommended in
the guidelines
(C) No dose adjustments required for mild, moderate, or severe hepatic impairment
(Child-Pugh class A, B, or C). Safety and efficacy is unknown in those with
decompensated cirrhosis.
(3) Formulation: Oral; ledipasvir 90 mg/sofosbuvir 400 mg tablet
(4) Treatment duration; 12 or 24 weeks with or without ribavirin, depending on
genotype, previous treatment experience, and presence of cirrhosis
(5) Adverse events
(A) Most common include fatigue, headache, nausea, diarrhea, and insomnia.
(B) Safety data in prescribing guidelines are pooled from three phase III trials
in subjects with genotype 1 with or without compensated cirrhosis. Very few
subjects permanently discontinued therapy because of adverse events: 0%, less
than 1%, and 1% who received therapy for 8, 12, or 24 weeks, respectively.
(C) Laboratory abnormalities
• Bilirubin elevations: Greater than 1.5 times ULN in 3%, less than 1%, and 2%
of subjects treated for 8, 12, and 24 weeks, respectively
• Lipase elevations: Asymptomatic, transient, greater than 3 times ULN in less
than 1%, 2%, and 3% of subjects treated for 8, 12, and 24 weeks, respectively
• Creatine kinase: Not assessed in ledipasvir/sofosbuvir treatment trials.
Asymptomatic elevations (grade 3 or 4) have previously been reported with
sofosbuvir therapy.
(D) Monitoring
• Drug interactions
§ Ledipasvir inhibits P-glycoprotein (P-gp) and breast cancer resistance
protein (BCRP); therefore, if substrates of these transporters are
administered, the potential exists for increased intestinal absorption.
§ P-gp inducers (i.e., rifampin or St. John’s wort) may decrease plasma
concentrations of ledipasvir (as well as sofosbuvir; see Sofosbuvir section
below), potentially decreasing therapeutic effect. Use is not recommended.
§ Table 21 provides information on selected drug interactions according
to package insert. The website Hep-druginteractions.org provides
additional information.

ACCP Updates in Therapeutics® 2016: Ambulatory Care Pharmacy Preparatory Review and Recertification Course

1-590
Gastrointestinal Disorders

• When reconciling medications, it is important to include OTC drugs


in assessment. If possible, recommend to avoid all acid-suppressing
medications during HCV therapy to optimize drug exposure. If acid-
suppressive therapy cannot be interrupted, see Table 21 and HCV treatment
guidelines for dosing recommendations.

Table 21. Established and Potentially Significant Drug-Drug Interactions with Ledipasvir/Sofosbuvira
Concomitant Effect on Drug
Drug Names/Regimens Recommendation
Drug Class Concentration
Separate antacids by 4 hr
H2RA may be administered concurrently
Antacids or spaced 12 hr from Harvoni (max
Acid-reducing
H2RA Decrease LDV equivalent is famotidine 40 mg BID)
agents
PPI
PPI may be administered concurrently
with Harvoni, max equivalent
omeprazole 20 mg daily
Digoxin Increases digoxin Therapeutic monitoring recommended
May result in serious bradycardia.
Antiarrhythmics Coadministration with sofosbuvir and
Amiodarone Unknown another DAA is not recommended. If
coadministration is required, cardiac
monitoring is recommended
Carbamazepine
Phenytoin Decreases LDV
Anticonvulsants Do not coadminister
Phenobarbital Decreases SOF b
Oxcarbazepine
Rifabutin
Decreases LDV
Antimycobacterials Rifampin Do not coadminister
Decreases SOF b
Rifapentine
Decreases LDV
Herbal supplements St. John’s wort Do not coadminister
Decreases SOF b
• Atazanavir/ritonavir +
Safety has not been established; consider
emtricitabine/tenofovir DF
alternative HCV or antiretroviral
• Darunavir/ritonavir + Increases
therapy. If coadministration necessary,
emtricitabine/tenofovir DF tenofovir
monitor for tenofovir-associated adverse
• Lopinavir/ritonavir +
reactions
emtricitabine/tenofovir DF
HIV antiretrovirals • Efavirenz + emtricitabine Increases Monitor for tenofovir-associated adverse
+ tenofovir DF tenofovir reactions
• Elvitegravir + cobicistat + Increases
Do not coadminister
emtricitabine, tenofovir DF tenofovir
Decreases LDV
• Tipranavir/ritonavir Do not coadminister
Decreases SOF b

ACCP Updates in Therapeutics® 2016: Ambulatory Care Pharmacy Preparatory Review and Recertification Course

1-591
Gastrointestinal Disorders

Table 21. Established and Potentially Significant Drug-Drug Interactions with Ledipasvir/Sofosbuvira (continued)
Concomitant Effect on Drug
Drug Names/Regimens Recommendation
Drug Class Concentration
Increases LDV
HCV treatment Simeprevir Do not coadminister
Increases SMV
HMG-CoA
Increases
Reductase Rosuvastatin Do not coadminister
rosuvastatin
Inhibitors
This table is not inclusive; see prescribing guidelines for additional information and clinical recommendations.
a

b
Includes SOF and inactive metabolite GS-331007.
DF = disoproxil fumarate.
Adapted from: Harvoni [package insert]. Foster City, CA: Gilead, March 2015.

(b) Ombitasvir
(1) FDA indications: Approved December 2014 for treatment of HCV genotype 1
infection, including for those with compensated cirrhosis, in combination with
paritaprevir, ritonavir, and dasabuvir (Viekira Pak) and approved July 24, 2015,
for treatment of patients without cirrhosis with genotype 4 in combination with
paritaprevir and ritonavir (Technivie)
(2) See section on Viekira Pak and Technivie for detailed information.
(c) Daclatasvir (Daklinza)
(1) FDA indications: Approved July 24, 2015, for treatment of HCV genotype 3 infection
in combination with sofosbuvir; approved for genotype 1 on February 5, 2016. Pan-
genotypic. Approved in the European Union for genotypes 1–4; recommended in
U.S. guidelines for use in genotypes 1–3 at time of editing. The SVR rates in patients
with genotype 3 infection with cirrhosis are lower than with other treatment options.
See guidelines for current recommendations.
(2) Dose and administration.
(A) 60 mg daily in addition to sofosbuvir 400 mg daily, with or without ribavirin
(B) Dose may be increased to 90 mg daily when used in combination with moderate
CYP3A4 inducers, or decreased to 30 mg daily when used in combination with
strong CYP3A4 inhibitors. See the package insert for management tips for drug
interactions. Daclatasvir is currently the only DAA with adjustable dosing for
HCV treatment.
(3) Formulations: Oral daclatasvir 60- and 30-mg tablet
(4) Treatment duration: 12–24 weeks; depends on both the viral genotype and the
patient population. For current information, see the HCV treatment guidelines (www.
hcvguidelines.org).
(5) Treatment discontinuation: The entire regimen should be continued until the
appropriate duration (weeks), based on genotype and patient population, is achieved.
(6) Use is contraindicated in combination with drugs that strongly induce CYP3A4
(e.g., phenytoin, carbamazepine, rifampin, St. John’s wort), which may lead to loss
of efficacy of daclatasvir.
(7) Adverse events with daclatasvir and sofosbuvir therapy (5%–14%): Headache,
fatigue, nausea, and diarrhea
(8) Drug interactions
(A) Daclatasvir is a substrate of CYP3A4 and P-gp.
(B) See package insert for details on dose adjustments.

ACCP Updates in Therapeutics® 2016: Ambulatory Care Pharmacy Preparatory Review and Recertification Course

1-592
Gastrointestinal Disorders

(d) Elbasvir
(1) FDA indications: Approved January 28, 2016 for treatment of HCV genotype 1 and
4 infection, including for those with compensated cirrhosis, in combination with
grazoprevir (Zepatier)
(2) See section on Zepatier for detailed information
e. NS5B Polymerase inhibitors
i. Inhibit viral replication by inhibiting the HCV NS5B RNA polymerase
ii. Available agents
(a) Sofosbuvir (Sovaldi)
(1) FDA indication (approved December 2013): Treatment of chronic HCV infection in
subjects with genotypes 1, 2, 3, or 4, including those with hepatocellular carcinoma
meeting Milan criteria (awaiting liver transplantation) and those with HCV/HIV-1
coinfection as a component of a combination antiviral treatment regimen
(2) Dose and administration
(A) Sofosbuvir 400 mg by mouth once daily in combination with ribavirin or in
combination with pegylated interferon and ribavirin or simeprevir or daclatasvir.
The recommended treatment regimens depend on both viral genotype and
patient population. Sofosbuvir should not be administered as monotherapy.
(B) Doses can be taken with or without food.
(C) No dose adjustment required for mild or moderate renal impairment. Limited
data on safety and efficacy in patients with severe renal impairment (estimated
GFR less than 30 mL/minute/1.73 m2) or with end-stage renal disease. Use in
patients with a GFR less than 30 mL/minute/1.73 m2 is not recommended in the
guidelines but is used off-label in clinical practice.
(D) No dose adjustment required for mild, moderate, or severe hepatic impairment
(Child-Pugh class A, B, or C). Evaluated in patients with decompensated
cirrhosis receiving compassionate use. Recommended for use in combination
with ledipasvir (as Harvoni) in guidelines for patients decompensated cirrhosis
(E) Combination therapy with ribavirin or pegylated interferon and ribavirin
• If ribavirin and/or pegylated interferon–related adverse reactions occur, dose
reduction and/or discontinuation of these agents may be considered. See
prescribing guidelines for additional information.
• Sofosbuvir should also be discontinued if pegylated interferon or ribavirin is
permanently discontinued.
(F) Avoid concomitant administration of potent intestinal P-gp inducers (e.g.,
rifampin, St. John’s wort).
(3) Formulations: Oral; sofosbuvir 400-mg tablet. In addition, available as a fixed-dose
combination tablet Harvoni (ledipasvir 90 mg/sofosbuvir 400 mg)
(4) Treatment duration: 12–24 weeks; depends on both the viral genotype and the
patient population. For current information, see the HCV treatment guidelines (www.
hcvguidelines.org). Consideration for treatment for 8 weeks in treatment-naive
patients without cirrhosis having a baseline viral load less than 6 million IU/mL
(5) Treatment discontinuation: The entire regimen should be continued until the
appropriate duration (weeks), based on genotype and patient population, is achieved.
(6) Contraindications
(A) Hypersensitivity to sofosbuvir
(B) Pregnant women and male partners of pregnant women (because of birth defects
and fetal deaths associated with ribavirin therapy)

ACCP Updates in Therapeutics® 2016: Ambulatory Care Pharmacy Preparatory Review and Recertification Course

1-593
Gastrointestinal Disorders

(C) Contraindications for pegylated interferon and ribavirin also apply when using
these therapies in combination with sofosbuvir and should also be considered.
(7) Adverse events: Most common (greater than 20% in registration trials) are as follows:
(A) Sofosbuvir plus ribavirin therapy: Fatigue and headache
(B) Sofosbuvir plus ribavirin plus pegylated interferon therapy: Anemia, fatigue,
headache, insomnia, and nausea
(C) Adverse events for pegylated interferon and ribavirin also apply when using
these therapies in combination with sofosbuvir and should also be considered.
(8) Drug interactions
(A) Sofosbuvir is rapidly converted to its predominant metabolite, GS-331007, which
accounts for greater than 90% of drug-related systemic exposure. Sofosbuvir is a
substrate of the P-gp drug transporter and BCRP, whereas GS-331007 is not.
(B) P-gp inducers may decrease sofosbuvir plasma concentrations and should not be
concomitantly administered. Table 22 shows the drug-drug interactions that may
occur with concomitant sofosbuvir administration.

Table 22. Potentially Significant Drug-Drug Interactions with Sofosbuvira


Concomitant Effect on Effect on
Drug Names Comment
Drug Class Sofosbuvir GS-331007
May result in serious bradycardia.
Coadministration with sofosbuvir and
Antiarrhythmics Amiodarone Unknown Unknown another DAA is not recommended. If
coadministration is required, cardiac
monitoring is recommended
Carbamazepine,
phenytoin,
Anticonvulsants Decrease Decrease
phenobarbital,
oxcarbazepine
Rifabutin
Antimycobacterials Rifampin Decrease Decrease Avoid coadministration
Rifapentine
Herbal
St. John’s wort Decrease Decrease
supplements
HIV PIs Tipranavir/ritonavir Decrease Decrease
This table is not inclusive.
a

DAA = direct acting antiviral; PI = protease inhibitor


Adapted from: Sofosbuvir [prescribing guidelines]. Foster City, CA: Gilead, August 2015.

(9) Monitoring: In addition to what is recommended for pegylated interferon and


ribavirin therapy (as discussed above), the following should be considered when
using combination therapy that includes sofosbuvir.
(A) Adverse effects: Although the reported incidence of adverse effects in sofosbuvir
clinical trials is lower, clinicians should assess adverse effects at each visit and
provide continual education reminding patients to contact their health care
providers should they have any adverse events, new symptoms, or changes in
their clinical status.

ACCP Updates in Therapeutics® 2016: Ambulatory Care Pharmacy Preparatory Review and Recertification Course

1-594
Gastrointestinal Disorders

(B) Pharmacists should perform medication reconciliation at each visit to ensure


proper adherence to the HCV treatment regimen as well as the avoidance of
medications recommended not to be administered concomitantly.
(b) Dasabuvir
(1) FDA indication (approved December 2014): Treatment of chronic HCV genotype
1 infection including for those with compensated cirrhosis in combination with
ombitasvir, paritaprevir, and ritonavir (Viekira Pak)
(2) See section on Viekira Pak for more information.
f. Viekira Pak and Technivie
i. Viekira Pak combines three DAAs with distinct mechanisms of action: Ombitasvir (NS5A
inhibitor), paritaprevir (NS3/4A PI), and dasabuvir (NS5B palm polymerase inhibitor).
Ritonavir is also included; however, it is not active against HCV, it is a potent CYP3A inhibitor
used within this regimen to increase peak and trough plasma concentrations of paritaprevir
and overall exposure. Technivie combines two DAA medications with distinct mechanisms of
action: ombitasvir and paritaprevir/ritonavir.
ii. FDA indication Viekira Pak (approved December 2014): Treatment of chronic HCV infection
in subjects with genotype 1, including those with compensated cirrhosis and in liver transplant
recipients with normal hepatic function and mild fibrosis (Metavir of F2 or less). Used in
combination with ribavirin in certain patient populations. Technivie was approved July
24, 2015, for treatment of patients with genotype 4 infection without cirrhosis for use in
combination with ribavirin.
iii. Dose and administration
(a) Viekira Pak: Ombitasvir 25 mg plus paritaprevir 150 mg plus ritonavir 100 mg once
daily (in the morning) and dasabuvir 250 mg twice daily (morning and evening), with or
without ribavirin. Patients with genotype 1A will need ribavirin coadministered.
Technivie: Ombitasvir 25 mg plus paritaprevir 150 mg plus ritonavir 100 mg once daily
(in the morning); coadministered with ribavirin. See above sections and package inserts
for ribavirin dosing.
(b) Take with food without respect to fat or calorie content.
(c) No dose adjustment required for those with mild or moderate renal impairment
(d) No dose adjustment required for mild hepatic impairment (Child-Pugh class A).
Not recommended in those with moderate impairment (Child-Pugh class B) and
contraindicated in severe impairment (Child-Pugh class C)
iv. Formulations: Oral; Viekira Pak contains ombitasvir 12.5 mg, paritaprevir 75-mg, ritonavir
50-mg fixed-dose combination tablets copackaged with dasabuvir 250-mg tablets. Technivie
contains ombitasvir 12.5 mg, paritaprevir 75-mg, ritonavir 50-mg fixed-dose combination tablets
v. Treatment duration depends on patient characteristics (e.g., genotype subtype and presence of
cirrhosis).
vi. Contraindications
(a) Severe hepatic impairment
(b) If used in combination with ribavirin, contraindications for ribavirin also apply (see
prescribing guidelines).
(c) If coadministered with drugs with the following characteristics:
(1) Highly dependent on CYP3A for clearance; they increase plasma concentration and
potentially associated with serious and/or life-threatening events
(2) Strong inducers of CYP3A and CYP2C8; may lead to reduced efficacy of Viekira Pak
(3) Strong inhibitors of CYP2C8; may increase dasabuvir plasma concentrations and
risk of QT prolongation

ACCP Updates in Therapeutics® 2016: Ambulatory Care Pharmacy Preparatory Review and Recertification Course

1-595
Gastrointestinal Disorders

(d) Box 3 summarizes drugs that are contraindicated to be coadministered with Viekira Pak.

Box 3. Drugs Contraindicated with Viekira Pak Administration According to FDA Prescribing Informationa
Alfuzosin Midazolam (oral)
Carbamazepine Phenobarbital
Dihydroergotamine Phenytoin
Efavirenz Pimozide
Ergonovine Rifampin
Ergotamine St. John’s wort
Ethinyl estradiol−containing medications Sildenafil (for pulmonary hypertension when dosed as Revatio)
Gemfibrozil Simvastatin
Lovastatin Triazolam
Methylergonovine
aThis list is not inclusive. These medications are contraindicated; however, other drugs metabolized by similar enzyme systems also may have significant
drug interactions requiring dose adjustments and additional monitoring (see prescribing guidelines for additional information).
Adapted from: Viekira Pak [package insert]. North Chicago, IL: AbbVie, December 2014.

vii. Adverse events


(a) The most notable adverse events from use of Viekira Pak with ribavirin were fatigue,
nausea, pruritus, other skin reactions, insomnia, and asthenia. In Viekira Pak alone:
Nausea, pruritus, and insomnia
(b) Safety data in prescribing guidelines were based on data from six phase III trials in subjects
receiving Viekira Pak with or without ribavirin for 12–24 weeks. As expected, some
adverse events occurred more commonly in those with compensated cirrhosis treated for 24
weeks compared with 12 weeks. For example, fatigue, skin reactions, and dyspnea occurred
at least 5% more often when treated for 24 weeks. However, adverse events leading to
treatment discontinuation occurred in 2% of patients in both the 12- and 24-week arms.
(c) Skin reactions: Most events were mild in severity, with no serious events such as Stevens-
Johnson syndrome, toxic epidermal necrolysis, erythema multiforme, or drug rash with
eosinophilia and systemic symptoms.
(d) Laboratory abnormalities
(1) Serum ALT elevations: Asymptomatic; about 1% had elevations greater than 5 times
ULN with increased incidence in women taking concomitant ethinyl estradiol. Typically
occurred within the first 4 weeks of therapy, and most resolved with ongoing therapy. ALT
elevations did not appear to be associated with bilirubin elevations, and cirrhosis was not a
risk factor in the clinical trial data.
(2) Serum bilirubin elevations: Typically occurred after therapy initiation, peaked by study
week 1 and resolved with ongoing therapy. Elevations were predominantly indirect and
related to bilirubin transporter inhibition by paritaprevir- and ribavirin-induced hemolysis.
(3) Anemia/decreased hemoglobin: Mean change in hemoglobin concentration in subjects
treated with Viekira Pak in combination with ribavirin was −2.4 g/dL compared with
−0.5 g/dL when treated with Viekira Pak alone. Hemoglobin concentration changes
resulted in a ribavirin dose reduction during treatment in 7% of subjects, requirement
for a blood transfusion in three subjects, and epoetin in five subjects. Data indicated
that only one subject discontinued therapy because of anemia. No patient taking
Viekira Pak alone had a hemoglobin concentration less than 10 mg/dL.
(4) When used in special populations (i.e., HCV/HIV-coinfected and liver transplant
recipients), the incidence of adverse events was increased. See prescribing guidelines
for details.

ACCP Updates in Therapeutics® 2016: Ambulatory Care Pharmacy Preparatory Review and Recertification Course

1-596
Gastrointestinal Disorders

viii. Drug interactions


(a) Viekira Pak and Technivie have the potential to affect other drugs that are substrates
of CYP3A, UGT1A1, BCRP, OATP1B1, or OATP1B3, resulting in increased plasma
concentrations of such drugs. Below is a list of the specific enzymes and transporters
combinations inhibited.
(1) Ombitasvir inhibits UGT1A1 and BCRP.
(2) Paritaprevir inhibits UGT1A1, BCRP, OATP1B1, and OATP1B3.
(3) Dasabuvir inhibits UGT1A1 and BCRP.
(4) Ritonavir inhibits CYP3A4.
(b) Coadministration of other drugs may affect one or more components of Viekira Pak.
(1) Paritaprevir and ritonavir are primarily metabolized by CYP3A enzymes; thus, if they
are administered together with CYP3A inhibitors, their concentrations may increase.
(2) Dasabuvir is primarily metabolized by CYP2C8 enzymes; thus, if it is administered
with CYP2C8 inhibitor, its concentration may increase.
(3) Ombitasvir is minimally metabolized by CYP enzymes; rather, its metabolism
occurs through amide hydrolysis.
(4) All components of Viekira Pak and Technivie are substrates of P-gp, and all except
ritonavir are substrates of BCRP. Paritaprevir is also a substrate of OATP1B1 and
OATP1B3. Thus, inhibition of P-gp, BCRP, OATP1B1, or OATP1B3 may lead to
increases in various components of the HCV combination.
(c) Information regarding established and potential drug interactions studied is available
in the prescribing information.
g. Zepatier
i. FDA indication (approved January 28, 2016): Treatment of chronic HCV infection in subjects
with genotypes 1 or 4.
ii. Dose and administration
(a) 1 tablet (50 mg of elbasvir and 100 mg of grazoprevir) by mouth once daily with or
without food
(b) No dose adjustments required for mild, moderate, or severe renal impairment. Approved
for use in end-stage renal disease and in dialysis.
(c) No dose adjustments required for patients with compensated cirrhosis (Child-Pugh class A).
iii. Formulation: Oral; elbasvir 50 mg/grazoprevir 100mg tablet
iv. Treatment duration; 12 or 16 weeks with or without ribavirin, depending on genotype,
previous treatment experience, and presence of baseline NS5A polymorphisms. See the
package insert for details.
v. Contraindications
(a) Use is contraindicated in patients with decompensated cirrhosis (Child-Pugh class B or C)
(b) Contraindicated in patients taking strong OAT1B1/3 inhibitors, strong CYP3A4 inducers,
and efavirenz
vi. Adverse events
(a) Most common include fatigue, headache, and nausea.
(b) Laboratory abnormalities
(1) ALT elevations to 5 times the ULN occurred in 1% of patients in clinical trials, at
or after week 8 of treatment. Most were asymptomatic and most resolved during
treatment or upon completion of treatment

ACCP Updates in Therapeutics® 2016: Ambulatory Care Pharmacy Preparatory Review and Recertification Course

1-597
Gastrointestinal Disorders

`vii. Monitoring
(a) Testing prior to treatment initiation:
(1) If genotype 1a, test for the presence of NS5A resistance-associated polymorphisms to
determine treatment duration and if use of ribavirin is necessary.
(2) Obtain hepatic laboratory testing at baseline.
(b) Must check ALT at week 8 and as clinically indicated.
(1) For patients receiving 16 weeks of treatment, check again at week 12.
(c) Must monitor for ribavirin-associated reactions for patients also receiving ribavirin
viii. Drug interactions
(a) Contraindicated in patients taking strong OAT1B1/3 inhibitors, strong CYP3A4 inducers,
and efavirenz
(b) Information regarding established and potential drug interactions studied is available in
the prescribing information.
9. Prevention
a. No HCV vaccine currently available
b. Goals of HCV prevention and control (CDC: National HCV Prevention Strategy)
i. Reduce the incidence of new infection by reducing disease transmission.
ii. Reduce the risk of chronic liver disease in infected individuals through appropriate medical
management and counseling.
c. Methods to achieve goals
i. Identify individuals at risk, and provide them with education, screening, and medical services.
ii. Preventing or changing behaviors in these high-risk groups
(a) Targeted screening and outreach programs
(b) Syringe exchange programs
(c) Inmate education
10. Patient education resources (Domain 2, Task 4,5)
a. www.cdc.gov/hepatitis/HCV/PatientEduHCV.htm
b. Medication assistance programs are often available for patients who meet income criteria.
See Table 20 for more information.

Patient Cases

8. A 44-year-old woman (weight 79 kg) with a history of chronic HCV (genotype 1a) infection and GERD
presents for assessment. Recent laboratory values include the following: SCr 1.1 mg/dL, AST 157 IU/mL,
ALT 321 IU/mL, total bilirubin 1.1 g/dL, INR 1.1, albumin 3.3 g/dL, TSH 1.8 mIU/L, and HCV RNA
387,000 IU/mL. She has never been treated for HCV. A liver biopsy performed 3 weeks ago showed stage 2
fibrosis. Which is the best option for initiating HCV therapy in this patient?
A. Ledipasvir/sofosbuvir once daily plus ribavirin twice daily for 24 weeks.
B. Ledipasvir/sofosbuvir once daily for 12 weeks.
C. Paritaprevir/ritonavir/ombitasvir once daily plus dasabuvir twice daily for 12 weeks.
D. Paritaprevir/ritonavir/ombitasvir once daily plus dasabuvir twice daily for 24 weeks.

ACCP Updates in Therapeutics® 2016: Ambulatory Care Pharmacy Preparatory Review and Recertification Course

1-598
Gastrointestinal Disorders

V.  MALABSORPTION SYNDROME

A. Definition: Failure of the GI tract to absorb nutrients, ions, or water adequately

B. Pathophysiology (Domain 1, Task 2)


1. Damage to the GI tract with or without morphologic change, resulting in defective absorptive
processing during the luminal, mucosal, or removal phase of digestion; may involve a variety of
factors, including genetics, immune mediated, infectious, inflammation, and toxins. In addition, if a
portion of the GI tract is removed because of trauma, disease, or surgery, normal absorptive processes
may be impaired.
2. Common or associated causes of malabsorption include the following:
a. Amyloidosis
b. Bariatric surgery
c. Carbohydrate malabsorption (e.g., lactose intolerance)
d. Celiac disease
e. Crohn disease (CD)
f. Infection
g. Short-bowel syndrome
h. Small-bowel bacterial overgrowth
i. Vitamin B12 deficiency
j. Whipple disease
k. Zollinger-Ellison syndrome

C. Clinical Presentation
1. Range of functional impairment depends on extent of disturbance to the GI tract. Common signs and
symptoms may include abdominal bloating and cramping, diarrhea, flatulence, and weight loss.
2. Long-term effects may include deficiencies in iron, vitamins, and proteins, leading to a variety of
consequences; for example:
a. Poor vision, night blindness, and dermatitis caused by impaired vitamin A absorption
b. Vitamin B12, folate, and iron-deficiency anemia
c. Bleeding because of vitamin K malabsorption
d. Osteomalacia caused by calcium deficiency
e. Osteoporosis caused by protein deficiency

D. Diagnosis (Domain 1, Task 1; Domain 4, Task 3)


1. Comprehensive history (including medications, travel history, diet, and family history) and physical
examination will guide evaluation.
2. Stool evaluation and laboratory tests to assess for hydration and electrolyte status, together with screening
tests for various causes of malabsorption (e.g., the hydrogen breath test for carbohydrate malabsorption;
serologic testing to screen for celiac disease). Microbiologic and stool studies to assess stool volume and
appearance may be helpful in some cases. Consider diagnostic studies and imaging to obtain additional
information (e.g., ultrasonography, endoscopy with or without biopsies, barium studies).

E. Treatment (Domain 1, Task 3,4,6,7)


1. Fluid and electrolyte correction
2. Ongoing nutritional support to manage deficiencies

ACCP Updates in Therapeutics® 2016: Ambulatory Care Pharmacy Preparatory Review and Recertification Course

1-599
Gastrointestinal Disorders

3. Depends on underlying mechanism of malabsorption and often includes removing or avoiding the
likely pathogen or cause, when possible. For example:
a. Celiac disease
i. Results when the intestinal lining is damaged by gluten
ii. Treatment consists of a diet devoid of gluten, which will restore the damaged mucosal lining
and allow it to function normally.
iii. Treatment of refractory disease in those not responding to a gluten-free diet may consist of
corticosteroids and immunosuppressants (e.g., azathioprine).
b. Lactose intolerance
i. Results from the inability to digest lactose completely because of a lactase deficiency.
Considered intolerance when signs and symptoms are present
ii. Treatment is a lactose-free or lactose-reduced diet. OTC products containing lactase may
help; however, reports regarding efficacy are variable and inconclusive.
c. Whipple disease
i. Extremely uncommon; results from a systemic bacterial infection with Tropheryma whipplei;
which results in lesions on the mucosa of the small intestine and damage to the villi; if
untreated, can spread to the CNS
ii. Treatment is long-term use of antibiotics; duration may be 1–2 years. In most cases,
patients are initiated on intravenous ceftriaxone therapy for 14 days, followed by oral
sulfamethoxazole/trimethoprim orally. Antibiotic therapy may be tailored in severe cases
involving additional organ systems or relapse.
d. Bariatric surgery: An elective weight-loss surgical procedure aimed at reducing caloric intake by
modifying the anatomy of the GI tract. These procedures can be restrictive, malabsorptive, or a
combination of both.
i. The laparoscopic Roux-en-Y gastric bypass (LRYGB) involves both a restrictive and a
malabsorption process, reducing the available surface area of the stomach and small intestine.
(a) This procedure creates iron, vitamin, and protein deficiencies.
(b) May affect drug absorption and metabolism. Various pharmacokinetic studies of patients
who have undergone LRYGB have alterations in pharmacokinetic profiles. Pharmacists
caring for these patients need to assess the impact of these procedures, not only for
nutritional deficiencies but also for potential differences in drug pharmacokinetics.
ii. The laparoscopic sleeve gastrectomy creates a small or “sleeved” stomach without altering
the intestine.
(a) Unlikely to cause malabsorption deficiencies or alterations in drug absorption
and metabolism
(b) Normal drug pharmacokinetics in this patient population are assumed, but studies are
warranted. In this patient population, patients’ overall total food consumption post-
surgery can be expected to decline; thus, nutritional supplementation may be warranted.
e. Teduglutide (Gattex) for short-bowel syndrome in adults
i. GLP-2 (glucagon-like peptide 2) analog
ii. Dose: 0.05 mg/kg subcutaneously once daily. Reduce by 50% for moderate and severe renal
impairment (CrCl less than 50 mL/minute/1.73 m2). No adjustments necessary for mild to
moderate hepatic impairment; not studied in severe hepatic impairment
iii. Most commonly reported adverse events (10% or more of clinical trial subjects): Abdominal
pain and distension, injection-site reactions, nausea, headaches, and upper respiratory infection

ACCP Updates in Therapeutics® 2016: Ambulatory Care Pharmacy Preparatory Review and Recertification Course

1-600
Gastrointestinal Disorders

VI.  DIARRHEA (GASTROENTEROLOGY 1999;116:1464-86)

A. Definition
1. Diarrhea is an altered passage of stool resulting in increased frequency of bowel movements (i.e.,
three or more in 24 hours) and/or increased stool weight (greater than 200 g) and/or decreased stool
consistency (i.e., increased fluidity of stool).
2. The subtypes of diarrhea, acute and chronic, are defined with respect to symptom duration; acute
diarrhea lasts less than 2 weeks, persistent diarrhea lasts 2–4 weeks, and chronic diarrhea lasts more
than 4 weeks.

B. Pathophysiology (Domain 1, Task 2)


1. Normally, during the digestive process, the small intestine and colon absorb water. Diarrhea may occur
when this process is altered. The following mechanisms can cause diarrhea. Two or more mechanisms
may be present simultaneously.
a. Osmotic: A hyperosmolar gradient is created within the intestinal lumen; there is decreased
absorption of solutes from intestines (e.g., celiac disease).
b. Motility: Shortened transit time of food traveling through intestines; water and electrolytes are
not properly absorbed
c. Exudative/inflammatory: Injury or ulceration produces intestinal swelling, causing increased
mucus, serum proteins, and blood into the bowel lumen; impaired water and electrolyte absorption
by inflamed intestine
d. Secretory: Increased electrolytes/ions pass into the lumen of the intestines; additional fluid follows
into the intestines. (e.g., cholera toxin).
2. Common causes may include infection, diseases, lactose intolerance, artificial sweeteners
(e.g., sorbitol and mannitol), surgical procedures, and medications.
a. Of importance, the definition of diarrhea does not include abdominal pain. This must be
considered to differentiate diarrhea from other diseases of which diarrhea may be a symptom
(e.g., irritable bowel syndrome [IBS] and inflammatory bowel disease [IBD]).
b. More than 700 medications have been implicated as causing diarrhea, accounting for about
7% of drug-related adverse effects. Box 4 lists some of the most common medications leading
to medication-induced diarrhea.

Box 4. Common Medications Leading to Medication-Induced Diarrhea


α-Glucosidase inhibitors Levothyroxine
Antacids containing magnesium Lithium
Antibiotics Metoclopramide
Antineoplastics Mycophenolate-containing immunosuppressants
Cholinesterase inhibitors NSAIDs
Colchicine Orlistat
Digoxin PIs
Laxatives Prostaglandins
NSAID = nonsteroidal anti-inflammatory drug; PI = protease inhibitor.

C. Clinical Presentation: Varies; those with mild illness may report diarrhea without other concerns, whereas those
with more severe disease may report abdominal cramping/pain, fever, and symptoms of dehydration

ACCP Updates in Therapeutics® 2016: Ambulatory Care Pharmacy Preparatory Review and Recertification Course

1-601
Gastrointestinal Disorders

D. Diagnosis (Domain 1, Task 1; Domain 4, Task 3)


1. Comprehensive history will guide evaluation. Determine the characteristics and frequency of
evacuation as well as the duration of symptoms. Assess the patient’s family, travel, and food histories;
specifically, assess whether the patient has had any recent changes in travel or diet.
2. Important to distinguish between the acute and chronic forms of disease because the etiologies,
diagnostic testing, and treatment differ
3. Evaluate for common medical conditions associated with diarrhea (e.g., IBS, IBD).
4. Evaluate for drug-induced causes (e.g., recent antibiotic use, laxatives). Important to seek information regarding all
current medications, including OTC drugs, nutritional supplements, illicit drugs, and alcohol
5. Physical examination and laboratory tests (e.g., CBC, chemistry panel) and stool analysis. Some tests
are not universally performed but are ordered if appropriate according to the clinical situation (e.g.,
check fecal occult blood test, assess for ova/parasites if infectious cause is suspected, and evaluate for
C. difficile if recent hospitalizations or antibiotics).
6. Further diagnostic investigation with an abdominal CT scan and/or endoscopy may be necessary in
select patients.

E. Treatment (Domain 1, Task 3,4,6,7)


1. Identify and remove/treat cause when possible. If a medication is likely the cause, consider switching
or removing the agent.
2. Fluid and electrolyte correction, especially in large-volume diarrhea
a. Severely ill patients will probably require hospitalization for intravenous solutions of water, salt,
and sugar.
b. Those with mild disease who are not vomiting may benefit from oral rehydration solutions such as
WHO-ORS (Jianas Brothers) and Rehydrate (Ross Laboratories) or CeraLyte, which is available OTC.
c. Sports drinks such as Gatorade are not equivalent to oral rehydration solutions; usually, they have
additional sugar carbohydrate calories.
3. Diet modifications
a. If diarrhea is determined to be temporally related to specific foods, instruct the patient to avoid those
foods, and monitor for improvement of symptoms. Examples of ingredients that may cause diarrhea:
i. Lactose; avoid dairy products. Can add lactase to assist with breakdown of lactose
ii. Sorbitol; avoid sugar-free products made with sorbitol
iii. Olestra; avoid fat-free products made with olestra
b. BRAT diet (e.g., bananas, rice, applesauce, and toast) includes foods that are plain and low in fiber;
this diet is commonly recommended in conditions producing GI distress such as diarrhea.
4. Pharmacotherapy
a. Antibiotics; may be necessary in patients with a known or suspected infectious cause (see the
Infectious Diseases II chapter for management of infectious causes)
b. Although options for treatment are available, an algorithm for management is not clear. Guidelines
for chronic diarrhea are expected from AGA in 2017. Controlled trials evaluating overall efficacy,
impact on acute versus chronic disease, and economic aspects of therapy are lacking. Often,
pharmacotherapy is used in combination with other treatments. See Table 23 for a summary of
commonly used agents.
i. Loperamide: Used as an adjuvant to other therapies
ii. Octreotide: Believed to be less effective than opioids. Second-line therapy because of non-oral
route of delivery and expense
iii. Bismuth subsalicylate: Effect on chronic diarrhea has not yet been determined.

ACCP Updates in Therapeutics® 2016: Ambulatory Care Pharmacy Preparatory Review and Recertification Course

1-602
Table 23. Summary of Select Antidiarrheal Agentsa
Route of Common Adverse
Medication Indication/Use Adult Dose Administration Effects Comments
• Available OTC
4 mg initially; then Abdominal pain • 16 mg/day maximal daily
Loperamide
Acute and chronic diarrhea 2 mg after each Oral Constipation dose
(Imodium)
unformed stool Xerostomia • Minimal CNS effects (does
not cross BBB)
2.5–5 mg QID Abdominal discomfort,
until initial control nausea, vomiting,
Diphenoxylate • 20 mg/day maximal daily
Acute and chronic diarrhea achieved; then Oral dizziness, sedation,
(Lomotil) dose
reduce dose to somnolence, euphoria, or
amount necessary malaise
• Avoid in pregnancy and
525 mg (two tablets Constipation, diarrhea, lactation, age <12 yr
Bismuth
Acute diarrhea, traveler’s or 30 mL) every 30 nausea or vomiting, • May bind other medications
subsalicylate Oral
diarrhea minutes to 1 hr as black stool and tongue and interact with
(Pepto-Bismol)

1-603
needed discoloration anticoagulants
• 4200 mg/day is maximal dose

Treatment of severe non- doses range; 50 Subcutaneous Injection-site reaction,


Gastrointestinal Disorders

infective diarrhea, vasoactive mcg 1–2 per day, or intravenous; nausea, abdominal pain, • Up to 750 mcg/day in two to
Octreotide acetate
intestinal peptide-secreting and titrate dose intramuscular constipation, diarrhea, four divided doses is maximal
(Sandostatin)
tumors or chemotherapy- according to intragluteal also flatulence, glycemic daily dose
induced diarrhea indication available changes, and gallstones

Probiotics
(lactobacilli, • Used mainly to prevent
Manage and prevent acute Variable depending Flatulence, but usually
Saccharomyces) Oral diarrhea associated with
diarrhea on product selected well tolerated
• Many products antibiotic use
available
a
Information in this table is not all-inclusive. Please see prescribing guidelines for complete information.
BBB = blood brain barrier; CNS = central nervous system; OTC = over the counter.

ACCP Updates in Therapeutics® 2016: Ambulatory Care Pharmacy Preparatory Review and Recertification Course
Gastrointestinal Disorders

VII. CONSTIPATION (Am J Gastroenterol 2014;109:S2-26; Gastroenterology 2013;144:218-38)

A. Chronic Idiopathic Constipation (CIC): Also called functional constipation


1. Defined (according to Rome III) as the presence of two or more of the following criteria:
a. Straining during defecations (at least 25% of the time)
b. Lumpy or hard stools (at least 25% of the time)
c. Sensation of incomplete evacuation (at least 25% of the time)
d. Sensation of anorectal obstruction or blockage (at least 25% of the time)
e. Manual maneuvers to facilitate evacuation (at least 25% of the time)
f. Less than three defecations per week
2. Chronic in that symptom onset must occur at least 6 months before diagnosis and the above criteria
should be fulfilled for at least 3 months
3. Chronic constipation does not fulfill the criteria for IBS because of the lack of symptoms of discomfort
and pain; however, it is important to understand that in practice, a clear separation between CIC and
IBS may be challenging.

B. Pathophysiology (Domain 1, Task 2)


1. May be primary (i.e., idiopathic) or secondary and is often multifactorial. Other options:
a. Slow-transit constipation: Peristalsis is absent or deficient.
b. Outlet-dysfunction (defecatory disorders); pelvic floor or anal sphincter dysfunctional or unable to relax
2. Risk factors include female, older adult, and lower socioeconomic status or education level, low activity
and/or inactivity, nutritional issues (e.g., low dietary fiber, inadequate fluid intake), depression
3. Common causes of constipation may include the following:
a. Gastrointestinal (e.g., IBS, dyspepsia, GERD, diverticulosis)
b. Metabolic and endocrine causes or disorders (e.g., diabetes, hypothyroidism, Addison disease,
hypokalemia, hypercalcemia, pregnancy)
c. Neurologic/neurogenic (e.g., spinal cord injury, stroke, Parkinson disease, multiple sclerosis)
d. Functional (e.g., dehydration, decreased fiber and food intake, decreased activity level,
withholding/ignoring urge to defecate)
e. Medication induced (see Table 24)

Table 24. Common Medications That Can Cause Constipation (not all-inclusive)
Anticholinergics Neural Agents Other Over-the-counter Medications
Antidepressants Antihypertensives Polystyrene sodium sulfonate Antacids
Antiparkinsonian Calcium channel blockers Some chemotherapeutic Antidiarrheal agents
agents Diuretics agents Antihistamines
Antipsychotics Opioids Phenothiazines Calcium supplements
Antispasmodics Serotonin-3 antagonists Iron supplements
Anticonvulsants Tricyclic antidepressants Nonsteroidal anti-inflammatory
medications

C. Clinical Presentation
1. Variable; signs and symptoms may include infrequent bowel movements (less than three per week);
stools that are hard, small, or dry; abdominal bloating and/or discomfort; difficulty and/or pain when
defecating; and feeling of incomplete evacuation (tenesmus). Loose stools rarely present without the
use of laxatives.

ACCP Updates in Therapeutics® 2016: Ambulatory Care Pharmacy Preparatory Review and Recertification Course

1-604
Gastrointestinal Disorders

2. Alarm symptoms and signs indicate a need for diagnostic testing (Box 5).

Box 5. Constipation Alarm Symptoms and Signs


Hematochezia
Weight loss (> 4.5 kg)
Family history of colon cancer or IBD
Anemia
Positive fecal occult blood tests
Acute onset of constipation in older adults
IBD = inflammatory bowel disease.

D. Diagnosis (Domain 1, Task 1; Domain 4, Task 3)


1. CIC is not associated with abnormal laboratory, radiographic, or endoscopic findings; diagnosis
depends on clinical assessment and ruling out other etiologies.
2. A comprehensive history will guide the evaluation. Determine the duration of symptoms (must be
greater than 6 months) as well as the characteristics and frequency of evacuation (must meet two or
more of the criteria listed above for a minimum of 3 months). In addition, try to identify the features
most distressing to the patient.
3. Evaluate for common medical conditions associated with constipation (e.g., malignancy, neurologic disorders).
4. Evaluate for drug-induced causes (i.e., opioids, iron supplementation). Information should be obtained
regarding all current medications, including OTC and nutritional supplements, with interest focused on
agents that may induce constipation.
5. Evaluate nutritional intake and typical diet patterns, with interest focused on deficiencies in high-fiber food.
6. Physical examination (including perineal/rectal examination) and laboratory tests necessary to assess
hydration, thyroid function, and other potential causes that may have been identified
7. Diagnostic imaging studies indicated in individuals with alarm symptoms and signs (Box 5)
8. Colon cancer screening recommended in individuals 50 years and older
9. Certain patient populations, including those with continuing symptoms despite treatment, should be
referred to a gastroenterologist.

E. Treatment (Domain 1, Task 3,4,6,7)


1. Indicated when symptoms reduce patients’ quality of life
2. Goals of therapy are to (1) relieve symptoms, (2) reestablish normal bowel habits, and (3) improve
quality of life.
3. Identify and remove/treat cause when possible. If a medication is the likely cause, consider switching
or removing the agent.
4. Typically, a combination of nonpharmacologic and pharmacologic measures are used.
a. Nonpharmacologic
i. Increase hydration; recommended to drink at least eight 8-oz glasses of water per day
ii. Alter lifestyle by increasing physical activity (exercise), and plan for dedicated time to defecate.
iii. Modify diet: Gradually increase fiber consumption to 20–30 g daily.
(a) Fiber can be part of ingested food or taken as a supplement and is classified as soluble
(e.g., psyllium) or insoluble (e.g., bran).
(1) Guidelines recommend psyllium (Metamucil, Konsyl) to increase stooling frequency;
commonly associated with bloating. Safe in pregnancy
(2) No recommendations for the use of other agents (calcium polycarbophil, methylcellulose,
and wheat bran) because of insufficient data
(3) Common adverse effects: Abdominal bloating, distension, and flatulence. No
differences reported with respect to adverse events among bulking agents

ACCP Updates in Therapeutics® 2016: Ambulatory Care Pharmacy Preparatory Review and Recertification Course

1-605
Gastrointestinal Disorders

(b) Data suggest that fiber intake is beneficial; however, definitions of improvement vary
among trials. Therapy may be limited because of adverse events such as bloating,
distension, flatulence, and cramping. It is recommended to gradually increase intake to
minimize adverse events.
b. Pharmacologic (see Table 25 for select agents)
i. Use empiric therapy without diagnostic testing in constipation without alarm symptoms.
ii. If alarm symptoms present, further workup is needed to identify underlying disease. If cause
is identified, attempts should be made to correct it (e.g., malignancy identified; surgical
resection, when possible). Empiric therapy may or may not be initiated simultaneously,
depending on the clinical situation.
iii. Laxative therapy (first-line therapy): Indicated for intermittent or chronic constipation.
Although data are minimal, the osmotic (i.e., polyethylene glycol 3350 electrolyte and
lactulose) and stimulant (i.e., bisacodyl) agents appear to be effective. Other agents have not
been adequately studied.
(a) Osmotic: Contain poorly absorbed ions or molecules, which retain water in intestinal lumen
(1) Lactulose: Nonabsorbable disaccharide metabolized by colonic bacteria; causes
colon to retain fluid, increasing stool frequency and consistency
(2) Polyethylene glycol (MiraLAX) improves stool frequency and consistency. It is
not metabolized by colonic bacteria or absorbed systemically, so there are minimal
adverse effects. Daily use of 17 g (low dose) for up to 6 months is safe and effective.
(3) Others – Saline laxatives
(A) Magnesium hydroxide
• Guidelines provide no recommendations regarding its use because of
insufficient data.
• Adverse effects: Electrolyte abnormalities, hypovolemia, diarrhea
(B) Magnesium citrate, magnesium sulfate, and sodium phosphate
• Indicated for occasional constipation
• Available OTC and by prescription
(b) Stimulant: Stimulates fluid and electrolyte excretion by the colon or induces peristalsis,
subsequently inducing defecation
(1) Onset: 6–12 hours after administration
(2) Include Senna (Senokot, Ex-Lax) and bisacodyl (Dulcolax, Correctol, Carter’s Little Pills)
(3) Typically reserved for patients whose bulking and osmotic laxative therapy fails;
should be used intermittently to avoid reliance
(4) Common adverse events: Abdominal cramping and electrolyte imbalance
iv. Pro-secretory agents: Effective in CIC and well tolerated; no comparative studies of
lubiprostone and linaclotide have been conducted
(a) Lubiprostone (Amitiza): Prescription product; chloride channel activator; increases
intraluminal fluid secretion, which softens stool and increases GI transit time
(b) Linaclotide (Linzess): Prescription agent; synthetic amino acid peptide; increases
intestinal fluid and motility
v. Probiotics – Few studies report that they are effective for CIC. However, there were no
randomized controlled trials and pooled data were not statistically significant. Currently,
evidence is insufficient to recommend use of probiotics.
vi. Biofeedback – Targets pelvic floor dysfunction or dyssynergia, which may be potential causes
of constipation. Performed by trained and skilled therapist, is effective in CIC symptom relief
in those with pelvic floor dysfunction

ACCP Updates in Therapeutics® 2016: Ambulatory Care Pharmacy Preparatory Review and Recertification Course

1-606
Gastrointestinal Disorders

vii. Others – Emollients and glycerin can be considered; however, not effective for constipation
that is chronic; rather to be used before it becomes chronic. Emollients (i.e., stool softeners):
Increase stool moisture content, which should make stool easier to pass

Table 25. Summary of Select Agents Used for Constipationa


Route of Common
Medication Indication/Use Adult Dose Comments
Administration Adverse Effects
Abdominal
cramping, bloat-
Used in chronic,
ing, flatulence,
normal, and Oral (rectal also Onset 1–3 days if oral;
Lactulose 15–60 mL daily nausea; diarrhea
slow-transit an option) 15–30 min if rectal
and electrolyte
constipation
imbalances may
also occur
Onset in 1–3 days for
1 tbsp (17 g)
Abdominal daily use; 30–60 min
Polyethylene Used as a bowel mixed in 8 oz of
cramping, for prep
glycol prep before liquid daily (or Oral
nausea, bloating,
(MiraLAX) colonoscopy as directed for Available OTC
anal irritation
prep)
Safe in pregnancy
Abdominal
Docusate cramping,
sodium Prevention of Sodium salt: diarrhea,
(Colace) constipation 100 mg BID; nausea; well
Oral Onset 1–3 days
Docusate (including calcium salt: tolerated;
calcium opioid-induced) 240 mg daily may lead to
(Surfak) soiling of
undergarments
CIC: 145 mcg
by mouth once Pregnancy category C
daily at least 30
Contraindicated
min before the
in patients < 6 yr
first meal of the Abdominal
Linaclotide and in patients
CIC and IBS-C day; IBS-C: 290 Oral pain, diarrhea,
(Linzess) with mechanical
mcg by mouth flatulence
obstruction; avoid use
once daily at
in patients 6–17 yr
least 30 min
before the first
meal of the day

ACCP Updates in Therapeutics® 2016: Ambulatory Care Pharmacy Preparatory Review and Recertification Course

1-607
Gastrointestinal Disorders

Table 25. Summary of Select Agents Used for Constipationa (continued)


Route of Common
Medication Indication/Use Adult Dose Comments
Administration Adverse Effects
Onset in 24–48 hr
Pregnancy category C
CIC and IBS-C
CIC: 24 mcg Expensive; reserved
in women;
BID with food Dose-dependent for those whose
opioid-induced
Lubiprostone and water; IBS- diarrhea, conventional therapies
constipation Oral
(Amitiza) C: 8 mcg BID headache, and have failed
in patients
with food and nausea
with chronic Contraindicated
water
noncancer pain in patients with
mechanical
obstruction
Treat or prevent
Glycerin constipation or 3 g suppository Rectal irritation,
Rectal Onset in 30–60 min
(Fleet’s) fecal impaction daily – BID cramping
or for bowel prep
CIC = chronic idiopathic constipation; IBS-C = constipation-predominant irritable bowel syndrome.

VIII.  NAUSEA AND VOMITING

A. Definition
1. Nausea: Subjective sensation of the need to vomit that precedes vomiting and may occur in the
absence of vomiting
2. Vomiting: Physical event producing rapid and forceful ejection of gastric contents

B. Pathophysiology (Domain 1, Task 2)


1. Cause is likely to be multifactorial.
2. The vomiting center is located within the medulla and can be stimulated directly or indirectly. Four
principal areas (i.e., cerebral cortex, chemoreceptor trigger zone, visceral region, and vestibular region)
are involved in activation of the vomiting center. On receptor firing, neuronal impulses are sent from
these areas to the vomiting center, producing emesis.
a. Dopamine, histamine, muscarinic, serotonin, and neurokinin receptor types are involved.
The specificity of receptor-type activation within the area varies depending on the type of
clinical stimuli involved.
b. Table 26 summarizes examples of clinical stimuli and types of receptors activated within
each anatomic area.

ACCP Updates in Therapeutics® 2016: Ambulatory Care Pharmacy Preparatory Review and Recertification Course

1-608
Gastrointestinal Disorders

Table 26. Examples of Clinical Stimuli and Types of Receptors Activated


Anatomic Area Examples of Clinical Stimuli Receptors Activated
Unclear,
Cerebral cortex Noxious orders, tastes, or visions
cannabinoid (CB1)
Dopamine (D2)
Histamine (H1)
CTZ Medications, metabolic, bacterial toxins, radiation therapy Muscarinic (M1)
Neurokinin (NK1)
Serotonin (5-HT3)
Gastric irritants, distension, peritonitis, chemotherapy, Dopamine (D2)
Visceral
pharyngeal stimulation Serotonin (5-HT3)
Histamine (H1)
Vestibular Labyrinthine disease, motion sickness
Muscarinic (M1)
CTZ = chemoreceptor trigger zone.

3. Several different common medical conditions are associated with nausea and vomiting (not all inclusive).
a. Cardiopulmonary disease (e.g., heart failure, myocardial infarction)
b. Drug withdrawal (e.g., opiates, benzodiazepines)
c. GI (e.g., gastroparesis, CD, gastric outlet obstruction)
d. Infectious (e.g., gastroenteritis [bacterial or viral])
e. Medication induced: May be an adverse effect of the medication or related to a subtherapeutic or
supratherapeutic medication serum concentration (Box 6)
f. Metabolic and endocrine disorders (e.g., pregnancy, uremia, Addison disease)
g. Neurologic (e.g., migraine, seizure, motion sickness)
h. Postoperative
i. Psychiatric diseases (e.g., anxiety, psychogenic vomiting)

Box 6. Common Medications Causing Nausea and Vomiting


Anesthetics Nicotine
Antibiotics Opioids
Anticonvulsants Radiation therapy
Chemotherapy Theophylline
Dopamine agonists

C. Clinical Presentation
1. Variable; signs and symptoms range from mild to severe. May include signs of dehydration and
electrolyte imbalance
2. Associated symptoms such as abdominal pain, headache, fever, queasiness, weight loss, and vertigo
may also be present.

ACCP Updates in Therapeutics® 2016: Ambulatory Care Pharmacy Preparatory Review and Recertification Course

1-609
Gastrointestinal Disorders

D. Diagnosis (Domain 1, Task 1; Domain 4, Task 3)


1. Comprehensive history and physical examination will guide evaluation. Determine symptom severity,
frequency, and duration. Differential varies between acute and chronic presentations.
2. Evaluate for common medical conditions associated with nausea and vomiting (e.g., gastroenteritis,
gastroparesis, pregnancy, postsurgical, or malignancy).
3. Evaluate for potential drug-induced causes (e.g., opioids or antibiotics). Assess for information
regarding all current medications, including OTC agents and any nutritional supplements, with interest
focused on agents that may induce nausea and/or vomiting.
4. Evaluate nutritional intake and typical diet patterns, with interest focused on foods or temporal relation
of food consumption to nausea and vomiting.
5. Laboratory tests, diagnostic imaging, or additional studies may be necessary, depending on findings.

E. Treatment (Domain 1, Task 3,4,6,7)


1. Identify and remove associated cause when possible. If medication is a possible cause, consider
switching or removing the agent.
2. Nonpharmacologic management
a. Dietary modifications
b. Psychological/behavioral interventions (e.g., cognitive distraction, guided imagery, relaxation,
biofeedback acupuncture)
c. Acupressure wristband (Sea-Band); used for motion sickness. Nondrug option for pregnant women
and children
3. Correct fluid, electrolyte, or nutritional deficiencies.
4. Pharmacologic therapies (Table 27)
a. First-line drugs for general disease are phenothiazines and serotonin antagonists.
i. Often used empirically when exact cause is unidentified
ii. Serotonin antagonists have become the primary treatment because of their diffuse blockade
and minimal adverse effects.
iii. If adequate response is not achieved, may require several drugs with distinct mechanisms and/
or different routes of administration to achieve optimal efficacy
b. Response to therapy varies, likely because of various clinical signals and their distinct receptor-
type involvement. Therefore, therapy should be selected according to the likely clinical stimuli and
the associated neuronal pathways.
i. For example: Motion sickness
(a) Proposed mechanism of nausea and vomiting associated with motion sickness: Histamine
and muscarinic receptors within the vestibular area are activated, sending signals to the
vomiting center to induce emesis.
(b) Therapy is directed at inhibiting these histaminergic and muscarinic signals (i.e.,
appropriate therapy would be histamine or muscarinic antagonists).
ii. Examples of clinical conditions and the most appropriate therapy
(a) Motion sickness: Antihistamine, scopolamine
(b) Gastroparesis: Metoclopramide
(c) Postoperative: Serotonin antagonist, scopolamine
(d) Pregnancy: Pyridoxine, antihistamines
(e) Chemotherapy induced: Serotonin antagonist, phenothiazines, neurokinin inhibitors, and
others. Selection of therapy is based on the emetic risk of chemotherapy agents.

ACCP Updates in Therapeutics® 2016: Ambulatory Care Pharmacy Preparatory Review and Recertification Course

1-610
Gastrointestinal Disorders

Table 27. Summary of Select Antiemeticsa


Most Routes of
Drug Class Drug Names Key Adverse Effects
Effective In Administration
Cyclizine (Marezine)
Dimenhydrinate (Dramamine) Migraine
Anticholinergic,
Diphenhydramine (Benadryl) Motion
Oral confusion, dizziness,
H1RAs Doxylamine-pyridoxine sickness
fatigue, sedation,
(Diclegis) Vertigo
tremors
Doxylamine (Unisom)
Meclizine (Antivert)
Migraine
Motion
Oral
Promethazine (Phenergan) sickness Anticholinergic, EPS,
Phenothiazines Rectal
Prochlorperazine (Compazine) Vertigo injection site, sedation
Parental
CINV
PONV
Agitation, QTc
Droperidol Oral
Butyrophenones prolongation,
Haloperidol Parenteral
restlessness, sedation
Ondansetron (Zofran, Zuplenz)
Serotonin-3
Granisetron (Kytril) CINV Oral Headache, diarrhea,
receptor
Palonosetron (Aloxi) PONV Parenteral fatigue
antagonists
Dolasetron (Anzemet)
Domperidone (Motilium) Oral Fatigue, EPS,
Prokinetics Gastroparesis
Metoclopramide (Reglan) Parenteral hyperprolactinemia
Phosphorated
Phosphoric acid/dextrose/fruc- Flatulence, abdominal
carbohydrate Nausea Oral
tose (Emetrol) pain, diarrhea
solution
Ataxia, dizziness,
Cannabinoid Dronabinol (Marinol) CINV Oral hypotension, euphoria,
sedation
Neurokinin-1 Aprepitant, fosaprepitant Fatigue, hiccups,
CINV Oral
receptor antagonist (Emend), rolapitant (Varubi) dyspepsia
Blurred vision,
Motion
Antimuscarinic Scopolamine (Transderm Scop) Transdermal somnolence,
sickness
xerostomia
Information in table is not all-inclusive. Please see prescribing guidelines for complete information.
a

CINV = chemotherapy-induced nausea and vomiting; EPS = extrapyramidal symptom; PONV = postoperative nausea and vomiting; QTc = corrected QT
interval.

ACCP Updates in Therapeutics® 2016: Ambulatory Care Pharmacy Preparatory Review and Recertification Course

1-611
Gastrointestinal Disorders

IX.  IRRITABLE BOWEL SYNDROME

A. Irritable Bowel Syndrome (Am J Gastroenterol 2014;109:S2-26)


1. Defined (according to Rome III criteria) as recurrent abdominal pain and/or discomfort at least 3 days/
month in the past 3 months associated with two or more of the following:
a. Improvement with defecation
b. Onset associated with changes in frequency of stool
c. Onset associated with a change in appearance/form of stool
2. Chronic in that symptom onset must occur at least 6 months before diagnosis and the above criteria
should be fulfilled for at least 3 months.
3. Depending on bowel habits, IBS can be categorized as one of these subtypes (individuals may alternate
between subtypes):
a. Constipation-prominent IBS (IBS-C)
b. Diarrhea-prominent IBS (IBS-D)
c. Mixed IBS (IBS-M)
d. Unclassified (IBS-U)
4. Comorbid psychiatric disorders such as anxiety, somatization, and symptom-related fears are also
common and likely contribute to the reduced quality of life and increased health care costs for
individuals with IBS.

B. Pathophysiology (Domain 1, Task 2)


1. Uncertain, but likely to involve abnormal CNS processing and altered GI motility and secretion.
Possible contributing factors include genetics, motility factors, inflammation, chronic infections, and
stress and other psychological factors.
2. Prevalence is 5%–10% worldwide; 10%–20% in North America, Europe
a. IBS-D and IBS-M most common
b. More common in women, individuals younger than 50 years, and those of lower socioeconomic status

C. Symptoms
1. Usually constipation (IBS-C) or diarrhea (IBS-D) predominant; may be mixture (IBS-M) of constipation
and diarrhea or unclassifiable (IBS-U). In addition, those with IBS may alternate between subtypes.
2. Abdominal pain is common with all subtypes. Other typical symptoms may include bloating,
distension, and urgency. Often, individuals do not seek medical care until pain becomes severe.

D. Diagnosis (Domain 1, Task 1; Domain 4, Task 3)


1. The Manning criteria (1978) and the Kruis score (iteration to improve on Manning criteria) were used
historically. The Rome criteria replaced these (1990, updated in 1999 and 2006). Use of the Rome criteria
can guide entry into clinical trials; is useful for diagnosis, yet not universally adopted by clinicians
2. Historically, IBS was a diagnosis of exclusion. Several costly procedures were performed to rule out
other differential diagnoses before diagnosis. Current recommendations decrease the emphasis on
procedures if symptoms fit the criteria.
3. Although psychological factors are often associated with IBS, they are not a mandatory component of
the diagnosis.
4. According to the guidelines, a correct IBS diagnosis can be made in patients who fulfill the IBS
definition and have no alarm features (see Box 7).

ACCP Updates in Therapeutics® 2016: Ambulatory Care Pharmacy Preparatory Review and Recertification Course

1-612
Gastrointestinal Disorders

Box 7. Irritable Bowel Syndrome Alarm Features


Family history of colorectal cancer, IBD, and celiac sprue
Fecal soilage
Fever
Iron-deficiency anemia
Nocturnal symptoms
Onset after age 40
Rectal bleeding
Weight loss
IBD = inflammatory bowel disease.

5. The guidelines recommend the following regarding diagnostic testing:


a. No routine diagnostic testing is recommended for those with typical IBS symptoms and no
alarm features.
b. Colonic imaging (to exclude organic disease): Recommended in patients with IBS symptoms with
alarm features or those older than 50 years (for colorectal cancer screening). Random mucosal biopsies
should be considered to rule out microscopic colitis in patients with IBS-D who undergo a colonoscopy.
c. Screening for celiac disease in those with IBS-C and IBS-D
d. Lactose intolerance; consider using the lactose hydrogen breath test
e. SIBO (small intestinal bacterial overgrowth); role in IBS is unclear and controversial; to date,
no available gold standard to diagnose small intestinal bacterial overgrowth; thus, testing not
recommended
f. Food allergy testing: Data analyses do not suggest that specific types of food exacerbate IBS;
thus, testing is not recommended currently. Many patients believe food allergies contribute to
symptoms, but studies have not confirmed this.

E. Treatment (Domain 1, Task 3,4,6,7)


1. Initiate when symptoms reduce functional status and affect quality of life.
2. Options include a mixture of pharmacologic and psychological strategies; some studies show a
combination approach increases efficacy.
3. Psychological therapies include cognitive behavioral therapy, dynamic psychotherapy, and gut-directed
hypnosis. Evidence supports improved efficacy regardless of the psychological approach used; however,
extent of benefit remains to be quantified.
4. Diet: Patients often think certain foods can trigger IBS symptoms. Data are limited regarding the role
of diet in the management of IBS. Patient reports of improvement in symptom when following a gluten-
free or low-carbohydrate diet may suggest that specific diets are beneficial.
5. Fiber: Role in the management of IBS remains split. Insoluble fiber may exacerbate symptoms of
bloating, distension, flatulence, and cramping (especially if not introduced gradually), whereas soluble
fiber may provide relief in chronic constipation.
6. Microbiota modification (i.e., probiotics, prebiotics, and antibiotics). It has been suggested that gut
bacteria play a role in IBS. According to the guidelines, although data are insufficient to provide
conclusive recommendations on their use, the data suggest a beneficial effect with bloating and flatulence.
The antibiotic rifaximin has shown improvement in non-constipation IBS and is well tolerated.
7. Pharmacologic therapy (see Table 28)
a. Selection is based on predominant IBS symptoms and severity.
b. Goal is to normalize bowel habits and/or decrease abdominal pain.

ACCP Updates in Therapeutics® 2016: Ambulatory Care Pharmacy Preparatory Review and Recertification Course

1-613
Gastrointestinal Disorders

c. Therapy for constipation (IBS-C subtypes) (also see Table 25 in Constipation Section)
i. Laxatives
(a) Psyllium: Associated with gas and bloating because of bacterial fermentation in the
colon; gradual titration/tapering recommended
(b) Calcium polycarbophil (bulk forming): Semisynthetic bulking agent; less susceptible to
fermentation but less evidence; preferred to placebo in one study
(c) Polyethylene glycol (MiraLAX): Reports show increased stool frequency and improved
consistency, with no improvement in pain or overall symptom relief.
(d) Stimulant laxatives not recommended; may worsen abdominal pain
ii. Linaclotide (Linzess): Guanylate cyclase-C agonist
iii. Lubiprostone (Amitiza): Chloride channel activator
iv. Tegaserod (Zelnorm): Serotonin-4 partial agonist
(a) FDA approved for emergency use in IBS-C
(b) Common adverse events: Headache, flatulence, nausea, diarrhea, and CV events
(c) Removed from market in March 2007; reintroduced in July 2007 by restrictive access
program in emergency situations; for women without known or preexisting cardiac
disease or risk factors
d. Therapy for diarrhea (IBS-D subtype) (also see Table 23 in Diarrhea section)
i. Loperamide (Imodium): Reduces stooling frequency but has no overall impact on pain or
other IBS symptoms. Evidence is insufficient to recommend its use.
ii. Alosetron (Lotronex): Serotonin-3 antagonist
(a) FDA approved for IBS-D
(b) Associated with ischemic colitis (rare)
(c) Use is restricted to women with severe IBS-D who are unresponsive to other medications.
iii. Eluxadoline (Viberzi): Locally acting mu-opioid receptor agonist and delta opioid receptor
antagonist
(a) FDA approved for IBS-D in 2015; reduced abdominal pain and improved stool
consistency versus placebo over 26 weeks
(b) Adverse effects include constipation, nausea, and abdominal pain. The most serious
known risk is a spasm in the sphincter of Oddi, the smooth muscle that surrounds the end
of the common bile and pancreatic ducts, which can result in pancreatitis. Do not use in
patients with a history of bile duct obstruction, pancreatitis, severe liver impairment, or
severe constipation or in patients who drink more than three alcoholic beverages per day.
e. Therapy for bloating (any IBS subtype)
i. Rifaximin (Xifaxan): Nonabsorbable antibiotic: 400 mg by mouth three times daily for 10–14 days
ii. Probiotics
(a) Role in IBS is unclear; some studies suggest improvement in overall IBS-related symptoms;
but guidelines state that evidence is insufficient to recommend probiotics in CIC.
(b) Lactobacillus alone was not effective; current guidelines support the use of probiotics
containing Bifidobacterium infantis.
(c) Consider using probiotics because of their favorable adverse effect profile, especially
in those without symptom improvement from other therapies (Pharmacotherapy
2008;28:496-505).

ACCP Updates in Therapeutics® 2016: Ambulatory Care Pharmacy Preparatory Review and Recertification Course

1-614
Gastrointestinal Disorders

f. Therapy for pain (any IBS subtype)


i. Antispasmodics: May provide short-term relief of abdominal pain associated with IBS.
Thought to work directly on smooth muscles and on cholinergic receptors
(a) Dicyclomine (Bentyl) and hyoscyamine (Levsin, Levsin SL). Effective; use limited by
anticholinergic adverse effects
(b) Peppermint oil: Limited data suggest functions by relaxing smooth muscles. Small
studies; show peppermint oil is superior to placebo in improving IBS-related symptoms
with no greater adverse effects
ii. Antidepressants
(a) Low-dose tricyclic antidepressants and selective serotonin reuptake inhibitors are more
effective than placebo at relieving global IBS symptoms and may decrease abdominal pain.
(b) Data in IBS population are minimal. There are no head-to-head comparator trials,
but selective serotonin reuptake inhibitors are often preferred because of their lack of
anticholinergic effects, different adverse effect profile, and different effects on the GI tract.
(c) Unclear whether lessening of depression or anxiety is related to associated benefit with
these drugs

Table 28. Pharmacotherapy Options for IBS According to Symptom Typea


Symptom Possible Therapy
Antidepressants
Laxatives (non-stimulant)
Constipation
Linaclotide (Linzess)
Lubiprostone (Amitiza)
Alosetron (Lotronex)
Antidepressants
Antispasmodics
Diarrhea
Loperamide (Imodium)
Probiotics
Rifaximin (Xifaxan)
Rifaximin (Xifaxan)
Bloating
Probiotics
Antispasmodics
Pain
Antidepressants (TCAs and SSRIs)
Information in table is not all-inclusive. Please see prescribing guidelines for complete information.
a

IBS = irritable bowel syndrome; SSRI = selective serotonin reuptake inhibitors; TCA = tricyclic antidepressants.

g. Herbal therapies and acupuncture are not recommended; additional information is necessary.
h. Emerging therapies are under development for IBS; it is hoped that they will offer additional
treatment options with fewer adverse effects to address symptoms related to IBS.

F. Role of the Pharmacist (Domain 1, Task 5; Domain 2, Task 1,5; Domain 5, Task 2)
1. Because of their accessibility, pharmacists are likely to be the first contact for individuals with IBS-related
symptoms in the ambulatory care setting. Pharmacists can assist with selecting appropriate OTC empiric
therapy and provide education regarding the effective and safe use of prescription IBS therapies.
2. Pharmacists should also be able to identify individuals with alarm features (see Box 7) and those using
OTC therapies for more than 2 weeks, requiring referral to a GI specialist.

ACCP Updates in Therapeutics® 2016: Ambulatory Care Pharmacy Preparatory Review and Recertification Course

1-615
Gastrointestinal Disorders

X.  INFLAMMATORY BOWEL DISEASE

A. Definition: IBD is a chronic disorder consisting of two conditions: ulcerative colitis (UC) and CD, which
differ with respect to the location and extent of inflammation within the GI tract. Indeterminate colitis is
when UC and CD cannot be differentiated.

B. Pathophysiology (Domain 1, Task 2)


1. Exact inflammatory cause is unknown; thought to involve the same factors for both conditions
2. Immunologic, genetic, allergic, infectious, metabolic, psychosomatic, environmental, and dietary
elements thought to be involved, most likely in combination
a. NSAIDs: Believed to disrupt epithelial barrier
b. Cigarette smoking: Seems to be protective against UC, but can worsen patients with CD

C. Clinical Presentation
1. Course of disease is characterized by alternating periods of active and inactive illness.
2. Presentation varies according to the type of IBD and location of disease; however, some signs seem to
overlap with both conditions, including abdominal tenderness or pain, diarrhea, rectal bleeding, and
weight loss.
a. Ulcerative colitis
i. Symptoms: Diarrhea, rectal bleeding, and abdominal pain. Blood in the stool and tenesmus
more likely with UC than with CD
ii. Disease location is usually limited to the rectum and colon. Subtyped as distal or extensive
disease according to its proximity to splenic flexure
(a) Proctitis: Confined to the rectum
(b) Distal or left sided: Distal to splenic flexure, encompassing the descending colon
(c) Extensive: Proximal to splenic flexure
(d) In rare cases, disease may involve terminal ileus; this is called backwash ileitis.
iii. Disease pattern is continuous.
iv. Typically superficial, not extending below submucosal layer of GI tract
v. Local complications may include hemorrhoids, anal fissures, or perirectal abscesses. Toxic
megacolon is a major complication that can occur.
b. CD
i. Symptoms: Abdominal pain/tenderness/mass, cachexia, diarrhea, pallor, perianal fissures,
fistula, fever, rectal bleeding, hematochezia, ulcers, weight loss
ii. Disease location: Can occur in any part of the GI tract, with terminal ileum and colon being
the most common sites
iii. Disease pattern is considered discontinuous because likely to have segments of diseased
bowel separated by areas of normal bowel (e.g., “cobblestone” appearance)
iv. Inflammation extends deeper into the layers of the GI tract than in UC disease, leading to
fissures, fistulas, abscesses, and strictures, which can lead to obstruction.
v. Malabsorption is common.
3. Extraintestinal manifestations with UC and CD: Primarily caused by systemic inflammation; may
include CV, dermatologic, hepatobiliary, joint, neurologic, ocular, and pulmonary abnormalities

ACCP Updates in Therapeutics® 2016: Ambulatory Care Pharmacy Preparatory Review and Recertification Course

1-616
Gastrointestinal Disorders

D. Diagnosis (Domain 1, Task 1; Domain 4, Task 3)


1. IBD is diagnosed on the basis of clinical symptoms and is supported by appropriate laboratory, endoscopic,
and imaging findings. Overlapping features with other GI disorders may make diagnosis difficult. The
sequence of performing diagnostic evaluations depends on the presenting signs and symptoms.
2. Laboratory tests may include hemoglobin/hematocrit, stool studies, and tests to confirm inflammation
(e.g., presence of fecal leukocytes, erythrocyte sedimentation rate, or C-reactive protein).
3. Endoscopic evaluation is performed to confirm the presence of inflammation, differentiate between
UC and CD, determine the disease location, and classify the extent of disease. The type of evaluation
performed (i.e., upper endoscopy, colonoscopy, or proctosigmoidoscopy) depends on signs and
symptoms. Histologic findings from biopsy tissue may assist with the differential diagnosis and help
determine the depth of tissue involvement.
4. Video capsule endoscopy, a noninvasive method, has proved useful in IBD diagnosis; however, there
is a risk of capsule retention within strictures. Thus, imaging studies are recommended before use to
assess for strictures. More recent technology uses materials that dissolve with time; thus, surgery to
remove retained capsules is less likely.
5. Imaging studies (e.g., ultrasonography, CT, or MRI) are considered in some cases; assist with
establishing disease location and identifying intra-abdominal complications
6. Additional considerations
a. In patient interview, should inquire about potential exacerbating factors of IBD (e.g., NSAID use
and cigarette smoking)
b. Appropriate workup should be performed to rule out possible infectious etiologies (e.g., to
distinguish IBD from infectious colitis).
c. Genetic testing for CD is not recommended currently according to the guidelines; however, this
may be important in the future.
7. Disease severity
a. Severity is used to guide therapy. Many scales have been developed to assess the severity of IBD,
which typically include both subjective and objective values.
b. For UC, the Truelove and Witts severity index and the Sutherland index are commonly used. The
guidelines and the Truelove and Witts severity index classify the severity of UC as mild, moderate,
severe, or fulminant, as summarized in Table 29.

Table 29. Classification of Ulcerative Colitis Disease Severitya,b


< 4 stools/day ± blood
Mild No systemic signs of toxicity
Normal ESR result
> 4 stools/day
Moderate
Minimal signs of systemic toxicity (e.g., fever)
≥ 6 stools/day + blood
Evidence of systemic toxicity (e.g., fever [temperature > 37.8°C], tachycardia > 90 beats/
Severe
min, anemia < 10 g/dL, elevated ESR > 30 mm/hr)
Weight loss
> 10 stools/day with continuous bleeding
May require blood transfusions
Fulminant Abdominal distension/tenderness
Evidence of systemic toxicity
Colonic dilation
Based on the Truelove and Witts severity index.
a

b
Systemic toxicity = fever, tachycardia, anemia, or elevated erythrocyte sedimentation rate.
ESR = erythrocyte sedimentation rate.

ACCP Updates in Therapeutics® 2016: Ambulatory Care Pharmacy Preparatory Review and Recertification Course

1-617
Gastrointestinal Disorders

c. CD severity is classified as mild to moderate, moderate to severe, or severe to fulminant. These


categories incorporate the CDAI (Crohn’s Disease Activity Index), a tool often used in the
research setting to quantify patients’ symptoms. A score less than 150 is considered asymptomatic
remission. Table 30 summarizes the criteria for each category.

Table 30. Classification of Crohn Disease Severitya


Ambulatory
No signs of dehydration
Mild to moderate
No signs of systemic toxicity
(CDAI score 150–220)
No abdominal tenderness, mass
No obstruction
Failure to respond to treatment for mild to moderate disease
Moderate to severe Fever, abdominal pain/tenderness, vomiting, weight loss
(CDAI score 220–450) Anemia
Intestinal obstruction
Persistent symptoms despite outpatient therapy
Severe to fulminant Abdominal pain, rebound tenderness
(CDAI score > 450) High temperature, persistent vomiting
Possible obstruction, abscess
Online calculators are available for score determination based on signs and symptoms.
a

CDAI = Crohn’s Disease Activity Index.

E. Management (Domain 1, Task 3,4,6,7)


1. Pharmacologic therapy is used for symptom reduction and maintenance to prevent relapse; there is no
known cure for IBD.
2. Goals of IBD therapy
a. Acute disease: Eliminate acute disease-related symptoms, induce remission, and improve quality of life.
b. Disease in remission: Maintain remission, reduce need for long-term corticosteroid use, and
minimize risk of colorectal cancer.
c. The primary goal of therapy focuses on symptom reduction, but mucosal healing is also a goal.
Currently, a standard process to evaluate mucosal healing as an end point is lacking.
3. Recommendations for medical management depend on disease location, severity, and associated
complications (i.e., intestinal and extraintestinal).
a. Induction therapies are initiated to control acute symptoms. On symptom control, maintenance
therapy is implemented. If symptom control is not achieved, a different treatment option should be
used and continued until symptom improvement and implementation of maintenance therapy.
b. Maintenance therapies are typically administered once an acute attack is under control; this is
especially necessary in patients with extensive disease or a history of disease relapse. Therapies
should be tapered to the lowest effective dose to minimize adverse effects.
4. Treatment approaches (Domain 1, Task 2, 5)
a. Step-up: This is the current practice, which involves initiating therapy with standard, conventional agents
(e.g., aminosalicylates, topical corticosteroids). Immunomodulators and biologic agents are reserved for
maintenance therapy and for those whose condition does not respond to therapy or who relapse.
b. Top-down: Involves early therapy with immunomodulators or biologic agents to achieve symptom
control and then a transition to standard, conventional agents. The rationale is that early aggressive
therapy may result in mucosal healing and prevent complications and disease progression.

ACCP Updates in Therapeutics® 2016: Ambulatory Care Pharmacy Preparatory Review and Recertification Course

1-618
Gastrointestinal Disorders

5. Medical management of UC (Am J Gastroenterol 2011;106:S2-25; Am J Gastroenterol 2010;105:501-23)


a. Clinical decision support tool (http://campaigns.gastro.org/algorithms/UlcerativeColitis/)
b. Tool provides algorithms for identification, assessment, and initial medical treatment for CD.
c. Steps according to the algorithm include the following (see website for details of each step):
i. Assess inflammatory status.
ii. Assess comorbidities and disease- and therapy-related complications.
iii. Stratify according to colectomy risk (low vs. high)
(a) Low risk: Inductive and maintenance therapy
(b) High risk: Identify patient requiring hospitalization.
(1) Outpatient: Inductive and maintenance therapy, or therapy for high-risk outpatient
not in remission
(2) Inpatient: Inductive and maintenance therapy
iv. Use the care pathway, or review a summary of the medical management from the guidelines
(provided in Table 31).

Table 31. Summary of Medical Management Options for Ulcerative Colitis Based on Disease Severitya
Disease Severity
Category of
Mild to Moderate Disease
Therapy Severe
Distal Extensive
Standard therapyb: Standard therapyb: Standard therapyb:
-Topical mesalamine -Oral sulfasalazine or -Not hospitalized
-Oral sulfasalazine or aminosalicylate ± topical -Infliximab, adalimumab,
mesalamine mesalamine golimumab, vedolizumab
-Topical corticosteroids -Budesonide ER -Oral aminosalicylate if
-Topical + oral Additional optionsc: previously taken
aminosalicylate -Oral corticosteroids Hospitalized
Induction
Additional optionsc: -Thiopurines (when -IV corticosteroid 7–10 days
Oral corticosteroids IV corticosteroids not -add IV cyclosporine (if no
-Infliximab or adalimumab indicated) response after 3–5 days);
or golimumab -Infliximab, adalimumab, combination with thiopurine
vedolizumab recommended
-Oral aminosalicylate if
previously taking
Standard therapyb: Standard therapyb: Standard therapyb:
-Topical mesalamine -Oral sulfasalazine or -In general: Continue what
-Oral sulfasalazine or aminosalicylate provided symptom improvement
aminosalicylate -Thiopurines -Corticosteroids – Taper to lowest
-Topical + oral -Infliximab or adalimumab effective dose; cyclosporine
Maintenance
aminosalicylate or golimumab – Transition to oral; consider
Additional optionsc: thiopurine therapy as well as
-Thiopurines infliximab, adalimumab, or
-Or adalimumab or golimumab
golimumab
Table provides overall summary based on guidelines; management is variable and should be tailored to the individual patient.
a

b
Standard therapy includes options typically used as first-line and conventional therapies; additional information provided within the chapter text.
Additional options include options for those with refractory disease or intolerant of standard therapies; additional information provided within the chapter text.
c

ER = extended release; IV = intravenous.

ACCP Updates in Therapeutics® 2016: Ambulatory Care Pharmacy Preparatory Review and Recertification Course

1-619
Gastrointestinal Disorders

d. Mild to moderate distal disease (defined by the guidelines as a disease that is located distal to
splenic flexure, affecting the descending colon)
i. Aminosalicylates are first-line therapy. Topical (enema or suppository) aminosalicylates are
preferred because they have proven more efficacious than oral aminosalicylates or topical
corticosteroids. Select formulation according to disease location: Suppositories (proctitis), enemas
(distal disease)
ii. If refractory, combination therapy with an oral and topical aminosalicylate is superior to
either administered alone in induction and maintenance phases.
iii. Maintenance therapy
(a) If refractory to standard therapies administered at full dose: Consider thiopurines
(azathioprine, mercaptopurine); infliximab, adalimumab, or golimumab may be considered.
(b) Topical corticosteroids (hydrocortisone or budesonide) not effective for maintenance
e. Mild to moderate extensive disease (defined in the guidelines as a disease proximal to splenic
flexure, encompassing transverse and ascending colon)
i. Induction therapy
(a) Refractory to oral plus topical aminosalicylates: Oral corticosteroids, budesonide
(b) Corticosteroid refractory: Thiopurines for those who can take oral medications. If
acutely ill and requiring intravenous therapy or condition not responding to steroids
plus azathioprine or mercaptopurine: Infliximab or adalimumab or golimumab or
vedolizumab (for moderate to severe disease)
ii. Maintenance therapy
(a) Thiopurines as a steroid-sparing option or for those whose disease is inadequately
controlled by aminosalicylates
(b) Chronic corticosteroid therapy not recommended
f. Severe disease
i. Induction therapy for those with disease refractory to oral/topical aminosalicylates or
corticosteroids
ii. Anti-TNFs if refractory to maximal oral therapy with aminosalicylates, prednisone, and
topical formulations and not hospitalized
iii. Antibiotics: Metronidazole or ciprofloxacin has been used; however, data regarding efficacy
are inconclusive.
iv. Colectomy is last line when all other options have failed.
v. Toxic megacolon: Bowel decompression and systemic antibiotics; colectomy may be necessary
vi. Pouchitis (idiopathic inflammation after ileoanal anastomosis procedure): Antibiotic
(e.g., metronidazole, ciprofloxacin) therapy
6. Medical management of CD (Gastroenterology 2013;145:1459-63; Am J Gastroenterol 2011;106:S2-25;
Am J Gastroenterol 2009;104:465-83)
a. Clinical decision support tool (http://campaigns.gastro.org/algorithms/IBDCarePathway/)
b. Tool provides algorithms for identification, assessment, and initial medical treatment for CD.
c. Steps according to the algorithm include the following (see website for details of each step):
i. Assess inflammatory status.
ii. Assess comorbidities and disease- and therapy-related complications.
iii. Assess current and prior disease burden, and categorize the patient as having low- or
moderate/high-risk disease (see Table 32).

ACCP Updates in Therapeutics® 2016: Ambulatory Care Pharmacy Preparatory Review and Recertification Course

1-620
Gastrointestinal Disorders

Table 32. Crohn Disease Burden Risk Assessment


Low Moderate/High
Age at initial diagnosis, yr Older than 30 Younger than 30
Anatomic involvement Limited Extensive
Perianal and/or severe rectal disease No Yes
Ulcers Superficial Deep
Prior surgical resection No Yes
Stricturing and/or penetrating behavior No Yes
Adapted from: American Gastroenterological Association. AGA Institute Guidelines for the Identification, Assessment and Initial Medical Treatment in
Crohn’s Disease. Available at http://campaigns.gastro.org/algorithms/IBDCarePathway/. Accessed October 15, 2015.

iv. Treatment – Therapy decisions are determined by risk assessment and whether disease is
active or in remission (see AGA guidelines [Gastroenterology 2013;145:1459-63] and clinical
decision support tool website [http://campaigns.gastro.org/algorithms/IBDCarePathway/]).
(a) Initial treatment (i.e., induction)
(1) Low risk
(A) Diffuse or descending colon with none to minimal systemic symptoms:
Prednisone taper with or without azathioprine
(B) Ileum and/or proximal colon with none to minimal systemic symptoms
• Budesonide with or without azathioprine
• Prednisone taper with or without azathioprine
(2) Moderate/high risk
(A) Anti-TNF monotherapy over no therapy or thiopurine monotherapy
(B) Anti-TNF and thiopurine combination therapy over using either agent as
monotherapy
(C) Methotrexate for those who cannot tolerate thiopurine plus anti-TNF
(b) In-remission treatment (i.e., maintenance)
(1) Low risk
(A) Discontinue therapy and observe; however, high chance of relapse within first year
(B) Budesonide delays time to relapse by 114 days; however, benefits are lost after 1 year.
(C) Immunosuppressive therapy (azathioprine, mercaptopurine, and methotrexate)
effective to maintain steroid-induced remission. Long-term exposure associated
with rare risk of infection and lymphoma
(2) Moderate/high risk (therapy typically selected according to type of therapy that was
required to induce remission)
(A) Steroids required to induce remission
• Taper steroids to lowest effective dose.
• Use immunomodulator over no immunomodulator.
• Use anti-TNF with or without thiopurine over no anti-TNF.
• If remission does not endure for at least 6 months, consider combination
therapy with immunosuppressant plus anti-TNF. Combination therapy offers
improved efficacy to anti-TNF monotherapy.
(B) Anti-TNF or anti-TNF plus thiopurine required to induce remission – Use anti-
TNF with or without thiopurine over no anti-TNF.
(c) Not-in-remission treatment
(1) Low risk
(A) Immunosuppression
(B) Assess drug concentrations.
(C) Consider anti-TNF therapies.

ACCP Updates in Therapeutics® 2016: Ambulatory Care Pharmacy Preparatory Review and Recertification Course

1-621
Gastrointestinal Disorders

(2) Moderate/high risk


(A) Anti-TNF monotherapy over no therapy or thiopurine monotherapy
(B) Anti-TNF plus thiopurine combination therapy over using either agent as
monotherapy. Combination therapy more likely to induce remission, but the
increased benefit of attaining remission versus the risk of increased toxicities
requires further evaluation. The clinical tool provides the following additional
steps for evaluating response.
• If failure to respond, assess drug and TNF antibody concentrations.
§ If low/undetectable drug and antibody concentrations, increase drug dose.
§ If low/undetectable drug concentration and high antibody concentrations,
switch within drug class.
§ If drug concentration therapeutic with low/undetectable antibody
concentrations, recommended to assess inflammation
• Inflammation present:
§ Switch to another drug class.
§ Natalizumab, vedolizumab use reserved for those whose condition does not
respond to any of the above therapies and for those who could not tolerate
first-line therapies or who developed anti-TNF antibodies
• Inflammation not present: Continue drug and look for other causes. For
example, evaluate the need for surgical intervention (e.g., mass, obstruction).
(C) In steroid-responsive disease, could consider induction with steroid and
immunomodulator
(D) Additional supportive therapy may be indicated in specific patient populations.
Intravenous fluids and electrolytes if dehydrated, blood transfusions if anemic
or hemorrhaging, nutritional support in those not achieving nutritional
requirements for 5–7 days
d. Perianal and fistulizing disease (Gut 2014;63:1381-92)
i. Evaluate type of fistula (e.g., simple, complex, multiple), and determine whether abscess is
present to help guide management.
ii. Medical treatment is recommended, but evidence-based data are minimal. Options include
antibiotics, immunomodulators, anti-TNF therapy, and integrin antagonists.
(a) Surgical drainage and antibiotics are first-line therapy.
(b) Anti-TNF agents with or without thiopurines are recommended, especially when fistulas
are not closing with standard therapies (i.e., steroids, antibiotics, and immunomodulators).
iii. Tacrolimus is an option for refractory conditions, together with surgery, which is indicated to
treat complications or disease that is refractory to medical management.
7. Pharmacotherapy
a. Aminosalicylates
i. Sulfasalazine (Azulfidine, Azulfidine Entab)
(a) Prodrug cleaved by gut bacteria into aminosalicylate and sulfapyridine metabolites. Inactive
metabolite (sulfapyridine) believed to be responsible for many of the adverse effects
(b) Effective and inexpensive for those who can tolerate it
(c) Dosing (Table 33)
(d) Products available include immediate-release and enteric-coated formulations.

ACCP Updates in Therapeutics® 2016: Ambulatory Care Pharmacy Preparatory Review and Recertification Course

1-622
Gastrointestinal Disorders

(e) Adverse effects; some may be dose related


(1) Arthralgia, headache, GI, rash, fever, pneumonitis, nausea, hepatotoxicity, bone
marrow toxicity, pancreatitis
(2) Disruption in folate absorption; folic acid supplementation warranted
(3) Avoid in patients with a sulfa allergy.
(f) Place in therapy: Useful for IBD induction and maintenance therapy
ii. Aminosalicylates
(a) Includes mesalamine, olsalazine, and balsalazide
(b) Formulated to contain only the aminosalicylate component; the sulfa moiety and adverse
effects related to sulfapyridine are not an issue
(c) Most patients who cannot tolerate sulfasalazine have improved tolerability with this class.
(d) Dosing varies depending on the product selected (see Table 33)
(e) Route of delivery
(1) Several routes of delivery available, allowing drug delivery throughout the bowel
(2) Drug selection is based on site of delivery, which most closely corresponds to
disease location.
(A) Topical (rectal administration) therapy: Proctitis and left-sided distal disease.
Advise patient of proper administration, and consider recommending at bedtime
to allow maximal drug retention.
• Suppository: Retain for 1–3 hours or longer.
• Enema: Retain all night, about 8 hours.
(B) Oral therapy: Small-bowel or colonic disease
(C) Combination therapy with several formulations is often used. For example,
disease that lacks response to oral therapy may respond to oral and topical
combination therapy.
(f) Place in therapy: Used for induction and maintenance therapy. Often, first-line therapy
and preferred to sulfasalazine because of reduced toxicities. However, sulfasalazine use at
times prevails because of reduced cost.

ACCP Updates in Therapeutics® 2016: Ambulatory Care Pharmacy Preparatory Review and Recertification Course

1-623
Gastrointestinal Disorders

Table 33. Summary of Aminosalicylate Therapy for IBD


Drug Trade Name Formulation Induction Dose Maintenance Dose Site of Action
1–1.5 g three or four 500 mg three or four
Sulfasalazine Azulfidine Tablet Colon
times daily times daily
1000 mg once daily 1000 mg once daily
Canasa Suppository Rectum
at bedtime at bedtime
4 g once daily at 4 g once daily at Rectum and
Rowasa Enema
bedtime bedtime descending colon
800 mg BID
Asacol, Asacol Enteric 800–1600 mg three Terminal ileum
(or)
HD coated times daily and colon
400 mg QID
Mesalamine Delayed Terminal ileum
Lialda 2.4–4.8 g daily 2.4 g once daily
release and colon
Delayed 800 mg three times Terminal ileum
Delzicol 800 mg BID
release daily and colon
Jejunum, ileum,
Extended
Apriso N/A 1.5 g once daily terminal ileum,
release
and colon
Extended Small bowel and
Pentasa 1 g QID 1 g QID
release colon
1–3 g daily in two to
Olsalazine Dipentum Capsule 500 mg BID Colon
four divided doses
2250 mg three times
Colazal Capsule N/A Colon
Balsalazide daily
Giazo Tablet (men) 3.3 g BID N/A Colon
IBD = inflammatory bowel disease; N/A = not applicable.

b. Corticosteroids
i. Rapid onset of action; use for induction therapy; no role in maintenance therapy
ii. Common adverse effects associated with chronic systemic use: Adrenal suppression, cataracts,
glucose intolerance, hypertension, impaired wound healing, and osteoporosis. To reduce
adverse effects, recommended to taper dose as soon as possible and not to use as chronic
maintenance therapy
iii. Dosing varies; available preparations
(a) Oral: Prednisone, prednisolone, and budesonide
(1) Prednisone 20–60 mg/day
(2) Budesonide (Entocort EC, Uceris)
(A) More potent than prednisone
(B) Induction: 9 mg/day
(C) Effective for disease located in ileum or ascending (right-sided disease) colon; less-
systemic effects, so often used when long-term use of prednisone is of concern
(b) Intravenous: Hydrocortisone and methylprednisolone
(c) Topical (rectal): Cortenema, Cortifoam, Anucort-HC, and Proctocort; dose varies
according to product. Commonly used in patients with left-sided distal disease
(d) Place in therapy: Induction therapy for IBD; systemic adverse effects limit long-term use
for maintenance. However, sometimes used when symptom control cannot be achieved or
when patient is severely ill (see medical management section above)

ACCP Updates in Therapeutics® 2016: Ambulatory Care Pharmacy Preparatory Review and Recertification Course

1-624
Gastrointestinal Disorders

c. Immunomodulators – Mechanism of action varies among agents.


i. Thiopurines: Mercaptopurine (Purinethol): 1–1.5 mg/kg/day orally; azathioprine (Imuran):
1–3 mg/kg/day orally
(a) Adverse effects (dose-dependent)
(1) Mercaptopurine: Nausea, allergic reactions, acute pancreatitis, hepatitis, increased
risk of infection, malignancy, and bone marrow suppression
(2) Azathioprine: Nausea, allergic reactions, acute pancreatitis, hepatitis, increased
risk of infection, malignancy, and bone marrow suppression. Thiopurine
methyltransferase (TPMT) assessment should done before initiating therapy.
Individuals inheriting nonfunctional TPMT alleles are at risk of life-threatening
myelosuppression; thus, identifying this subpopulation in advance will allow for
appropriate dose adjustments (Clin Pharmacol Ther 2011;89:387-91).
(3) Long-term use of thiopurines is associated with increased risk (1.5- to 5-fold) of
lymphoma, especially when combined with TNFα inhibitors; however, these agents
continue to be used because benefit seems to outweigh risks.
(b) Place in therapy: Maintenance therapy for IBD. Reserved for those whose condition is not
responding to other therapies and used to spare the use of corticosteroids. After initiating
thiopurine therapy, effects are delayed for weeks to months, so it is recommended to
continue administering other therapies (e.g., aminosalicylates and corticosteroids); this is
why these agents are not recommended as induction therapy.
ii. Methotrexate: 15–25 mg/week intramuscularly or subcutaneously
(a) Adverse effects: Hepatoxicity (increased risk when cumulative dose exceeds 1.5 g),
pneumonitis, increased risk of infection, malignancy, alopecia, stomatitis, and
myelosuppression
(b) Place in therapy: CD: Maintenance; steroid-sparing effect; UC: Not effective
iii. Calcineurin inhibitors
(a) Agents
(1) Cyclosporine (2–4 mg/kg/day): Intravenous administration. Use reserved for those
with severe to fulminant UC disease unresponsive to intravenous corticosteroids and
as an alternative to colectomy
(2) Tacrolimus (0.05 mg/kg twice daily); studied in treatment of fistulas associated with
CD and in disease failing to respond to corticosteroids or infliximab
(b) Adverse effects: Nephrotoxicity, neurotoxicity, hypertension, and electrolyte imbalances
(c) Place in therapy: Induction and maintenance therapy for IBD. Tacrolimus has not been
studied for maintenance.
d. Anti-TNFα therapies (see Table 34)
i. Agents
(a) Infliximab (Remicade): Indicated for both CD and UC moderately to severely active
disease and maintenance therapy; also fistulizing CD
(b) Adalimumab (Humira): Induction and maintenance for moderately to severely active CD
and in patients with UC not responding to conventional therapies
(c) Certolizumab (Cimzia): Indicated for reducing signs and symptoms of CD and
maintaining clinical response in those with moderately to severely active disease who
have had inadequate response to conventional therapy
(d) Golimumab (Simponi): Indicated in moderately to severely active disease in patients with
steroid dependency or those whose conventional therapies have failed (Pursuit study.
Gastroenterology 2014;146:96-109)

ACCP Updates in Therapeutics® 2016: Ambulatory Care Pharmacy Preparatory Review and Recertification Course

1-625
Gastrointestinal Disorders

ii. Predisposition to infection may occur with all the anti-TNF agents; thus, risk-benefit should
be evaluated for those with chronic infections. Before initiating therapy, patients should be
screened for TB, HBV, and HCV. Educate patients to avoid live vaccines during therapy.
iii. Appears not to be associated with increased risk of lymphoma; however, this is yet to be fully
understood because the clinical trial follow-up period may not have been long enough to
determine impact
iv. Place in therapy – Patients whose condition initially responds to therapy but who then become
intolerant or develop anti–TNF-resistant disease are classified as secondary nonresponders.
Options for these patients include the following:
(a) If undetectable antibodies, consider increasing the dose or decreasing dosing frequency
(b) If detectable antibodies, consider switching to an alternative anti-TNFα therapy
e. Anti-a4 integrin receptor antagonists (see Table 34)
i. Agents
(a) Natalizumab (Tysabri): Effective for reducing remission and for maintenance in moderate
to severe CD (outpatients)
(b) Vedolizumab (Entyvio): Effective for inducing and maintaining clinical response and remission,
improving endoscopic appearance of the mucosa, and achieving steroid-free remission
ii. See Table 34 for limitations to use.

ACCP Updates in Therapeutics® 2016: Ambulatory Care Pharmacy Preparatory Review and Recertification Course

1-626
Table 34. Immunomodulator Therapy for Crohn Disease and Ulcerative Colitis
Infliximab Adalimumab Golimumab Certolizumab
Natalizumab (Tysabri) Vedolizumab (Entyvio)
(Remicade) (Humira) (Simponi) (Cimzia)
Chimeric Humanized, Anti-a4 integrin receptor antagonists – Humanized
Mechanism monoclonal TNF Humanized monoclonal TNFα antibody pegylated monoclo- monoclonal antibody that is an integrin receptor
antibody nal TNFα antibody antagonist
Induction: 5 mg/kg Induction:
intravenous infusion Subcutaneous Induction: 200 mg
over 2 hr at weeks injection. Day 1: 160 (two injections)
0, 2, and 6; dose. mg (may administer subcutaneously; Induction: 400 mg Induction and
300 mg intravenously at
Maintenance: 5 mg/ as four injections of then 100 mg subcutaneously maintenance: 300 mg
0, 2, and 6 wk; every 8
kg intravenously 40 mg in 1 day or two (one injection) (as two 200-mg IV infusion over 1 hr
wk thereafter
Dose every 8 wk injections of 40 mg subcutaneously at injections) at weeks every 28 days
daily for 2 consecutive week 2; then 100 0, 2, and 4, and If no benefit observed
Recommended to If no benefit observed
days). Day 15: 80 mg subcutaneously then every 4 wk for after 14 wk, therapy
premedicate with after 12 wk, therapy
mg. Day 29: 40 mg; every 4 wk maintenance should be discontinued
antihistamine, should be discontinued
acetaminophen, maintenance: continue Renal and hepatic

1-627
and/or 40 mg every other dosing not defined
corticosteroids week
Prefilled syringe kit: Intravenous 300 mg natalizumab
Prefilled syringe
Gastrointestinal Disorders

Intravenous powder 40 mg/0.8 mL solution: 50 mg/4 in 15 mL (20 mg/mL) 300 mg vedolizumab in


200 mg/1 mL and
for solution: 100 Subcutaneous solu- mL single-use vial; must a 20-mL single-use vial;
Supplied as lyophilized powder
mg tion: 10 mg/0.2 mL, Subcutaneous be reconstituted before must be reconstituted
for reconstitution:
40 mg/0.8 mL, 20 solution: 50 mg/0.5 infusion before infusion
200 mg
mg/0.4 mL mL, 100 mg/1 mL
Arthralgia, bronchitis,
Infusion reactions, Multifocal
cough, back pain, rash,
increased infection leukoencephalopathy
pruritus, sinusitis,
and malignancy (monitor mental status
nasopharyngitis,
risk, hepatoxicity, Infusion reactions, increased infection and malignancy risk, bone changes), infusion
nausea, pyrexia, upper
ADRs heart failure marrow risk reactions, depression,
respiratory tract
exacerbations, increased infection
infection, fatigue,
bone marrow and malignancy risk,
influenza, oropharyngeal
suppression, bone marrow toxicity,

ACCP Updates in Therapeutics® 2016: Ambulatory Care Pharmacy Preparatory Review and Recertification Course
pain, and pain in
vasculitis hepatotoxicity
extremities
Table 34. Immunomodulator Therapy for Crohn Disease and Ulcerative Colitis (continued)
Infliximab Adalimumab Golimumab Certolizumab
Natalizumab (Tysabri) Vedolizumab (Entyvio)
(Remicade) (Humira) (Simponi) (Cimzia)
Use is contraindicated
in combination
with other
immunosuppressants or
anti-TNFα agents, and
in those with underlying
Contraindicated in No cases of PML
hepatic disease
patients with active have been observed
infection, latent Limited use because in clinical trials; risk
TB (untreated), of rare risk of remains since reported
Warnings/ heart failure, recent progressive multifocal with natalizumab
concerns malignancies, leukoencephalopathy
optic neuritis, (PML) Monitoring for any new
or preexisting or worsening neurologic
Available only signs or symptoms are
demyelinating
through the TOUCH recommended

1-628
disorder
prescribing program;
all prescribers, infusion
centers, and pharmacies
Gastrointestinal Disorders

must be enrolled before


prescribing, distributing,
or infusing the product
Place in CD and UC:
Moderate to severe: Refractory to standard therapies and treatment of fistulas
therapy: — In those with moderately
associated with CD
CD to severely active
Moderately to severely disease who have had
active disease refrac- an inadequate response,
Place in Mild to moderate extensive disease: Refractory to standard tory to, or for those who lost response, were
therapy: therapies — cannot tolerate, standard intolerant of TNFα
UC therapies and in those or immunomodulator
who develop TNFα or dependent on
antibodies corticosteroids

ACCP Updates in Therapeutics® 2016: Ambulatory Care Pharmacy Preparatory Review and Recertification Course
Gastrointestinal Disorders

f. Antibiotics and probiotics


i. Inducing remission
(a) It is thought that IBD triggers a gut antigen, which in turn stimulates an immune
response. Although specific antigens within the gut flora have not been identified, a
potential treatment would target the antigen (i.e., gut flora) in addition to modulation of
immune system. Probiotics are one approach; studies report low efficacy for UC and CD.
(b) According to the guidelines, antibiotics are effective and may be considered for reducing
fistula drainage in CD/infections. Guidelines do not recommend antibiotic use currently.
ii. Preventing relapse: Data analyses show a statistically significant effect of antibiotics in
preventing relapse in CD; however, available evidence is insufficient to recommend use.
Use of antibiotics to prevent relapse in CD is not recommended according to the American
College of Gastroenterology guidelines.

Patient Case

9. A 53-year-old man with CD for the past 12 years is hospitalized because of disease relapse. His colonoscopy
is consistent with moderate to severe disease located in the colon and terminal ileum. He has not responded
to a 5-day course of intravenous methylprednisolone, and on review of the patient’s medical record and a
discussion with him, you determine that he was taking prednisone 40 mg orally once daily for 12 days before
this hospital admission. His severe symptoms continue; his temperature is 101.3°F, with blood pressure 131/81
mm Hg and heart rate 68 beats/minute. He has normal kidney and liver function, and a workup for infectious
disease is negative. Which therapy would be best to initiate?
A. Metronidazole 500 mg orally three times daily.
B. Natalizumab 300 mg intravenously.
C. Azathioprine 100 mg orally once daily.
D. Hydrocortisone 100 mg intravenously every 8 hours.

Special Acknowledgments
Mia Schmiedeskamp-Rahe, Ph.D., Pharm.D., BCPS
Tiffany E. Kaiser, Pharm.D., MSCR, FCCP, BCPS

ACCP Updates in Therapeutics® 2016: Ambulatory Care Pharmacy Preparatory Review and Recertification Course

1-629
Gastrointestinal Disorders

REFERENCES
Gastroesophageal Reflux Disease Chronic Liver Disease
1. Abraham NS, Hlatky MA, Antman EM, et al. ACCF/ 1. Kamath PS, Kim WR. The model for end-stage liver
ACG/AHA 2010 expert consensus document on the disease (MELD). Hepatology 2007;45:797-805.
concomitant use of proton pump inhibitors and thi- 2. Powell LW. Overview: liver disease and trans-
enopyridines: a focused update of the ACCF/ACG/ plantation. J Gastroenterol Hepatol 2009;24(suppl
AHA 2008 expert consensus document on reduc- 3):S97-104.
ing gastrointestinal risks of antiplatelet therapy and
NSAID use. Am J Gastroenterol 2010;105:2533-49. Ascites, Spontaneous Bacterial Peritonitis, and
2. Haag S, Andrews JM, Katelaris PH, et al. Hepatorenal Syndrome
Management of reflux symptoms with over- 1. European Association for the Study of the Liver.
the-counter proton pump inhibitors: issues and EASL clinical practice guidelines on the manage-
proposed guidelines. Digestion 2009;80:226-34. ment of ascites, spontaneous bacterial peritonitis,
3. Katz PO, Gerson LB, Vela MF. Guidelines for the and hepatorenal syndrome in cirrhosis. J Hepatol
diagnosis and management of gastroesophageal 2010;53:397-417.
reflux disease. Am J Gastroenterol 2013;108:308-28. 2. Gordon FD. Ascites. Clin Liver Dis 2012;16:285-99.
4. Thomson AB, Sauve MD, Kassam N, et al. Safety 3. Runyon BA. Management of adult patients with
of the long-term use of proton pump inhibitors. ascites due to cirrhosis: update 2012. Hepatology
World J Gastroenterol 2010;16:2323-30. 2013;1-27.
5. Tutuian R. Adverse effect of drugs on the esopha- 4. Xia HH. Spontaneous bacterial peritonitis. World J
gus. Best Pract Clin Gastroenterol 2010;24:91-7. Gastroenterol 2009;15:1042-9.

Peptic Ulcer Disease Hepatic Encephalopathy


1. ACCF/ACG/AHA 2010 Expert Consensus 1. Bass NM, Mullen KD, Sanyal A, et al. Rifaximin
Document on the Concomitant Use of Proton Pump treatment in hepatic encephalopathy. N Engl J Med
Inhibitors and Thienopyridines: a focused update 2010;362:1071-81.
of the ACCF/ACG/AHA 2008 Expert Consensus 2. Neff G. Factors affecting compliance and persis-
Document on Reducing the Gastrointestinal Risks tence with treatment for hepatic encephalopathy.
of Antiplatelet Therapy and NSAID Use. J Am Coll Pharmacotherapy 2010;30(5 pt 2):22S-27S.
Cardiol 2010;56:2051-66. 3. Neff G. Pharmacoeconomics of hepatic encepha-
2. Chey WD, Wong BC. American College of lopathy. Pharmacotherapy 2010;30(5 pt 2):28S-32S.
Gastroenterology guideline on the management of 4. Neff GW, Kemmer N, Zacharias VC, et al. Analysis
Helicobacter pylori infection. Am J Gastroenterol of hospitalizations comparing rifaximin versus
2007;102:1808-25. lactulose in the management of hepatic encepha-
3. Grad YH, Lipsitch M, Aiello AE. Secular trends in lopathy. Transplant Proc 2006;38:3552-5.
Helicobacter pylori seroprevalence in adults in the 5. Schiano TD. Clinical management of hepatic
United States: evidence for sustained race/ethnic encephalopathy. Pharmacotherapy 2010;30(5 pt
disparities. Am J Epidemiol 2012;175:54-9. 2):10S-15S.
4. Laine L, Jensen DM. Management of patients with 6. Schiano TD. Treatment options hepatic encepha-
ulcer bleeding. Am J Gastroenterol 2012;10:345-60. lopathy. Pharmacotherapy 2010;30(5 pt 2):16S-21S.
5. Lanza FL, Chan FKL, Quigley EMM, et al. 7. Thompson JR. Treatment guidelines for hepatic
Guidelines for the prevention of NSAID-ulcer com- encephalopathy. Pharmacotherapy 2010;30(5 pt 2):
plications. Am J Gastroenterol 2009;104:728-38. 4S-9S.
8. Vilstrup H, Amodio P, Bajaj J et al. Hepatic
Gastrointestinal Bleed encephalopathy in chronic liver disease: 2014
1. Barkun A, Bardou M, Marshall JK, et al. Consensus Practice Guideline by the American Association
recommendations for managing patients with for the Study of Liver Diseases and the European
nonvariceal upper gastrointestinal bleeding. Ann Association for the Study of the Liver. Hepatology
Intern Med 2003;139:843-57. 2014;60:715-35.

ACCP Updates in Therapeutics® 2016: Ambulatory Care Pharmacy Preparatory Review and Recertification Course

1-630
Gastrointestinal Disorders

Portal Hypertension and Varices (ACIP), part II: immunization of adults. MMWR
1. Garcia-Tsao G, Lim J. Management and treatment 2006;55(RR16):1-33.
of patients with cirrhosis and portal hyperten- 3. Centers for Disease Control and Prevention (CDC).
sion: recommendations from the Department of Guidance for evaluation health-care personnel
Veterans Affairs Hepatitis C Resource Center for hepatitis B virus protection and for admin-
Program and the National Hepatitis C Program. istering postexposure management. MMWR
Am J Gastroenterol 2009;104:1802-29. 2013;62(RR10):1-19.
2. Garcia-Tsao G, Sanyal AJ, Grace ND, et al. 4. Centers for Disease Control and Prevention (CDC).
Prevention and management of gastroesophageal Recommendations for identification and public
varices and variceal hemorrhage in cirrhosis. health management of persons with chronic hepati-
Hepatology 2007;46:922-38. tis B virus infection. MMWR 2008;57(RR08):1-20.
5. LeFevre ML. Screening for hepatitis B virus infec-
Hepatitis A tion in nonpregnant adolescents and adults: U.S.
1. Centers for Disease Control and Prevention (CDC). Preventive Services Task Force recommendation
Prevention of hepatitis A through active or passive statement. Ann Intern Med 2014;161:58-66.
immunization: recommendations of the Advisory 6. Lok AS, McMahon BJ. Chronic hepatitis B: update
Committee on Immunization Practices (ACIP). 2009. Hepatology 2009;50:1-36.
MMWR 2006;55(RR07):1-23. 7. McMahon BJ. The natural history of chronic hepa-
2. Centers for Disease Control and Prevention titis B virus infection. Hepatology 2009;49:S45-55.
(CDC). Update prevention of hepatitis A after 8. Scaglione SJ, Lok AS. Effectiveness of hepatitis
exposure to hepatitis A virus and international trav- B treatment in clinical practice. Gastroenterology
elers: updated recommendations of the Advisory 2012;142:1360-8.
Committee on Immunization Practices (ACIP). 9. Sorrell MF, Belongia EA, Cost J, et al. National
MMWR 2007;56:1080-4. Institutes of Health Consensus Development
3. Centers for Disease Control and Prevention. Conference Statement: management of hepatitis B.
In: Hamborsky J, Kroger A, Wolfe S, eds. Ann Intern Med 2009;150:104-10.
Epidemiology and Prevention of Vaccine- 10. Yapali S, Talaat N, Lok AS. Management of hepatitis
Preventable Diseases, 13th ed. Washington, DC: B: our practice and how it relates to the guidelines.
Public Health Foundation, 2015. Available at Clin Gastroenterol Hepatol 2014;12:16-26.
www.cdc.gov/vaccines/pubs/pinkbook/index.
html#chapters. Accessed October 15, 2015. Hepatitis C
4. Fiore AE. Hepatitis A transmitted by food. Clin 1. American Association for the Study of Liver
Infect Dis 2004;38:705-15. Diseases (AASLD) and Infectious Diseases Society
of America (IDSA) Recommendations for Testing,
Hepatitis B Managing, and Treating Hepatitis C. Available at
1. Centers for Disease Control and Prevention (CDC). www.hcvguidelines.org. Accessed October 15, 2015.
A comprehensive immunization strategy to elimi- 2. Centers for Disease Control and Prevention (CDC)
nate transmission of hepatitis B virus infection Testing for HCV infection: an update of guid-
in the United States: recommendations of the ance for clinicians and laboratories. MMWR
Advisory Committee on Immunization Practices 2013;62:362-65.
(ACIP), part I: immunization of infants, children 3. Centers for Disease Control and Prevention (CDC).
and adolescents. MMWR 2005;54(RR16):1-33. Recommendations for the identification of chronic
2. Centers for Disease Control and Prevention (CDC). hepatitis C virus infection among persons born
A comprehensive immunization strategy to elimi- during 1945–1965. MMWR 2012;61(RR04):1-18.
nate transmission of hepatitis B virus infection 4. McHutchison JG, Bacon BR. Chronic hepatitis C:
in the United States: recommendations of the an age wave of disease burden. Am J Manag Care
Advisory Committee on Immunization Practices 2005;11:S286-S295.

ACCP Updates in Therapeutics® 2016: Ambulatory Care Pharmacy Preparatory Review and Recertification Course

1-631
Gastrointestinal Disorders

5. Mohammad RA, Bulloch MN, Chan J, et al. 4. Ramkumar D, Rao SS. Efficacy and safety of
Provision of clinical pharmacist services for traditional medical therapies for chronic consti-
individuals with chronic hepatitis C infection. pation: systematic review. Am J Gastroenterol
Pharmacotherapy 2014;34:1341-54. 2005;100:936-71.

Malabsorptive Syndrome Nausea and Vomiting


1. American Gastroenterological Association (AGA) 1. ACOG Practice Bulletin No. 52. Nausea and vomit-
Institute. Technical review on the diagnosis and ing of pregnancy. Obstet Gynecol 2004;103:803-15.
management of celiac disease. Gastroenterology 2. American Gastroenterological Association
2006;131:1981-2002. (AGA). Technical review on nausea and vomiting.
2. Schulzke JD, Troger H, Amasheh M. Disorders of Gastroenterology 2001;120:263-86.
intestinal absorption and discretion. Best Pract Res 3. Flake ZA, Scalley RD, Bailey AG. Practical
Clin Gastroenterol 2009;23:395-406. selection of antiemetics. Am Fam Physician
3. Shaukat A, Levitt MD, Taylor BC, et al. Systematic 2004;69:1169-76.
review: effective management strategies for lactose
intolerance. Ann Intern Med. 2010;152:797-803. Irritable Bowel Syndrome
4. Tandra S, Chalasani N, Jones DR, et al. 1. Chang L, Lembo A, Sultan S. American
Pharmacokinetic and pharmacodynamic alterations Gastroenterological Association technical
in the Roux-en-Y gastric bypass recipients. Ann Surg review on the pharmacological management
2013;258:262-9. of irritable bowel syndrome. Gastroenterology
2014;147:1149-72.
Diarrhea 2. Ford AC, Moayyedi P, Lacy BE, et al. American
1. American Gastroenterological Association (AGA). College of Gastroenterology monograph on the
Technical review of the evaluation and man- management of irritable bowel syndrome and
agement of chronic diarrhea. Gastroenterology chronic idiopathic constipation. Am J Gastroenterol
1999;116:1464-86. 2014;109:S2-26.
2. Chassany O, Michaux A, Bergmann JF. Drug- 3. Mayer EA. Irritable bowel syndrome. N Engl J
induced diarrhoea. Drug Saf 2000;22:53-72. Med 2008;358:1692-9.
3. Theilman NM, Guerrant RL. Acute infectious 4. Wilhelm SM, Brubaker CM, Varcak EA, et al.
diarrhea. N Engl J Med 2004;350:38-47. Effectiveness of probiotics in the treatment of
4. Thomas PD, Forbes A, Green J, et al. Guidelines irritable bowel syndrome. Pharmacotherapy
for the Investigation of Chronic Diarrhoea, 2nd ed. 2008;28:496-505.
Gut 2003;52(suppl V):V1-15.
Inflammatory Bowel Disease
Constipation 1. AGA Institute Guidelines for the Identification,
1. American College of Gastroenterology Chronic Assessment and Initial Medical Treatment in
Constipation Task Force. An evidence-based Crohn’s Disease. Clinical Decision Support Tool.
approach to the management of chronic consti- 2015.
pation in North America. Am J Gastroenterol 2. American Gastroenterological Association Institute.
2005;100:S1-22. Technical review on the use of thiopurines, methotrex-
2. Bharucha AE, Pemberton JH, Locke GR. American ate, and anti-TNF-a biologic drugs for the induction
Gastroenterological Association technical review and maintenance of remission in inflammatory Crohn’s
on constipation. Gastroenterology 2013;144:218-38. disease. Gastroenterology 2013;145:1464-78.
3. Ford AC, Moayyedi P, Lacy BE, et al. American 3. Gecse KB, Bemelman W, Kamm MA, et al. A global
College of Gastroenterology monograph on the consensus on the classification, diagnosis and multi-
management of irritable bowel syndrome and disciplinary treatment of perianal fistulising Crohn’s
chronic idiopathic constipation. Am J Gastroenerol disease. Gut. 2014;63:1381-92.
2014;109:S2-26.

ACCP Updates in Therapeutics® 2016: Ambulatory Care Pharmacy Preparatory Review and Recertification Course

1-632
Gastrointestinal Disorders

4. Kornbluth A, Sachar DB. Ulcerative colitis prac-


tice guidelines in adults: American College of
Gastroenterology, Practice Parameters Committee.
Am J Gastroenterol 2010;105:501-23.
5. Talley NJ, Abreu MT, Achkar JP, et al. An evidence-
based systematic review on medical therapies for
inflammatory bowel disease. Am J Gastroenterol
2011;106:S2-25.
6. Terdiman JP, Gruss CB, Heidelbauh JJ, et al.
American Gastroenterological Association Institute
guideline on the use of thiopurines, methotrexate,
and anti-TNF-a biologic drugs for the induction and
maintenance of remission in inflammatory Crohn’s
disease. Gastroenterology 2013;145:1459-63.

ACCP Updates in Therapeutics® 2016: Ambulatory Care Pharmacy Preparatory Review and Recertification Course

1-633
Gastrointestinal Disorders

ANSWERS AND EXPLANATIONS TO PATIENT CASES


1. Answer: B rapid results, would not be useful in this instance because it
The patient’s symptoms are consistent with GERD; cannot test for H. pylori eradication.
on-demand therapy and lifestyle modifications have not
been effective. Empiric standard-dose PPI is first-line 4. Answer: A
therapy. Answer A, motility agent, is incorrect because it This patient has symptoms of ascites requiring treatment. A
is not first-line therapy and the patient has no evidence of combination regimen with spironolactone and furosemide
reduced upper GI motility. Answer B is correct; omeprazole is preferred, which should achieve more rapid results
is a PPI, and 20 mg is the standard dose. Prescribing than monotherapy while maintaining normokalemia.
frequency is once daily. Answer C is incorrect; although The initial recommended starting dose of these agents
scheduling famotidine is a better option than the patient’s is in a ratio of 40 mg of furosemide to every 100 mg of
current therapy, it is not first line for GERD symptoms spironolactone. Answer A is correct. This is continued
that are unresponsive to H2RAs. Answer D is incorrect; and increased every 3–5 days if symptoms persist.
although increasing and scheduling famotidine would be Answer B is incorrect; spironolactone monotherapy is not
better than the patient’s current therapy, omeprazole would preferred. Answer C is incorrect because the ratio between
be more effective for this patient’s GERD symptoms. spironolactone and furosemide is reversed. Answer D,
furosemide monotherapy, is not preferred.
2. Answer: D
Pharmacologic treatment options for H. pylori include 5. Answer: D
clarithromycin triple therapy or bismuth quadruple therapy From an assessment of mental status and symptoms
for 14 days. Although efficacy is similar with both options, (asterixis and slurred speech), this patient has stage 2 HE
the best regimen should be selected according to patient- according to the West Haven criteria. Treatment goals for
specific factors. The patient is allergic to penicillin; thus, HE are provision of supportive care, identification and
a regimen that contains amoxicillin is contraindicated. elimination of potential participating factors, and reduction
Answer A is incorrect; amoxicillin is contraindicated in in circulating toxins. Effective pharmacotherapy includes
penicillin-allergic patients, and a 7-day therapy is too short. lactulose and antibiotics (neomycin, metronidazole, or
Answer B is incorrect; although the therapy length is correct rifaximin). Lactulose is first-line therapy according to the
at 14 days, amoxicillin is contraindicated in penicillin- guidelines. Answer A is incorrect; although lactulose is
allergic patients. Answer C is incorrect; metronidazole is an first-line therapy, it should not be dosed as needed. The
appropriate antibiotic for someone with a penicillin allergy; recommended dose is 45 mL/hour until evacuation occurs,
however, treating for only 7 days is too short. Answer D, followed by dose tapering (15–45 mL every 8–12 hours) to
which contains metronidazole and a 14-day therapy, is the achieve 2 or 3 soft stools a day. Additional therapies may
most appropriate option. be given together with lactulose for synergy; however,
this choice is incorrect because of the dose and frequency
3. Answer: B of lactulose administration. Answer B is incorrect;
Non-endoscopic tests (antibody testing, UBT, and FAT) are neomycin is not indicated as a first-line therapy because
performed without an EGD, are noninvasive, are capable of its adverse event potential. In addition, this therapy is
of being performed in an outpatient clinic setting, and can contraindicated in this patient because of his chronic renal
test for H. pylori eradication. Answer A is incorrect; tissue insufficiency. Answer C is incorrect; lactulose is first-line
culture requires an endoscopy, which is invasive, is not done therapy according to the guidelines; however, this dose
in an outpatient setting, and is done to identify infection and is incorrect. Answer D is correct; lactulose is indicated
characterize antimicrobial sensitivities. Answer B is correct; as first-line therapy according to the guidelines, and the
a UBT can document the eradication of an infection and is the stated dose is the recommended dose.
most reliable method; it should be performed at least 4 weeks
after the patient finishes therapy. Answer C is incorrect; RUT
is an endoscopic diagnostic test that requires performing
an EGD and cannot document eradication. Answer D is
incorrect; a serum antibody test, although low in cost with

ACCP Updates in Therapeutics® 2016: Ambulatory Care Pharmacy Preparatory Review and Recertification Course

1-634
Gastrointestinal Disorders

6. Answer: C combination of paritaprevir (150 mg)/ritonavir (100 mg)/


Nonselective β-blockers decrease cardiac output ombitasvir (25 mg) plus twice-daily dosed dasabuvir
(β1-blockade) and reduce portal blood inflow (β2-blockade); (250 mg) and weight-based ribavirin (1000 mg for
thus, they are considered the first-line therapy for the individuals weighing less than 75 kg or 1200 mg for
primary prevention of variceal bleeds. This patient has individuals weighing 75 kg and more) for 12 weeks,
medium varices as visualized EGD; thus, he warrants or (4) sofosbuvir 400 mg and simeprevir 150 mg daily
primary prophylaxis, making Answer D incorrect. Although with or without weight-based ribavirin (1000 mg for
the patient has diabetes, the case states that his disease is individuals weighing less than 75 kg or 1200 mg for
controlled; thus, using this agent is not likely to potentiate individuals weighing 75 kg or more). The current
hypoglycemic episodes or mask hypoglycemic signs and guidelines provide no alternative options for this patient
symptoms. He should therefore receive non-selective population. Regimen selection should be done according
β-blocker prophylaxis, making Answer C incorrect. to individual patient characteristics such as comorbid
Propranolol is a recommended choice, making Answer C conditions, potential for drug-drug interactions with
correct. Atenolol is a selective β-blocker; therefore, it is not concomitant medications, and adverse events. Answer B
an accepted therapy for primary prophylaxis of variceal is correct, the ledipasvir/sofosbuvir combination for 12
bleeding, making Answer A incorrect. Long-acting nitrates weeks is one of the four recommended options. Answer
are not recommended because of increased mortality risk, A is incorrect because ribavirin is not required, and the
making Answer B incorrect. length of therapy should be 12 weeks, not 24 weeks, in
a treatment-naive patient without cirrhosis. Answer C
7. Answer: D is incorrect because when using paritaprevir/ritonavir/
This patient has immune active chronic HBV infection ombitasvir once daily plus dasabuvir twice daily for 12
(HBeAg positive, HBV DNA greater than 20,000 weeks, weight-based ribavirin must also be included as
IU/mL, and AST/ALT elevation), and she has not part of the regimen. Answer D is incorrect; the guidelines
seroconverted HBeAg since the diagnosis 6 months ago. recommend treating for 12 weeks rather than 24 weeks,
She has compensated liver disease. Antiviral treatment but ribavirin must be included in the regimen for a
is indicated, which may be initiated with any of the seven treatment-naive genotype 1a patient without cirrhosis.
approved antiviral medications; however, pegylated
interferon, entecavir, or tenofovir is preferred. Pegylated 9. Answer: B
interferon at 180 mcg/week is a correct treatment option; Although IBD has no known cure, pharmacologic
however, it is not the best choice for an individual with therapy is aimed at symptom reduction and maintenance
a history of depression, making Answer A incorrect. to prevent relapse. The goals of therapy for acute disease
Answer B is incorrect; this is a correct lamivudine dose are to eliminate acute disease-related symptoms, induce
given the patient’s calculated CrCl of 46 mL/minute/1.73 remission, and improve quality of life. The goals of
m2; however, lamivudine is not a recommended first- therapy for disease in remission are to maintain remission,
line therapy because of high resistance rates. Answer reduce need for long-term corticosteroid use, and
C is incorrect; tenofovir is a first-line treatment option minimize risk of colorectal cancer. Recommendations
for this patient; however, this dose is incorrect for her for medical management depend on disease location,
renal function. Answer D is correct; tenofovir is dosed severity, and associated complications. Induction
appropriately according to patient’s calculated CrCl of therapies are initiated to control acute symptoms. On
46 mL/minute/1.73 m2. symptom control, maintenance therapy is implemented.
If symptom control is not achieved, another treatment
8. Answer: B option should be used and continued until symptom
The following four treatment options with similar improvement and implementation of maintenance
efficacy are recommended for treatment-naive patients therapy. Options for induction therapy for moderate to
with chronic HCV genotype 1a infection: (1) daclatasvir severe CD include corticosteroids, methotrexate, anti-
60 mg and sofosbuvir 400 mg daily for 12 weeks, (2) TNF antibodies, and natalizumab. According to Tables
daily fixed-dose combination of ledipasvir (90 mg)/ 33 and 34 in the chapter, standard therapy includes oral
sofosbuvir (400 mg) for 12 weeks, (3) daily fixed-dose corticosteroids; for those with disease refractory to, or

ACCP Updates in Therapeutics® 2016: Ambulatory Care Pharmacy Preparatory Review and Recertification Course

1-635
Gastrointestinal Disorders

who are intolerant of, standard therapies, additional


options include anti-TNF antibodies, methotrexate, and
natalizumab; therefore, Answer A is incorrect because
antibiotics do not induce remission. Answer C is incorrect
because these agents do not induce remission. Answer D
is not the best option; because the patient has already
received oral and intravenous corticosteroids, switching
to a different formulation will likely be ineffective also.
Answer B is the best option; natalizumab is used to
treat moderately to severely active disease in those with
disease refractory to, or who cannot tolerate, standard
therapies and in those who develop TNFα antibodies.
The dose for induction and maintenance therapy is
300 mg intravenously every 28 days and should be
administered for 1 hour. Because of the rare risk of
PML, all prescribers, infusion centers, and pharmacies
must be enrolled in the TOUCH prescribing program
before prescribing, distributing, or infusing the product.

ACCP Updates in Therapeutics® 2016: Ambulatory Care Pharmacy Preparatory Review and Recertification Course

1-636
Gastrointestinal Disorders

ANSWERS AND EXPLANATIONS TO SELF-ASSESSMENT QUESTIONS


1. Answer: D 4. Answer: B
Lifestyle modifications are aimed at lessening the incidence Hepatic encephalopathy is a diagnosis of exclusion. On
of acid reflux and enhancing esophageal acid clearance. diagnosis, treatment of HE should be initiated. Lactulose
Some modifications include: dietary modifications, smoking is standard-of-care therapy according to the practice
cessation, avoidance of tight-fitting clothes, avoidance guidelines; it should be initiated at 45 mL/hour until
of medications that act on LES, elevation of the head of evacuation occurs and then tapered to achieve 2 or 3
the bed while sleeping, weight loss, and chewing gum to stools daily. Answer A is incorrect; rifaximin is approved
promote salivation. Answer A is incorrect; avoiding food pharmacotherapy for reducing the risk of OHE but is not
consumption within 1 hour of bedtime is not sufficient time first-line therapy and is commonly reserved for patients
– the patient should avoid food consumption within 2-3 hours whose condition does not respond to, or patients who are
of going to bed. Answer B is incorrect; eating peppermints intolerant of, lactulose. Answer B is correct; lactulose
after meals is not a recommended lifestyle modification; is first-line therapy according to the guidelines. Answer
mint can loosen the lower esophageal sphincter. Answer C is incorrect; drugs affecting neurotransmission (e.g.,
C is incorrect; the recommendation is NOT to wear tight- flumazenil and bromocriptine) are not recommended by
fitting clothing. Answer D is correct; smoking cessation is a the guidelines. Use of these agents should be reserved for
recommended lifestyle modification. those with conditions unresponsive or refractory to other
therapies. Answer D is incorrect; protein restriction is no
2. Answer: A longer a standard therapy. Although it is sometimes used
Preventive therapy should be selected according to a during acute episodes, long-term use should be avoided.
combined assessment of GI and CV risk. To calculate the
GI risk, risk factors should be assessed and tabulated (e.g., 5. Answer: D
no risk factors: low risk; one or two risk factors: moderate Individuals with large varices and cirrhosis should
risk; three or more risk factors or having a previous receive primary prophylaxis against variceal bleeding
ulcer complication or concomitant use of corticosteroids with nonselective β-blockers as first-line therapy. The
or anticoagulants: high risk). This patient has two GI guidelines call for titrating the β-blocker to the maximal
risk factors (e.g., age older than 65 years and aspirin dose tolerated. A goal heart rate of 55–60 beats/minute
therapy); she is thus categorized as having moderate GI is reasonable if the blood pressure allows it. Answer A
risk. Cardiovascular risk is defined as either low or high. is incorrect; according to the case, the patient seems
This patient has high CV risk because she takes low-dose to be tolerating propranolol; thus, changing from one
aspirin. Preventive therapy should be selected for someone nonselective β-blocker to another is not the best option.
with moderate GI/high CV risk. Therapy options include Answer B is incorrect; adding a long-acting nitrate
naproxen plus PPI (or misoprostol). Answer A is correct; would likely reduce portal pressure, but studies have
PPI is a recommended therapy. Answer B is incorrect; been inconclusive, and some suggest increased mortality.
standard-dose PPI is recommended therapy, which is Thus, this is not a recommended therapy for primary
once daily, not twice daily. Answer D is incorrect; from prophylaxis. Answer C is incorrect; 1 month of therapy
the GI and CV risk assessment, gastroprotective therapy is without meeting the targets shows that an intervention is
recommended. Answer C is incorrect; although misoprostol necessary; thus, making no changes is incorrect. Answer
is an approved therapy for moderate GI/high CV risk, D is correct; increasing the propranolol dose for someone
the dose indicated is low, which may reduce misoprostol who is tolerating the drug but not yet taking the maximal
adverse events. In addition, it is not dosed often enough. tolerated dose is the best option. Continued follow-up and
The recommended dose is 800 mcg four times daily. evaluation of the need for additional dose adjustments
according to heart rate and blood pressure are necessary.
3. Answer: A
Answer A is correct. Ribavirin is category X and must
not be taken during pregnancy or within 6 months of
the patient or patient’s partner becoming pregnant.
Answers B–D do not include ribavirin in the regimen, so
teratogenicity is not a concern.

ACCP Updates in Therapeutics® 2016: Ambulatory Care Pharmacy Preparatory Review and Recertification Course

1-637
Gastrointestinal Disorders

6. Answer: B disease are not included in the recommendations with


Patients develop virologic breakthrough while receiving respect to serologic testing for immunity. Answer D is
NA therapy because of medication nonadherence or incorrect; health care workers should be tested; however,
the development of antiviral resistance. The guideline testing for anti-HBe is incorrect, as is the time point of
recommendation for patients developing breakthrough 3–4 months.
is counseling regarding medication adherence and
confirmation of breakthrough by retesting HBV DNA 8. Answer: B
in 1–3 months. This case provides no information The GI fellow remembered correctly that there are
regarding the patient’s adherence but does provide two special precautions regarding infliximab prescribing.
different HBV DNA levels that are 5 weeks apart and According to the prescribing information, therapy is
detectable; thus, from the information given, the patient contraindicated in patients with active infection, latent TB
has developed virologic breakthrough while adherent to (untreated), heart failure (New York Heart Association
lamivudine therapy. To manage breakthrough, guidelines class III or IV), recent malignancies, optic neuritis, or
recommend either switching NA agents or adding preexisting demyelinating disorder. In addition, because
another different NA to the current therapy. The specific predisposition to infection may occur with all the anti-
management varies depending on the current NA therapy TNFα agents, the risk-benefit of use should be evaluated
(Table 15). The options for someone currently taking in those with chronic infections. Before initiating therapy,
lamivudine and developing a virologic breakthrough patients should be screened for TB, HBV, and HCV,
include adding adefovir or tenofovir OR discontinuing and patients should be educated to avoid live vaccines
lamivudine and switching to emtricitabine/tenofovir during therapy. Answer B is the best recommendation
(Truvada). Answer B is correct; adding tenofovir to because according to this patient’s medication list, the
lamivudine is recommended in the guidelines. Answer A patient takes entecavir, which suggests the patient has
is incorrect; adding entecavir to the lamivudine regimen a chronic infectious disease (e.g., HBV). Additional
is not an option. Switching to tenofovir monotherapy and information (e.g., the status of the patient’s chronic
discontinuing lamivudine is likely to induce additional infectious disease, whether the disease is active [HBV
mutations, making Answer C incorrect. Answer D is RNA level]) must be obtained to determine whether
also incorrect; this choice also requires a switch from infliximab therapy is indicated for this patient, making
lamivudine to adefovir. The recommendations state Answer B correct. Although premedicating with an
that adefovir should be added to lamivudine; thus, the antihistamine, acetaminophen, and/or corticosteroid
patient would be receiving combination therapy, which is is recommended when administering infliximab, this
different from what is stated in Answer D. is not the best answer. First, it should be determined
whether therapy is even indicated for this patient; thus,
7. Answer: C Answer B is better than Answer A. Infliximab therapy
Serologic testing for immunity is recommended only is associated with exacerbating underlying heart failure
for those whose subsequent clinical management relies and is contraindicated in a subpopulation of individuals
on knowing their status (e.g., health care workers, public with heart failure (class III and IV). However, according
safety workers) because of their high risk of continued to the information in the case, this patient does not have
exposure. The guidelines recommend testing for anti- heart failure, so an echocardiogram is not recommended
HBs concentrations 1–2 months after the last dose of (Answer C is incorrect). Because of infliximab’s action
the vaccine series, so Answer C is correct. Individuals on TNF, it is possible for latent infections such as
with anti-HBs concentrations of 10 IU/L or greater are TB to become reactivated during therapy. Therefore,
considered immune. Answer A is incorrect; although the performing a TB test before initiation is recommended;
serologic test (anti-HBs) and the time point (1–2 months however, this is not the best choice because it should first
after completion of vaccine series) are correct, the patient be determined whether therapy is even indicated for this
population with chronic liver disease is not. Answer B is patient; hence, Answer D is incorrect.
incorrect; the serologic marker (anti-HBe) and the time
point (3–4 months after completion of the vaccine series)
are incorrect. In addition, patients with chronic liver

ACCP Updates in Therapeutics® 2016: Ambulatory Care Pharmacy Preparatory Review and Recertification Course

1-638

You might also like